ASWB Social Work Practice Questions

¡Supera tus tareas y exámenes ahora con Quizwiz!

According to Analytical and Psychoanalytical theories, the individual protects themselves from damage while interfacing with reality (the REAL World) by using defense mechanisms. These defense mechanisms help control anxiety by deflecting and protecting the individual from the demands of the id and superego. Which one of the following BEST describes a defense mechanism? A) Reaction formation. B) Dependence. C) Transitional awareness. D) Diffusion.

A) Reaction formation.

The primary factor for a social worker to consider when setting up a behavioral treatment program for a client is: A. The nature of the dysfunctional behavior B. The outcome desired C. The dysfunctional thoughts behind the behavior D. The triggers for the behavior

A. The nature of the dysfunctional behavior

Which of the following is considered an unacceptable form of advertising? A. personal information about the social worker B. classified advertisement C. professional information about the social worker D. phone directory advertisement

A. personal information about the social worker

In developing an advertisement for group therapy sessions, a social worker in private practice can include all of the following EXCEPT: A. recommendations from former clients B. professional qualifications C. academic degrees D. policy on accepting third party payments.

A. recommendations from former clients----the others are acceptable

"Single system" research designs involve observing one client or system only (n=1) before, during, and after an intervention. Because of their flexibility and capacity to measure change over time, single system designs are frequently used by practitioners to evaluate: a. Their practice. b. Difficult clients. c. Conformation to policy. d. Regulation adherence.

A: Their practice. The evaluation process involves: 1) problem identification (called the "target" of the research); 2) operationalization (selecting indices that represent the problem that can be measured; 3) determining the "phase" (the time over which measurement will occur), including a "baseline phase" (without intervention) and an "intervention phase." This may also include a "time series design," where data is collected at discrete intervals over the course of the study.

A social worker is having difficulty assessing a client's disorder. Symptoms include compulsions, phobias, anxiety, depression, and mood swings. All of these are fairly common problems, so the social worker looks back into the patient's childhood. There she finds a history of sexual abuse by a guardian. This helps the social worker diagnose which disorder? A. Post traumatic stress disorder B. Dissociative disorder C. Bi-polar disorder D. None of the above

Answer: B -The client's history of sexual abuse along with symptoms such as compulsions, phobias, anxiety, depression, and mood swings are indicative of dissociative disorder. Dissociative disorder is also known as multiple personality disorder originating from a childhood trauma. Repeat abuse may cause an inability to recall information. Different identities form in an attempt to put these thoughts in their right place.

A 15-year-old girl has been sent home from school on three occasions for drinking alcohol. Each time, the mother becomes angry and threatens to punish the girl but is convinced by the father to give her another chance. Which of the following terms BEST characterizes the father's behavior in this situation? A Triangulating B Enabling C Advocating D Manipulating

B

A teacher reports to the school social worker that she has a student in her classroom that is always hungry, looks disheveled, and often lays his head on the desk and sleeps through her class. What should the social worker do first? a. discuss the situation with the student's paretns b. talk with the student c. report to department of public health d. ask the teacher to talk with the studen

B

Empirical research on diagnostic patterns has historically demonstrated that Black clients with the same symptom presentation as Whites will often be diagnosed A. less severely. B. more severely. C. the same. D. irrelevant.

B

The most common hallucinations in schizophrenia are A. visual. B. auditory. C. tactual. D. olfactory.

B

When a client is especially receptive a a social worker's interventions and highly positive results seem to occur in a brief period, the client may be A. seeking approval by being particularly accommodating. B. experiencing an acute state of crisis. C. in an anxiety-free state of equilibrium. D. particularly fortunate to be working with a skilled worker.

B

When evaluating client satisfaction with a particular agency service, you are using what type of model or approach: A. systems evaluation model B. outcome model C. process model D. case study approach

B

Which medication would MOST likely be used for treatment of depression? A Prolixin B Zoloft C Haldol D Zyprexa

B

John is hired by a social service agency to assist with the closing of cases which have been open for a long time. After discussing the situation with the executive director he understands that some cases have been open to services for 18 month or more with no real services being provided. The executive director suggests developing a bonus program which would pay a worker $50 for each month they kept their caseload under 35 open cases. This would reduce the number of cases which were open but did not need immediate services. John decides instead to offer a $50 bonus to each worker for every 8 cases they close which has been open for longer than 6 months. Relying on our behavioral background, which schedule of reinforcement is BEST described by John's plan? A) Fixed interval B) Fixed ratio C) Variable interval D) Variable ratio

B) Fixed ratio All of these schedules are called partial reinforcement, as opposed to continual reinforcement. In partial reinforcement, the response is reinforced only part of the time. The benefit to a partial reinforcement schedule is learned behaviors are acquired more slowly with partial reinforcement, but the response is more resistant to extinction.

A social worker is working as part of a team tasked with starting a community restoration effort. Of the following groups, which would be most important to involve first? a. faith based group b. volunteer groups c. web based or virtual groups d. local residents and aged based groups

D

One psychological mechanism that tends to obstruct identification and treatment of alcoholism is A. projection. B. sublimation. C. magical thinking. D. denial.

D

A Gestalt therapist meets with a 32-year-old client presenting with a variety of interpersonal concerns both at work and at home. All of the following interventions could be utilized EXCEPT: A. Analyzing how early childhood relationships impact current relational stressors. B. Using the empty chair technique to converse with people not present in the therapy room. C. Focusing on the here-and-now experience. D. Utilizing the empty chair technique to converse with parts of the client's self

The correct answer is A: Analyzing how early childhood relationships impact current relational stressors. This is not something that would be focused on in Gestalt Therapy since it focuses on the here-and-now.

A hospital patient is referred to social services after she complains of insufficient money for food. After talking with the patient, the worker's diagnosis is that the patient, although from a low income family, is not indigent. She seems to constantly have difficulties managing money and does not appear to handle her funds appropriately. Her diet appears nutritionally adequate. The worker's most appropriate action would be to: a. help the patient obtain assistance from a casework agency for help with money management b. provide continuing casework treatment though the hospital social services department to insure that her diet remains adequate c. suggest to the patient that she apply to the public welfare agency to determine eligibility for public assistance d. reassure the patient that her income can be made to cover her essential needs

a. help the patient obtain assistance from a casework agency for help with money management.

A patient in a psychiatric hospital is stealing food and hoarding towels. A program of positive and negative reinforcement techniques is starting to address this behavior. The stealing and hoarding was reduced as a result. In this incident, the reinforcement technique would be considered the: a. independent variable b. dependent variable c. parallel variable d. causal variable

a. independent variable

When measuring client progress and using a rapid assessment instrument, random error is assessed by: a. instrument reliability b. instrument validity c. external validity d. correlation

a. instrument reliability--reliability, to avoid random error, is key word

A client who has received services for several years in a dialysis unit appears for a routine visit. The nurse notices a markedly changed affect and refers the client to to the social worker. When seeing the social worker, the client seems detached, self-absorbed, and tearful. The social worker should first assess the client's a. life situation and recent changes b. compliance with medical care c. concerns about dying d. feelings about dialysis

a. life situation and recent changes

A 5 year old boy is watching his 18 month old sister have her diapers changed and becomes aggressive. The mother is concerned. What would your intervention be? a. normalize his behavior b. get the family in treatment c. get the 5 year old in treatment d. start treatment with the mother

a. normalize his behavior

You receive a phone call from clients who have a child who is refusing to shower. They report the child cries when they attempt to get him to shower, and when they back away and leave him alone, he stops crying. This is an example of: a. positive reinforcement b. negative reinforcement c. punishment d. modeling

a. positive reinforcement

In Piaget's theory of the cognitive stages of development, a 2-4 year old would be in the... a. pre-operational stages b. concrete operational stages c. sensory motor stages d. formal operational stages

a. pre-operational stages

A husband has been committed to a psychiatric hospital as a result of violent rages and assaultive behaviors directed toward his wife. In speaking with the social worker, the husband blames his wife and threatens to harm her. Later he retracts his threat and asks the social worker not to mention it to anyone. The social worker should FIRST: A notify the police B document the threat and notify the treatment team C request psychological testing to assess degree of danger D explore the client's ambivalence and develop a treatment plan

b

Which statement BEST describes resistance in the beginning stages of treatment, according to psychosocial theory? A The client anticipates criticism from the social worker following self-disclosure. B The client minimizes problems and claims that the presenting problem has been resolved. C The client misinterprets statements made by the social worker. D The client is dependent on the social worker for advice in dealing with problems

b

is afraid to leave her home without her "lucky" locket. She received this locket from her father who is now deceased. She believes that if she leaves her home without the locket, she will be harmed or something else life threatening will happen to her. In addition, she believes that the locket can prevent an event of this type from happening. The term that best describes this behavior is: (a) delusional paranoia (b) ideas of reference (c) hallucinations (d) paranoid ideation

b

Your have accepted a position working in a sheltered workshop. You will be trained by the last social worker before she leaves her post for a new job. The workshop is specifically designed to help individuals suffering from a developmental delay. You would expect the social worker training you to us a primarily focused on a behavioral approach. Of the following interventions, which would she be LEAST LIKELY to use to train you? a. Provide positive reinforcement to strengthen enhance work performance b. Assess current familial relationships c. Create a performance contract with the client d. Review a new client's past workplace behavior

b. Assess current familial relationships

In counseling battered women, social workers should keep in mind that most battered women are a. do not care for the batter and find it easy to leave him. b. believe that they can stop the violence by learning new techniques to control the behavior of their mates c. trust the confidentiality and protection of the agency d. do not experience ambivalence towards the batterer and their relationship

b. believe that they can stop the violence by learning new techniques to control the behavior of their mates

The NASW definition of social work includes an emphasis on a. clinical practice b. both macro and micro level practice c. political action d. continuing education

b. both macro and micro level practice

When services are provided to a patient in the hospital, the social worker makes regular entries in the patient's medical record PRIMARILY to: A demonstrate to the administration that the social service department is providing adequate services B protect the hospital should a legal suit arise C facilitate communication among staff members regarding the care that is being provided to the patient D provide documentation for accountability purposes

c

Which of the following characteristics is usually not found in families in which incestuous relationships have occurred? (a) Enmeshment of family members (b) Distorted patterns of communication (c) Symbiotic mother-child relationships (d) Moralistic attitudes to extramarital affairs

c

Which of the following techniques can assist a multi-problem client during the intake process? A Free association B Interpretation of dreams C Partialization D Reflective consideration of dynamic content

c

A client is diagnosed with obsessive compulsive disorder. It is likely that : a. his personal life will not be affected b. he will be excessively studious c. he will experience performance problems at school and work due to his inability to finish tasks on time d. he will not have low self esteem

c. he will experience performance problems at school and work due to his inability to finish tasks on time

A couple with two children is recently divorced. The mother is given custody of the children and the father does not visit or pay child support. When the mother tries to discipline the child, he constantly complains that his father was a better parent and wold always know how to help him when he needed. He complains that the mother can never understand what he is experiencing and wishes his father was still there. This is an example of: a. rationalization b. denial c. idealization d. intellectualization

c. idealization---Appears the child is dealing with his loss by attributing exaggerated positive qualities to his father.

A social worker employed at a residential youth services agency learns that a fellow employee has multiple convictions for sexual molestation of young children. The agency is unaware of his history. Sexual offenses automatically disqualify applicants for employment in the agency. The social worker's professional and ethical obligation is met by: a. saying and doing nothing b. urging the employee to reveal his history to management c. immediately notifying the responsible agency supervisor of the situation d. notifying the man's probation officer

c. immediately notifying the responsible agency supervisor of the situation.

According to the NASW code of ethics, a social worker's respect for a client's rights to confidentiality a. forbids sharing confidences revealed by clients with others including insurers and supervisors. b. is left to the worker's discretion c. includes sharing information revealed by clients with others, without their consent, for compelling reasons. d. requires workers to keep minimal records since these may not be absolute secure

c. includes sharing information revealed by clients with others, without their consent, for compelling reasons.

If a member of a therapy group breaches the confidentiality of another group member, the social worker should: A have a joint interview with the two clients involved B have an individual session with the client who has been betrayed C terminate the client who breached confidentiality D discuss the breach of confidentiality in the group

d

L. has been seeing a social worker for five sessions. On occasion L. has stated that she doesn't want to go on living. Of the behaviors listed below, which would cause the MOST concern that L. might try to harm herself? a. L. has changed employment b. L. has withdrawn from therapy c. L reports continual recurring cases of the "blues" d. L. reports that she has started giving away her close personal possessions to her friend

d. L. reports that she has started giving away her close personal possessions to her friends-Giving away close personal possessions is the most serious

For rapport to develop, it is important for a social worker to: a. always empathize with the client b. have similar issues as the client c. be congruent in words and behaviors d. be aware of personal biases

d. be aware of personal biases

Which of the following factors contributes the least to the quality of client services provided by a social worker? a. consultation b. self-evaluation c. continuing education d. problem-oriented records

d. problem-oriented records

The children of a 73 year old woman tell a social worker that their mother is growing increasingly forgetful. The social worker should first: (a) tell the children this is common in aging (b) request a psychiatric evaluation (c) admit the woman to a rehab center (d) offer to conduct family therapy

(b) request a psychiatric evaluation

A female social worker has been treating a male client for several weeks when she realizes that she is attracted to her client. At times, she finds herself wondering what it would be like to be with him outside of the therapy session. She discusses this with her supervisor. Her supervisor's first course of action should be to:(a) find out if this has happened before(b) advise the social worker that this is unethical and a must be stopped at once(c) explore the situation with the social worker(d) refer the client to a different therapist

(c) explore the situation with the social worker----- thinking is not ethical- acting on thoughts is

P, an 18 year old female who is generally of normal weight, reports that she has recently lost 12 pounds. Family members report that she has had a history of episodic eating of large quantities of food and abruptly disappears after meal time is complete. As of late, this behavior has become more common and is concerning her family and friends. The MOST likely diagnosis for this client is: (a) atypical eating disorder (b) uncontrolled dieting (c) anorexia nervosa (d) bulimia

(d) bulimia ---Client is of normal weight, eating large amounts of food

A 40-year-old male client explains that he is seeking care because he suffers from panic attacks. He notes that he gets chills, nausea, dizziness, and experiences fear about once a day. A social worker's FIRST step with this client would be to 1. Teach relaxation therapy 2. Set goals for reduction of anxiety 3. Arrange for a doctor's appointment 4. Assess the client's current situation

3. Arrange for a doctor's appointment RU SAFE

You are called to evaluate a with chronic obstructive pulmonary disease. He lives in an assisted living facility, and was brought to the emergency room by his daughter. She had taken him to lunch, and became distressed when he refused to return to the facility. He states he wants to live in his motor home, as he resents the loss of privacy at the facility. His daughter confirms he owns a working, fully self-contained motor home (i.e., stove, shower, refrigerator, etc). He has adequate funds. He plans to park the motor home in a nearby Kampgrounds of America (KOA) campground, where all utilities can be hooked up. He can have food and other supplies delivered. However, it is November and it is unseasonably cold. The doctor confirms that the patient is prone to pneumonia, and the daughter states "he will die if he doesn't return to the facility." The patient refuses to consider any other living situation. In this situation, the social worker should: 1. a. Call the police and have them take the patient back to the facility. 2. b. Call adult protective services for further intervention. 3. c. Allow the patient to move into his motor home. 4. d. Place the patient on an involuntary hold for suicidal behavior

3. c. Allow the patient to move into his motor home. Self determination if there is no imminent risk

A 24 yr old woman complains of depressed mood. She tells the social worker that she has felt this way the past 2-3 yrs. She has little energy for social or work activities, has difficulty making decisions and concentrating, and has poor appetite. There are no delusions or hallucinations. Her most likely DSM IV diagnosis is? (a) Dysthymic disorder (b) Bipolar disorder, depressed (c) Cyclothymic disorder (d) Major depression, recurrent

A

A neighborhood association, concerned about area health and education problems, employs a community worker to explore the problems further and develop new programs. The workers initially establishes relationships with a variety of community representatives and involves residents in preliminary discussions about the problems. As the organization's program develops and membership grows, the worker is likely to be concerned about A. developing leadership. B. developing a formal structure. C. envisioning the results for the members. D. providing support services for organization participants.

A

A nine-year-old boy exposes himself and the other children begin to tease him about this behavior. The more the children tease him the more he does the behavior. What is the best intervention for the social worker to conduct? a. assess the boy for possible sexual abuse b. ask the boy why he is exposing himself c. tell the boy's parents about his behavior d. ask the teacher if this is happening in the classroom as well

A

A psychological, cultural and political orientation to life that both determines and influences how people think, behave, make decisions and understand phenomena is usually called a A. world view. B. culture. C. personal opinion. D. perception.

A

A social worker doing joint or multiple interviewing will focus on A. the interactions between people and with the therapist. B. the basic personality patterns of each individual. C. the issues to be resolved. D. dividing the interviewing time evenly.

A

According to Beck, the treatment modality that seems to be least used in treating clients diagnosed as schizophrenic is A. individual therapy. B. group therapy. C. milieu therapy. D. community therapy.

A

B is a woman who is recently separated from her husband after repeated episodes of physical abuse. Her husband is requesting to visit her and their children. B thinks she would like to reunite with her husband. In therapy with the social worker, B asks the social worker if she would help her to plan for reunification with her husband. The social worker should: (a) help B clarify her reasons for wanting to reunite (b) notify and warn Child Protective Services (c) agree to assist the client with her desires (d) call B's husband and tell him of the possible legal ramifications if he does return

A

Borderline personality disorder is characterized by all of the following characteristics except (a) intense long term relationships (b) primitive delusional fantasies (c) lack of control of aggressive drives (d) self-destructive behavior

A

Generally speaking, addicted clients have poor treatment prognosis when A. they also have borderline or antisocial personality disorder. B. they are over 40. C. they are unemployed or homeless. D. they suffer from an anxiety disorder.

A

IN the therapeutic relationship termination is a core concept that must be addressed. Generally, termination is MOST COMMONLY associated with: a. evaluating goals, objectives and therapeutic accomplishments b. starting the actual plan of action c. setting limits in the sessions d. developing a contract

A

The value of the supervisory session to the social work supervisee is dependent on all the following factors EXCEPT: (a) the supervisor's willingness to disclose areas of weakness in practice skills (b) the supervisee's interpersonal skills in requesting information (c) the supervisor's skill in offering positive feedback (d) the supervisee's ability to identify specific practice competencies

A

When a client who is unused to self-reflection first reveals anxiety-provoking feelings and experiences severe anxiety, the social worker might A. use sustaining procedures, allaying the client's anxiety. B. involve the client in further understanding the dynamics of his or her reactions. C. not expect the client to return for another appointment. D. not discuss the unconscious conflicts with the client.

A

A recently formed board of directors made up representatives from various inner city agencies recently received word that they can open a drug rehabilitation facility in their city. They should FIRST: a. appoint a chairman of the board b. hire an executive director c. find a building d. get a building permit

A a. appoint a chairman of the board---must have a board in place before you can hire an executive director

You are discussing personality disorders with a younger colleague. They say they seem really confusing. You know personality disorders to be rather simple when you break down the initial diagnostic criteria for them and them separate them into their 3 primary groupings. Because of this, you know the BEST answer about personality disorders is... A) By definition, personality disorders must be present continuously since childhood. B) The majority of people diagnosed with one personality disorder meet the diagnostic criteria for at least one other personality disorder. C) Since personality disorders are treated differently than acute disorders, medical conditions, or life stressors by the DSM-IV, they are placed on Axis III. D) Because they are more debilitating, people with personality disorders seek treatment more often than people with acute disorders.

A) By definition, personality disorders must be present continuously since childhood. Personality DOs are an enduring pattern of psychological experience and behavior that differs prominently from cultural expectations, as shown in two or more of: cognition (i.e. perceiving and interpreting the self, other people or events); affect (i.e. the range, intensity, lability, and appropriateness of emotional response); interpersonal functioning; or impulse control.

During a monthly staff meeting, your supervisor asks every social work clinician to explain the difference between post-traumatic stress disorder and acute stress disorder as described in the DSM-IV TR. Your best response is A) The duration of the intrusive thoughts and flashbacks occur for more than one month and can be debilitating. B) the length of time between the trauma and the onset of the symptoms experienced by the client. C) The level of somatization, which can be observed in the responses to stressors or situations. D) the degree of trauma, including the nature of the trauma and any pathology that results from it.

A) The duration of the intrusive thoughts and flashbacks occur for more than one month and can be debilitating.

You have a student ask you about Lawrence Kohlberg's theory of moral development. They state that they know Kohlberg viewed his moral development as being driven by... A) family as the first driver of moral development. B) innate biological drives and are universal C) related specifically to peer interactions. D) school and education rules and religious teachings.

A) family as the first driver of moral development.

You are working with a mother and father who have an 8-year-old male child. During the first family session you notice the child does not seem very expressive. Each time the child begins to express emotions the parents jump in and squash it. Twice while describing an incident at school the child becomes emotional and then parents tell him "get control of yourself" and "boys don't whine." With this type of parenting you would expect to see which of the following symptoms currently? A) high anxiety levels. B) eating disorders. C) psychomotor problems. D) acting-out behavior and somatization

A) high anxiety levels.

Incest and family dynamics have several commonalities. You are evaluating a family who has been referred for possible incest issues. After the evaluation, you have identified several interaction styles and familial roles, which cause you, concern. Which of the following family characteristics would be the BEST indicator incest occurring: A) serious enmeshment in family relationships with highly stylized roles. B) attitudes of permissiveness regarding sexuality. C) permeable boundaries and extreme chaos. D) Relationships that are high in conflict.

A) serious enmeshment in family relationships with highly stylized roles.

You have just completed your 4th session with Matthew, a 35-year-old male who has served two brief prison terms. The first incarceration was for two years and the second for three. He has been out of prison for 7 years. He identifies himself as a devout churchgoer who never misses a service and enjoys being part of his church community. During your sessions, he peppers his language with bible quotes and admonitions. He came to therapy for complaints about stress and anxiety in his life. At this beginning of this session, he admitted that his stress was caused by a letter he received from the State Police Agency declaring him a "person of interest" in a current investigation involving insurance fraud. He stated that while he is an insurance agent he is also very religious. He does not understand why he is being "targeted" in this investigation. Upon questioning, he admits his prior two incarcerations involve fraud and larceny. He also shows no remorse for his prior actions and seems more upset about being arrested and losing his job than disappointing his church friends. You realize you may be dealing with a client who has a past diagnosis of anti-social personality disorder and he is deeply involved in the thinking error known as ... A) fragmented personality B) justifying C) fronting D) grandiosity

A) fragmented personality The fragmented personality thinking error is common in persons with antisocial features. It is a method where they can interpersonal conflict by separating themselves into two personality sets. They have a core belief that they are a good person and therefore could do no wrong. If they do something exploitive or hurtful they can justify it by making the logical leap: "If I am a good person and I hurt someone, they must have done something to deserve it, because I would not hurt them for any reason. They caused it. It has nothing to do with whether or not I am a good person." This thinking error allows them to refuse to look at the inconsistency between their beliefs and actions. B is INCORRECT Justifying is also an externalizing thinking error. It allows the user to place all blame outside of them and therefore be able to avoid responsibility. Statements you may hear which could clue you in on this error would include: "He yelled at me so I had a right to hit him." "She was mean to me so I broke her pottery." C is INCORRECT Fronting occurs when the client creates a persona, which they use to try to convince you they are something or someone they are not. This error is similar to a conscious splitting where they can deny behaviors they have committed by refusing or denying they committed the behaviors. This error responds well to a simple statement that you know they are fronting and they should stop. D is INCORRECT A person using the thinking error "Grandiosity" often has an exaggerated sense of self-importance or ability. They often feel they are the best or the best at doing something. They refuse to process any of their actions, which could conflict with this thinking pattern. This client is minimizing or maximizing the significance of an issue, and it justifies not solving the problem. Statements you may hear from a client involved in this thinking error may include: "I hate school; I could run the classroom better than that stupid teacher." Or "Coach is stupid; I am a better player than him. I should be playing Quarterback!"

An immigrant from Cuba arrives in your office to see you. She is obviously distraught. She speaks very little English, and you in turn speak very little Spanish. The best way to know whether she understands what the social worker is saying is to: A. Have her summarize in her own words what has been said in session B. Tell her to freely ask questions as she needs. C. Summarize what is said in the session on behalf of client. D. Immediately give a referral to a bilingual therapist who can communicate more effectively with her.

A. Have her summarize in her own words what has been said in session

The most proximate health consequence for the infant of a drug and alcohol use among pregnant women is often A. higher risk of birth defects, cardiovascular problems, impaired growth and development, prematurity, low birth weight and stillbirth. B. higher risk of raising children who replicate their parent's behavior. C. prosecution of the mother for child abuse. D. increased risk of mental retardation

A. higher risk of birth defects, cardiovascular problems, impaired growth and development, prematurity, low birth weight and stillbirth. The use of alcohol or drugs during pregnancy places the baby at high risk for multiple birth defects, prematurity and low birth weight. These are the most proximate health consequences.

A woman is referred to a community mental health agency by the local hospital. She was tested twice for AID. Though the test results were negative, she requested additional tests and continues to ask for more AIDS tests. She ask the social worker to intervene with the hospital on her behalf. The social worker should assess A. the reasons for her concern. B. her current mental health diagnosis. C. how to advocate on her behalf with the hospital. D. her AIDS risk behavior.

A. the reasons for her concern. Since the repeated negative results don't seem to resolve the woman's concern, there could be something else that holds the worry in place. The social worker needs to go to the next level of exploration and explore the reasons for her concern.

A plan for change and a new program is being launched in a community. What is the first step the social worker should take to increase engagement of the community? a. make contact with the users beneficiaries of the program b. work with a community board to discuss options c. make contact with state legislators to ensure their support d. post flyers and explanations of the program

Aa. make contact with the users beneficiaries of the program--since the program is being launched the first step would be to contact the users of the service to make sure they are aware of the service.

A client discloses information to a social worker about domestic violence that has involved the injury of a child. At a court proceeding, this is shared as circumstantial evidence. The client attempts to sue the social service agency, declaring her confidentiality was violated. The administrator looks at the case and determines: a. The social worker should never have disclosed this information b. The information should have been forwarded to administration c. The social worker acted appropriately considering the danger d. The social worker wanted to make a bigger case out of the proceeding

Answer - C - Administrators need to ensure the proper information on confidentiality is shared by supervisees with their clients. Potential consequences for disclosing information may occur in a court of law and should occur if there is an imminent danger situation. In this case, the client will lose the battle based on the dangerous situation a child has been placed in during domestic violence altercations.

Quantitative research analyzes data to reveal numbers and statistics. This procedure is not utilized very often in social work practice due to the lack of studies large enough for analytical figures. It is, however, helpful for larger projects. Why is the most widely used research method the qualitative model? a. It helps to determine percentages in data analysis b. It determines the quality of the researcher's data c. It specifies informative comparisons with other research methods d. It gathers in-depth understanding of human behavior

Answer - D - Qualitative researchers strive to gather information on human behavior and the reasoning for such behaviors. This method deals with the "why" and "how" of decision making. Samples are much smaller and allow the social worker to be more focused. Qualitative methods produce information on the particular case studied only.

The juvenile facilities in a city are clearly substandard. As a first attempt to aid in correcting the problem, the social worker should: A mobilize interest groups to explore and recommend alternatives B file a lawsuit to force the city to upgrade the facilities C organize the parents of the juveniles to demand change D upgrade prevention programs to keep more youths out of jai

Answer: A (Key A) is the correct answer in this situation since mobilizing interest groups would be the best starting point to begin assessing the facilities and establishing possible recommendations. (B), (C), and (D) might be done in the future, but these steps would be determined based on the outcome of the interest groups, which must be organized first.

A 15-year-old client has been meeting with a social worker due to a diagnosis of major depression and social anxiety. The client is wheelchair-bound and has been having difficulty managing the limitations of getting around her school building. The teen suggests that her disability causes her to be an outcast and her peers look at her and gossip about her in the hallways. What would be the MOST appropriate response from the social worker? A. Educate the client on her rights under the Americans with Disabilities Act (ADA) so she can advocate for herself. B. Assist the client in processing these emotions and help the client prioritize what issue she would like to focus on in treatment. C. Report the behavior to the principal as the student may be experiencing bullying. D. Provide the staff with psychoeducation on the child's condition and how the school can provide appropriate accommodations.

Answer: B Rationale: Although the client is a minor, it is important to meet them where they are at and respect their right to self determination. Given that the client has identified a variety of different stressors, and has been diagnosed with major depression and social anxiety, it is important to prioritize the issue(s) she would like to focus on in treatment.

Anxiety is described as a response to complications that affect our everyday lives. Examples would be a pending divorce, losing a job, or a child failing school. These are considered psychological triggers that motivate us to try and resolve the problem. When does anxiety go from a normal stage to a severe problem? a When thoughts turn to self-harm b When it interferes with problem solving c When it lasts for a short amount of time d When it alerts one to a problem

Answer: B - Anxiety disorders develop when we have a high response to minor or common problems. Individuals who suffer with this problem feel like the anxiety never ends and it interferes with problem-solving. Medication can help treat this disorder, but psychotherapy is also required.

Bethany was called in to have a conference with her supervisor at work. He wanted to discuss problems being observed with her work day. Many times Bethany has been seen dozing off at her desk. She forgets assignments and due dates for reports, while showing an inability to concentrate on tasks. Bethany explains she has been having headaches and will attempt to do better. What could Bethany being suffering from? Amnesia disorder Chronic fatigue disorder Cognitive delay syndrome Anti-social disorder

Answer: B - Chronic fatigue syndrome is characterized by extreme fatigue that appears to worsen with mental or physical activity. Most times the problem does not improve with rest. Symptoms are difficult to measure but this disorder should be taken seriously. The top eight signs to look for are loss of memory, fatigue, headaches, sore throats, enlarged lymph nodes, muscle pain, joint pain, and extreme exhaustion.

Bipolar disorder used to be called manic depressive disorder. The seriousness of this mental illness leads one to dangerous behaviors such as suicide. Less severe cases tend to damage personal relationships and careers. It is characterized by high and low mood swings that move from mania to depression. How is mania described? A. Sadness, crying, sense of worthlessness, loss of energy B. Restless, talkative, reckless, euphoric C. Anger, hostility, stress, tension D. Happiness, cheerful, excited, pleasure

Answer: B - Mania describes an increasingly restless, energetic, powerful period in one's mental state. These characteristics often cause lavish spending sprees or impulsive sex. This high-flying mood can spiral into irritation, confusion, or the feeling of being trapped.

Of the following types of studies, which are concerned with the effects of drugs on behavior, thought, perception, or mood? A. Pharmaceutical studies B. Psychopharmacological studies C. Psychoactive studies D. Chemical reactions

Answer: B - Psychopharmacology refers to the study of drug-induced changes in thinking, mood and behavior. The most widely studied drugs are those available for treatment of mental disorders. Psychopharmacology is primarily focused on chemical interactions with the brain.

Al is described by friends and family as being self-centered. He has problems with anger but never wants to accept the blame. Al's relationships with two different women failed because he was physically abusive. A social worker might assess this client with reaction formation disorder. Which category of reaction formation disorder would he fall into? A. Internalizers B. Externalizers C. Inhibitors D. All of the above

Answer: B -Al is most likely an externalizer. Externalizers tend to be aggressive towards others and overcompensate for fear of being wrong. Many will not only display anger physically, but also sexually and emotionally.

Sexual abuse, physical abuse, emotional abuse and neglect are attributable to a lack of appropriate parental care. When one of these problem is serious enough to adversely affect the progress and enjoyment of a child's life it is defined as what? A. Emergency removal B. Significant harm C. Poor parenting D. Medical neglect

Answer: B -Significant harm is the main component that justifies intervention in the best interest of a child. There are no absolute criteria to rely on when judging the extent. A single violent episode may constitute significant harm, but usually it is the accumulation of events that leads to damage in a child's development.

Which of the following statements is an assumption of crisis theory? A Anger is a useful dynamic if consciously used. B Insight is a prerequisite for change. C Ability to cope with stress can be learned over time. D Internal conflict prevents change.

Answer: C While options (A), (B), and (D) are correct for other theoretical approaches, option (key C) is the only assumption that is appropriate to crisis theory.

What is the first step in a psychological assessment? A. Family medical history B. Biographical history C. Bio psychosocial history D. Mental evaluation

Answer: C -The first step in a psychological assessment is a bio psychosocial history, which involves identifying chief complaint, past psychological history, present illnesses, and current mental status. Sometimes this information is difficult to obtain if the patient is unwilling to aid in disclosure. Once this information is obtained, neurological and psychological testing can take place. When all information is collected, the patient is informed of the results and treatment begins.

There are psychotherapeutic approaches aimed at solving problems concerning dysfunctional emotions such as post-traumatic stress disorder, obsessive compulsive disorder, and clinical depression. Technique-driven and direct treatments are given for specific psychological disorders. Therapy can be given individually or in a group setting with self-help methods.What is this treatment program called? Evidence-based treatment Exposure therapy Cognitive restructuring treatment Cognitive behavioral therapy

Answer: D Cognitive behavioral therapy was developed through a merging of behavior therapy and cognitive therapy. Both traditions are commonly utilized for focusing on the present issues and ways for alleviating symptoms of a specific disorder. Cognitive behavioral therapy has been evaluated for efficacy and is the treatment of choice for many mental health difficulties.

When we are faced with a physical threat that can result in serious harm or death, the "fight or flight" response is activated. This causes one to defend themselves or run away to escape injury. This same response is felt when problems complicate our lives in a non-physical manner. The loss of a job is a good example. The mind is motivated to resolve the problem and this is called what? Schizophrenia Hallucinatory reaction Anxiety Anti-social behavior

Anxiety We all experience anxiety in one form or another. Most forms of anxiety take on a psychological impact instead of a physical reaction. A mild version of the "fight or flight" response helps an individual resolve the problem. Anxiety disorder occurs when this psychological fear does not go away.

Jacob is a kindergartner at Lewis Elementary School. His teacher, Ms. Shirley, is concerned about specific behaviors the child is exhibiting. Jacob forgets his school supplies constantly and does not appear to be able to focus well. He becomes bored with a task after just a few minutes, cannot complete homework assignments, and does not listen when spoken to. Ms. Shirley feels that his behaviors are becoming a handicap in the classroom and refers him for what kind of diagnosis assessment? Attention Deficit Hyperactivity Disorder Dissociative Behavior Disorder Oppositional Defiant Disorder Developmental Disorder

Attention Deficit Hyperactivity Disorder

A 17-year-old mother of an infant wants to return to high school to complete her senior year with her class. The principal advises the student to remain at home until her child is older and informs her that evening classes are available. The student seeks the services of the school social worker. The social worker should: A assist the student in understanding the principal's recommendation B meet with the principal and student to advocate on the student's behalf C report the matter to the school board D explore with the student alternative educational opportunities

B

A 19-year-old male is progressing well in treatment. His parents, who are paying for his treatment, contact the social worker and say that they want to get information about their son's progress. The social worker should: a. arrange for a joint session with the parents and client b. explain to the parents that because of confidentiality she can neither confirm nor deny information related to their son c. release the information to the parents as requested d. immediately discuss this with the client

B

A hospital social worker sees a 55-year-old Asian woman who is hospitalized for chronic, severe abdominal pain. Tests conducted do not reveal any organic problems. History reveals that she is a refugee from Vietnam who emigrated 15 years ago with her husband and children. The MOST appropriate initial assessment of the client's problem is that she is: A being abused by her spouse B somatizing emotional pain C experiencing acculturation difficulties D dealing with a mid-life crisis

B

A husband and wife who are both in their early 80s, live in a suburban community on the outskirts of a large metropolitan city. The couple's son calls a social worker stating that he is concerned that his parents are at risk. He believes that the home is not sanitary, and neither parent is able to properly meals, etc. The son states that the couple refuses to leave their home and will not consider a more protective/supportive environment. The BEST responses for the social worker is: a. call Adult Protective Services and report the case. b. tell the son to call Adult Protective Services and report the case c. immediately report the case as we are mandated to do this by our code of ethics d. offer to visit the couple and assess the situation to help determine what is needed

B

Henry lives in an apartment with several tenants. He avoids all contact with his neighbors and does not have any friends. When Henry sees a neighbor coming down the hall, he will turn and go the other direction to evade simple conversation or eye contact. His life is very lonely and he makes no effort to rectify the situation. What kind of personality disorder does Henry suffer from? A Anti-social personality disorder B Avoidant personality disorder C Depressive disorder D Bi-polar disorder

B Avoidant personality disorder Answer: B - Henry suffers from Avoidant Personality Disorder, as apparent with the pattern of social inhibition, feelings of inadequacy, sensitivity to negative evaluation, and avoidance of social interaction. People with this disorder consider themselves to be personally unappealing and fear being ridiculed, humiliated, or rejected. Avoidant Personality Disorder is first recognized in early childhood and associated with rejection by peers or parents. Anti - Social does NOT mean "not social" it means being against societal rules and regulations

You have been asked by another social worker for help understanding some of the differences between schizophrenia and schizotypal personality disorder. Which of the four statements is NOT true? A) Both schizophrenics and schizotypals have difficulty on cognitive tasks. B) Both schizophrenics and schizotypals have enlarged ventricles. C) Both schizophrenics and schizotypals experience hallucinations and delusions. D) Both schizophrenics and schizotypals have odd or inappropriate emotional responses.

B) Both schizophrenics and schizotypals have enlarged ventricles.

You have been working with a therapist who is involved in biofeedback and cognitive behavior therapy. They are using a form of thermal (heat) biofeedback to teach a client how to control his blood flow in order to reduce body temperature and reduce his migraine headaches. You know that biofeedback works with the nervous system and particularly with the _____________ Nervous system. A) Sympathetic. B) Parasympathetic. C) Somatic D) Endocrine.

B) Parasympathetic. "Para" = Stop, slow down, calming down

You are working with a younger social worker who asks you about the behavioral model. They are specifically interested in the difference between reinforcement and punishment. You understand the BEST way to answer this question is... A) the stimuli and response mechanism. B) the increase in a behavior versus the decrease in a behavior. C) removing a pleasant or an unpleasant stimuli. D) presenting a pleasant or an unpleasant stimuli.

B) the increase in a behavior versus the decrease in a behavior.

A hospital social worker interviews a couple whose 5 month old infant has recently been diagnosed with muscular dystrophy. In the interview the parents appear reluctant to touch the child. Based on this observation, the social worker should:A. Provide the couple with written information on the disease.B. Explore the parent's reactions and feelings in regard to the illnessC. Refer the couple to a support groupD. Consider temporary placement for the child

B. Explore the parent's reactions and feelings in regard to the illness--- Followed by A

A counselor at a local high school contacts a social worker in the welfare protection agency and describes the situation of a 14-year-old girl reporting sexual abuse by her father. The social worker's FIRST step should be to: A. Remove the child from the home B. Interview the child immediately C. Contact the police D. Contact the family

B. Interview the child immediately

The family therapy theorist who would consider the identified patients behavior to manifest a problem in the family system is A. Erikson. B. Minuchin. C. Parad. D. Vinter.

B. Minuchin.Of all the theorists listed, only Minuchin is associated with family therapy theory, specifically Structural Family therapy. Erikson developed a psychosocial theory of development.

AIDS ACTION best represents an example of A. a self-help organization. B. an advocacy-brokerage program. C. a community development approach. D. a consumer organization.

B. an advocacy-brokerage program. AIDS action defends the interests of individuals with HIV/AIDS, agitates for improved services and more research and provides case services. It is an advocacy organization that also brokers services for clients and insures that they find services.

On several occasions a social worker observes that a co-worker in the agency seems to be under the influence of either drugs or alcohol. On one occasion the worker smelled alcohol. The co-worker is seeing clients. The social worker should A. talk to the administrator. B. first speak to the colleague about obtaining assistance. C. first speak to the supervisor. D. do nothing in anticipation that the worker's problems will be observed by the administrator.

B. first speak to the colleague about obtaining assistance.The new NASW Code of Ethics suggests that workers first try and correct problems with the colleague before discussing problems with management. The worker, however, would not attempt a direct discussion if he or she felt threatened or believed the worker might retaliate.

Supportive therapy would not be used if the client is A. diagnosed as psychotic B. seeking self-awareness C. able to solve problems not adversely affected by the unconscious. D. diagnosed as a schizophrenic.

B. seeking self-awareness Supportive therapy encourages clients to use their strengths. The search for self-awareness promotes anxiety and explores aspects of the self. A strong ego is required. People with psychotic symptoms are struggling with weakened egos and need help improving their functioning. They do not need further threats to ego integrity.

Which of the following best explains the primary reason why the Hawthorne studies are considered important to understanding administrative and organizational theory and behavior? A. showed that people are motivated by incentives B. they stressed the importance of individuals in organizations and how individual perceptions can affect organizational output C. showed that controlled experiments are essential to outcome determination D. looked at task completion times with incentives provided.

B. they stressed the importance of individuals in organizations and how individual perceptions can affect organizational output ---concept of reactivity

A social worker that works primarily with the elderly refers a child to another social worker in her office. In exchange the referring social worker is given a 10% finder's fee. This referral process is consideredA. accepted ethical behaviorB. unacceptable ethical behaviorC. acceptable if limited to 10% finders feeD. acceptable if agreeable by both social workers

B. unacceptable ethical behavior---- considered a split fee and this is unacceptable

5. Per Minuchin, psychosomatic families are most likely to be characterized by which of the following: A. frequent intense open conflicts between family members B. weak boundaries between family members C. family roles that are inflexible and stereotyped D. marked emotional distance between the husband and wife

B. weak boundaries between family members

You are hired by a private practice therapist who operates a court-supervised violent offender treatment program. One of your responsibilities is to screen new client referrals, to ensure that only low-risk, first-time offenders are accepted into the program. In this process, you are to have each client sign a treatment consent form, which also includes a detailed consent for release of information. You note that instead of the usual time and target limits, the form allows information to be released at any time to "any law enforcement agency," "any spouse, ex-spouse, or significant other," "any welfare or abuse protection agency," etc. You ask about the ethics of having clients sign this form, and you are told, "It's a hassle to try and get specific information releases, and the safety of the public is at stake. Use the form." Your BEST response is to: a. Use the form as directed. b. Refuse to use the form. c. Call your licensing board and discuss the form. d. Call law enforcement and discuss the form.

B: Refuse to use the form. No client or client population is beneath the ethical standards of the field. An appropriate information release form stipulates a limited period of time beyond which the form expires, the specific kind of information to be released, the specific purpose for which the information is to be provided, and a specific individual or entity to whom/which the information will be provided. While obtaining an information release is indeed a "hassle" it is the ethical standard of care in the field, and deviation from it can open a practitioner to legal liability. The fact that a given client, or client population, may be unaware of this does not excuse the therapist from using an ethically appropriate form in keeping with expected standards of care. Any limitations to confidentiality-such as mandatory reporting if a client expresses intent to commit a crime or harm another-belong on a treatment consent form, rather than on an information release form.

A disorder that is becoming quite common in today's society affects relationships and work with its long-term chronic patterns. Symptoms include intolerance of being alone, frequent episodes of inappropriate anger, self-injury, and a feeling of emptiness. Individuals with this problem demonstrate impulsiveness with substance abuse, money, shoplifting, and sexual relationships . Chronic fatigue syndrome ADHD Obsessive compulsive behavior Borderline personality disorder

Borderline personality disorder

A community social worker begins work in a neighborhood with a homogeneous, mainly white, working class population. The worker finds that three is little organized effort to improve the schools or any other local institutions. There is, however, a community consensus around the school system's major problems. These problems include ineffective leadership from the school board, a weak, vacillating superintendent and a lack of resources. The most appropriate model to use in considering strategies would be A. social goals. B. social planning. C. locality development. D. social action.

C

A patient complains of sleeplessness, loss of appetite, feeling hopeless, general apathy. To help diagnose a social worker would most need to know (a) medications the client is taking (b) family history of depression (c) how long the symptoms have persisted (d) if the patient lives alone

C

A school social worker sees Susie, a 15-year-old, for academic and behavioral problems. She is easily distracted, overwhelmed by her studies, extremely negative to self and others, mistrustful and critical and resentful of family members. She is strongly resistant to treatment. The social worker is most likely to recommend A. long-term treatment. B. group therapy. C. short-term trial treatment. D. psychoanalysis.

C

A social worker is working with a client who often cannot resist eats sweets when feeling stressed. From a behavioral perspective this behavior is typical in the family of origin and particularly resistant to treatment intervention and extinction. It is safe to say that this behavior has been maintained in the past by: a. consistent punishment b. consistent reinforcement c. intermittent reinforcement d. intermittent punishment

C

According to social learning theory, aggressive play in children is generally to: a. frustration b. genetics c. observational learning d. natural growth process

C

On an unconscious level, according to the theory underlying Ego Psychology, a client seeks help for motives that differ from those on the conscious level. These unconscious motives include A. an overpowering desire to receive mothering. B. reassuring unconscious concerns. C. keeping unconscious desires from consciousness. D. fear of intimacy.

C

One goal in treating a person diagnosed with a character or personality disorder is to A. increase the client's dependence on the worker. B. help him or her ventilate their feelings. C. make the ego-syntonic trait ego-alien. D. provide some concrete services.

C

The contemporary focus of treatment in schizophrenia in patients who have families available to them is A. the etiology of the condition, particularly with regard to family relationships. B. an understanding of the intrapsychic state of the individual. C. to focus on helping families respond to the patient and to help the family and the client use available community resources. D. the relationship among siblings.

C

The developmental tasks of self evaluation and skill learning are associated with what life stage? (a) toddlerhood 2-4 yrs (b) early childhood 4-6 yrs Brooks 2006 3 (c) middle school 6-12 yrs (d) early adolescence 12-18 yrs

C

Social work has a long history of using the ecological perspective in our clinical work. This is the basis for our "person-in-the-environment" approach. You can see the effects of this running throughout our ethical code. You know that according to the ecological perspective, this intervention ... A) should focus on the environment. B) should focus on the social agency is the key to effective treatment. C) should help clients use their own capacities. D) should resolve intrapsychic conflict.

C) should help clients use their own capacities.

While in therapy with a social worker, a client reveals that she is planning to physically harm another client. This other client is also being treated by the same social worker. What is the BEST ethical course of action for the social worker?(a) first, the social worker should alert her supervisor(b) the social worker can do nothing because she is bound by client confidentiality(c) the social worker should first warn the other client(d) the social worker should first tell the police of her client's intentions to harm another client

C. (c) the social worker should first warn the other client

The primary function of management is to A. Attend to the staffing of the organization B. Plan organizational activities C. Achieve the goals of the organization D. Maintain the morale of employees

C. Achieve the goals of the organization

A social worker is helping a client who has had a tumultuous past. The client's main is the way his past has come full circle and resulted in problems in the present. After a thorough assessment, the social worker decides to focus on the here and now. What theory is being followed? A. Bandura theory B. Horney theory C. Gestalt theory D. Freud theory

C. Gestalt theory

There are many different types of groups that social workers lead. The type of group MOST associated with self-improvement and opportunities to expand self-awareness is A. The Therapeutic Group B. The Socialization Group C. The Growth Group D. The Recreational Group

C. The Growth Group

In evaluating the appropriateness of social work services from a non-minority social worker to a minority client, the social worker should consider all the following except:A. The client's level of comfort with the worker allowing client self-discolsureB. The social worker's ability to show accurate empathy and genuineness for the client's situationC. The client's access to his/her own helping networksD. That race or ethnicity may be a barrier to effective practice with the client

C. the client's access to his/her own helping networks

Assertive casework is a technique in which A. the worker encourages the client to be more assertive n his interpersonal relationships. B. the worker is insistent that the client answers all questions to see how he or she functions under stress. C. within a very short time limit, the caseworker asks probing questions and provides direct interventions. D. the worker allows the client to tell the story in piecemeal fashion, allowing time to develop a relationship and increase his or her strength.

C. within a very short time limit, the caseworker asks probing questions and provides direct interventions.This is a brief treatment modality. It's based on assumptions that clients have a certain strength level and that there is only a limited time available to complete tasks.

In direct practice, social workers may utilize cognitive therapy to help a client overcome difficulty by identifying and changing emotional responses, behaviors, and dysfunctional thinking. When these thoughts are questioned, one's feelings about the subject matter are apt to what? Diminish Change Become stronger Get aggressive

Change Cognitive therapy is a form of direct practice that involves helping clients develop skills in identifying distorted thinking, modifying beliefs, changing behaviors, and relating to others in different ways. This consists of testing assumptions the client makes and locating thoughts that become distorted, unhelpful, and unrealistic.

This disorder was a mystery for years and not classified as a disorder until recently. Doctors have realized the severity of the problem and recommend therapy that will eliminate the problem. Symptoms of headaches, sore throats, swollen lymph nodes, muscle pain, memory loss, and extreme exhaustion are quite common with this disorder. Agoraphobia disorder Cognitive behavior disorder Chronic fatigue syndrome Bi-polar disorder

Chronic fatigue syndrome

Lisa suffers from social anxiety and self-treats the disorder by avoiding all situations where she comes into contact with strangers. This has been a problem for Lisa since she was a teenager and has never sought professional help. Social anxiety requires psychological treatment that uses techniques appropriate to the symptoms. What are the techniques offered for therapy? Medication and group counseling Being forced to talk with strangers Avoiding all social situations Cognitive therapy with behavioral interventions

Cognitive therapy with behavioral interventions

Children with this disorder often intimidate, threaten, or bully others. They can be physically cruel to people and animals, often initiating fights. Serious violations to rules start prior to the age of 13 and include running away from home or truancy from school. Some kids will destroy property by setting fires and causing extreme damage. What kind of problem do these symptoms demonstrate? Tourette Syndrome Oppositional defiant disorder Conduct disorder ADHD

Conduct disorder

A DSM-IV diagnosis of substance abuse disorder does not include A. repeated episodes of substance intoxication. B. substance related relationship difficulties at work or within the family. C. legal difficulties related to substance abuse. D. a period of at least two years of involvement.

D

A couple with one 7-year-old child is in the process of a very complicated and stressful divorce. Which of the following circumstances/conditions would most likely place the child at the greatest risk of child abuse? a. if the child refuses to follow directions on occasion b. if the child gets a bad grade in school. c. if the child has Down's Syndrome d. if the child is wetting the bed on occasion

D

A family with a 14-year-old pregnant daughter has come for family therapy. The parents are angry at the daughter and cannot agree with one another as to the course of action they should take. The social worker should FIRST: A establish trust with the daughter B assess the family motivation for services C evaluate the anger level of family members D identify each family member's view of the problem

D

A family with a teenage girl brings their daughter in for counseling due to suspected substance abuse. The daughter is defiant and says it is her parents who need help and she refuses any intervention. What should the social worker do FIRST? A. Refer the daughter for substance abuse counseling. B. Explain to the parents that her attitude is typical for her age C. Assist the daughter to express her anger with her parents D. Explore and clarify what the problem is that will need to be addressed.

D

A school social worker is asked to see a child who often talks to himself in class. The teacher has asked that the social worker help the child to exhibit more appropriate classroom behavior or he will have to be removed from the class. From a behavioral therapy perspective what is the FIRST thing the social worker should do: a. meet individually with the child and explore who he is talking too b. refer the child to a group for children with delayed social skills c. remove the child from the room when he begins to talk to himself d. determine how frequently the child is talking to himself.

D

A social work administrator is designing an after-school tutorial program for school-referred students with learning disabilities. On the administrator's staff are a psychologist and a classroom teacher. Which additional professional should be added to the staff to ensure a comprehensive assessment? A Recreational therapist B School nurse C Reading specialist D Special education teacher

D

In the first session with a school social worker a high school student reveals he has been cutting school and forging his mother's name on absence excuses. The social worker should: (A) inform the student that the social worker has a responsibility to inform his mother (B) avoid negatively reinforcing the attention-seeking behavior (C) suggest that it would be best for him to inform school authorities and accept responsibility for his behavior (D) explore the reasons for the absences and factors contributing to the behavior

D

Which type of interviewing question is MOST likely to generate client defensiveness? A (A)Open B Closed C Leading D Why

D

Severe trauma during early childhood can have the effect of personality states that continually hold power over an individual's behavior. Repetitive physical, sexual, and emotional abuse causes an inability to recall key personal bits of information. Memories tend to vary according to the different identities one assumes. What is the name of this disorder? A Alter-Ego Disorder B Post-Traumatic Stress Syndrome C Bi-Polar Disorder D Dissociative Identity Disorder

D Dissociative Identity Disorder Answer: D - Dissociative Identity Disorder, otherwise known as multiple personality disorder, is a fairly common problem. It is characterized by the two or more distinct "identities" that each have their own age, sex, and race. Symptoms include depression, suicidal tendencies, sleep disorders, compulsions, mood swings, phobias, and anxiety. Personality disorders are identified by their appearance at extremely early ages, many times triggered by trauma.

Bethany feels that a specific method will work best in her social work practice. She completes evaluations and research with her consenting clients. Once the findings are available, it is discovered the method is not as effective as others. Bethany does not want her colleagues to degrade her because of this information and does not share the findings in order to present a more positive approach. Would this be seen as ethical practice? a. Yes, if Bethany can prove it works on another client b. No. The social worker should change the names of the client. c. Yes. Bethany can better serve her clients if the research opens up new programs. d. No. The social worker should never falsify information.

D. No. The social worker should never falsify information.

If a social worker believes she cannot help a client, the best course of action is to: A. disengage from the client B. consult a colleague C. Obtain the education/skills needed to help the client D. Refer the client to another clinician who can provide the needed services

D. Refer the client to another clinician who can provide the needed services -- Always practice in your scope of expertise and when unable to assist always refer

A factor that contributes to the development of a "group-think" mentality within a policy-making committee is A. apathy. B. hostility. C. competition. D. amicability.

D. amicability.Group think suggests unquestioning unanimity and lack of critical thought. Groups that have achieved a high level of personal connection tend to overvalue relationships and can diminish the work functions of the group.

A young boy (age 14) states in session that he was witnessed a crime. He is frightened because some older boys beat up another boy after selling him drugs in his neighborhood. He feels he should tell the police but is afraid to do so. The social workers best ethical course of action is to:A. encourage the boy to contact the policeB. Inform the boy that confidentiality must be broken and report itC. contact the boy's parents and tell them what happenedD. explore the situation further, helping the boy to discuss available options.

D. explore the situation further, helping the boy to discuss available options.

Credit unions, peer tutoring programs, and locally sponsored nonprofit housing corporations are an example of A. consensus strategies. B. community action programs. C. social action efforts D. self-help programs.

D. self-help programs.This is a definitional question. Self-help programs are designed to provide services to people who have similar concerns and problems.

An unstable family relationship has caused Amber to have an overwhelming feeling of low self-esteem. She feels coping with her problems and managing stress requires her to take control of something. With this thought in mind, Amber tends to focus on her body. To master her emotions, she must achieve the ideal thinness to find happiness. What disorder is being described? Genetic predisposition disorder Family dynamic disorder Chronic fatigue syndrome Eating disorders

Eating disorders

Lisa is a social worker who has received a case referred for possible child abuse. She realizes keeping the children safe is her top priority. The household consists of two parents and three children. There are steps to determine if abuse may have occurred or if the family is at-risk. Which kind of assessment should she use to determine the best possible assistance for this referral? Financial assessment Child welfare assessment Parent assessment Family-centered assessment

Family-centered assessment The family-centered assessment focuses on the family as a whole with full participation. All cultures and ethnicities are considered throughout. Families will identify their strengths, needs and resources in an effort to develop a functioning service plan to maintain safety and permanency.

One of the key factors in achieving safety, permanence, and the well-being of a child is the creation of an effective assessment. This should engage the family and child as partners in planning. Face-to-face family team meetings are useful for assessing needs and recognizing strengths. Community and informal support systems may also be involved. What type of assessment is being described? Behavior analysis assessment Oppositional defiant assessment Functional assessment Behavior management assessment

Functional assessment Evidence of a good functional assessment of a child and family will be present when it flows naturally into service planning. The family is engaged in the entire process that is strengths-based, needs-based, and team-based. Functional assessments make use of family systems as relationships and interactions between members.

Today's society and media promotes the ideal body as being extremely thin. This supports the idea that people should have a slender physique to be recognized or liked. Individuals who have low self-esteem believe these notions to be true. They believe gaining control of their emotions requires extreme techniques for losing weight. What are symptoms of an eating disorder? Loss of menstrual cycles Memory loss Lack of concentration Temper tantrums

Loss of menstrual cycles

Michael has felt sad and low many times for brief periods of time. Lately, intense feelings of helplessness, hopelessness, and worthlessness have been lasting for weeks. Experiencing at least five symptoms of this disorder may give one what diagnosis? Insomnia Bi-polar disorder Major depression Post-traumatic stress disorder

Major depression

Since elementary school, Mark has demanded admiration for everything he has accomplished. He tends to exaggerate his achievements and demonstrates an overwhelming desire for success, power, and love. Mark feels he can only be understood by others like him who reign superior in some aspect of life. What disorder does this individual display? Chauvinistic personality disorder Borderline personality disorder Bi-polar disorder Narcissistic personality disorder

Narcissistic personality disorder

.All children will argue, talkback, and disobey adults at some time. This is most noticeable in the normal development of two and three-year-olds. When the behaviors become uncooperative and hostile in a consistent manner, there is a serious issue. When compared with other children of the same developmental level these behaviors affect social, family, and academic life. What diagnosis might align with these issues when seen in older children? Oppositional defiant disorder Cognitive behavioral disorder Autism disorder Attention deficit hyperactivity disorder

Oppositional defiant disorder

During supervision, a social worker talks about personal feelings of inadequacy that surfaced during sessions with a particular client. The social worker's reaction describes: a. Resistance b. Countertransference c. Transference d. Projective identification

Rationale: All the options except (B), countertransference, describe reactions by the client. Countertransference is the answer because it is the only response that deals with the reactions by the social worker.

What is the FIRST thing that social workers should do when attempting to evaluate institutionalized racism in their agencies? a. Analyze the agencies' policies and procedures b. Discuss the issue of fairness with their supervisors c. Review literature on eliminating discriminatory practices d. Review policies that have been successful in other agencies

Rationale: Analyzing policies and procedures (A) is the FIRST action to take initially. After assessing, options B, C, and D may be appropriate. A social worker may conduct research on ways to evaluate agency practices but not on eliminating discrimination (C). It is important to understand agency practices FIRST.

A client is referred to a social worker regarding depression that began eight months earlier. The client reports irritability, sleep problems, and loss of appetite. In exploring the onset of the depression, the social worker learns that the client had an abortion nine months ago. What should the social worker do FIRST? a. Complete a comprehensive eating disorder assessment b. Offer relaxation strategies to address sleep problems c. Refer the client to a grief and bereavement group d. Ask about the meaning of the abortion to the client

Rationale: The social worker should start with the problem presented by the client, which is depression. In assessing the depression, the worker would ask the client to explain her feelings about the abortion (D). Options A, B, and C may all be done later, but the FIRST action is to determine the meaning of the abortion for the client.

The ability to keep personal feelings and needs separate from work relationships BEST represents which social work ethical standard? a. Professional discretion b. Responsibility to the profession c. Professional boundaries d. Responsibility to colleagues

Rationale: This requires knowledge of the definition of professional boundaries(C). Since the question asks for the BEST response, the secondary responsibilitiesin the other options are not correct.

Allan has to give a huge presentation before the board members of his company. He has no problem getting his point across to fellow workers, but is extremely worried about this particular event. Once Allan enters the room and stands in front of his audience, he goes into a panic attack and has to leave. Allan is displaying signs of what problem? Social anxiety Anti-social behavior Paranoia Self-intimidation

Social anxiety

To a psychoanalyst who follows the work of Sigmund Freud, understanding the initial event which leads to obsessive-compulsive behavior is the result of fixation at a specific stage of psychosexual development. The specific stage is BEST labeled is... A) oral B) genital C) phallic D) anal

The correct answer is D Fixations at the anal stage were seen to result in obstinate, rigid, meticulous, compulsive and overly conscientious behaviors.

Calvin is an 8-year-old who has difficulty with involuntary movements and vocalizations. He will stamp his feet while echoing what others say. Calvin has very little control over his behaviors and symptoms appear to be increasing with age. The child needs to complete a series of foot stampings before he seems to calm down and relax. These symptoms mimic what kind of disorder? Narcissistic disorder Anti-social disorder Obsessive compulsive disorder Tourette syndrome

Tourette syndrome

A client has met weekly with a social worker for five weeks. The relationship with the social worker has been positive. During the sixth visit, the client becomes angry and challenging when the social worker makes a clarifying comment. The social worker's BEST response to the client's reaction is to explore: A the client's understanding of the social worker's statement B the client's resistance to change C limits for the client's behavior D the client's use of defense mechanisms

a

A client who expresses no overt anger accuses her social worker of being angry, hostile, suspicious and cold. The process being used is most likely (a) projective identification (b) paranoia (c) reaction formation (d) isolation of affect

a

Who would be most at risk for abusing his/her children? a. A young, single mother who drinks b. a man with a history of impulsive behavior c. a woman who often gets angry with her teenage son d. a woman who was raised in homeless shelters.

a. A young, single mother who drinks Intoxication creates higher probabilities to acting out of impulse/ out of control. B is incorrect because we do not know if the man has worked on his impulsive behavior. Just because this is in his history, does not make him automatically harmful *now* C is wrong because anger =/= abuse D is wrong because homelessness shows no indicator on the personal actions. Homelessness does not increase the likelihood of abuse if there is no past history of harm.

A 15 year-old male teenager is angry with his father. He cannot tell his father how he feels. He plays football and during practice he deliberately tackles a member of the other team fiercely knocking the other player to the ground. While charging the other player he is thinking about his father and the anger he has for him. This is an example of: a. Sublimation b. displacement c. reaction formation d. projection

a. Sublimation

You are a social worker practicing in a rural town. One of the local agencies asks you to test a child for ADHD. You would: a. Tell them you will need to speak with the child's parents b. Test the child c. Tell them you would need to speak with the school d. Refuse

a. Tell them you will need to speak with the child's parents

Which of the following most accurately describes a potential problematic triangulation within the therapeutic context? a. a couples therapist aligning with one spouse b. a family therapist offering individual therapy to one client c. an individual therapist referring one of his client to an outside group d. a child therapist organizing a sep. session with parents.

a. a couples therapist aligning with one spouse

The concept of gender roles is BEST defined as: a. attitudes and behaviors attached to individuals because of their sex b. an individual's sex orientation and/or preference c. biological characteristics that influence an individual's behavior d. interplay of biological and sociocultural influences

a. attitudes and behaviors attached to individuals because of their sex

During a first interview, a social worker observes that the client moves slowly, has a stooped posture, talks slowly and in a lifeless way, lacks spontaneity, and shows little change in facial expression. The worker would most likely suspect: a. depression b. a manic stage c. anxiety d. delusional thinking

a. depression

Which method should social workers use to assess practice activities and outcomes. a. evaluate client's goal attainment b. review their personal performance evaluations c. Keep and review case records and notes d. ask clients for anonymous evaluations.

a. evaluate client's goal attainment

A hospital social worker interviews a couple whose six month old infant has recently been hospitalized for cystic fibrosis. The social worker notices that the parents are reluctant to touch the child. Based on this information, the social worker's first intervention should be to: a. explore the parent's reaction to the child's illness b. refer the couple to a support group c. evaluate the situation for out-of-home placement d. provide the couple with information

a. explore the parent's reaction to the child illness

A client who expresses no overt anger accuses her social worker of being angry, hostile, suspicious and cold. The process being used is most likely a. projective identification b. paranoia c. reaction formation d. isolation of affect

a. projective identification

A lesbian asks a social worker fora referral to a sperm bank because she desires to have a child. What should the social worker do? a. refer her to a sperm bank b. ask her about her decision to become a parent c. complete a full assessment d. talk to her about parental responsibilities

a. refer her to a sperm bank

From a family therapy perspective in assessing family problems, the therapist would be least likely to: a. refer to the diagnostic and statistical manual of mental disorders b. complete a structured family analysis c. complete a developmental history on each member d. have the family complete a family circle/genogram

a. refer to the diagnostic and statistical manual of mental disorders---this is an individual diagnostic tool

During an adult support group, Jerry often speaks sarcastically of Henry's appearance and his inability to communicate well. Joe and Alice are irritated by Jerry's statements and finally tell them that they do not like his behavior. Jerry turns to the social worker and asks him what he did wrong. The social worker would: a. suggest to Jerry that he ask Joe and Alice what they meant b. do nothing c. ask Jerry why he speaks so sarcastically d. ask Jerry if he is aware of what he is doing

a. suggest to Jerry that he ask Joe and Alice what they meant

When conducting supervision for social work licensure, the requirement for acceptable supervisory sessions should contain: a. supervision sessions that are practice oriented and well documented are required b. group supervision without practice application is acceptable c. teaching without practice application is acceptable d. direct case presentations only

a. supervision sessions that are practice oriented and well documented are required

In treating a client in crisis, the caseworker should focus on... a. the immediate presenting problem b. a few specific problems in day-to-day functioning c. a wide range of problems in day to day functioning d. underlying personality problems

a. the immediate presenting problem

According to psychodynamic theory framework, projective identification describes the process of: a. unconsciously attributing and perceiving the behavior of another as a reflection of ones self b. linking characteristics of people in your past with people in your present c. building ego identity in an individual d. utilizing learning theory to address learned behaviors

a. unconsciously attributing and perceiving the behavior of another as a reflection of ones self

A psychological, cultural and political orientation to life that both determines and influences how people think, behave, make decisions and understand phenomena is usually called a: a. world view b. culture c. personal opinion d. perception

a. world view

In an initial interview a client appears suspicious of her surroundings and of the social worker. She says that she wants to reveal certain facts but feels that this would get friends in trouble. The appropriate response for the social worker is to: A encourage the client to talk about these things since she can trust the social worker to maintain confidentiality B tell the client that she understands how hard it is for her to feel safe in this new situation and that she should reveal information when she feels ready C explain to the client how social workers maintain confidentiality and show her how the files are locked to ensure privacy D involve the client in a discussion of a less threatening subject

b

The parents of children placed in foster care have made no progress after extended casework services in improving the behavior that resulted in the children being removed from their home. The child services agency staff has determined that procedures to terminate parental rights will begin in the next few months. The foster care worker's NEXT step should be to: A increase efforts with the parents to modify the parents' behaviors B begin to prepare the children for the transition of having parental rights terminated C ensure that the physical and mental health histories of the children's maternal and paternal families are available D request that a legal guardian for the children be appointed

b

Which of the following is an issue that lawmakers may need to decide? a. Anonymity b. Privilege c. Confidentiality d. Self-determination

b. Privilege Privileged communication in healthcare refers to communications that include private patient information that is protected by federal, state, and local laws in the US and in other countries. Privileged communications exist because society values the privacy or purpose of certain relationships. The established privileged communications are those between wife and husband, clergy and communicant, psychotherapist and patient, physician and patient, and attorney and client. A and C are wrong because -Anonymity and Confidentiality are protected under HIPAA- this is already pre-determined and in the hands of Social Workers to follow. Self determination is a social work value, not applicable in law.

When testifying in court, a social worker is asked specific questions concerning his client, who is a defendant in a criminal case. What concept should guide the social worker: a. The social worker has social work privilege and cannot be asked to provide confidential case information. b. The social work privilege is not absolute and the worker may be required to testify if ordered to do so by the judge. c. The Code of Ethics requires that the worker refuse to testify. d. Agency policy requires the worker to maintain absolute confidentiality

b. The social work privilege is not absolute and the worker may be required to testify if ordered to do so by the judge.

A social work supervisor who practices as a generalist is correctly described as: a. a social work expert in all areas of practice b. a practitioner who practices across specialization lines of the fields of practice population groups and problems c. practitioner who is considered interested in all forms of medical social work practice d. practitioner who specializes in family therapy but practices all types of social work practice

b. a practitioner who practices across specialization lines of the fields of practice population groups and problems

A social worker is interviewing a new patient who presents with some symptoms of depression. The patient reports recent changes such as diminished functioning, marked weight gain, early morning awakening, fatigue, inability to concentrate, and headaches. The patient mentions that he started a new medication for a medical condition three weeks ago. The worker would first. a. arrange a psychiatric consultation in order to have an anti-depressant prescribed b. ascertain the prescribed medication and investigate its side-effects c. proceed with a thorough psychosocial history and precipitating events d. hospitalize the patient

b. ascertain the prescribed medication and investigate its side-effects

During an initial interview with a social worker, a client begins to express suicidal ideation. What should the social worker's first response be. a. recommend hospitalization b. assess the client's level of risk c. establish a behavioral contract d. provide supportive reassurance

b. assess the client's level of risk

A social worker uses self-disclosure most appropriately during an interview with a client to a. communicate empathy b. facilitate discussion of sensitive topics c. express genuineness d. express positive feelings about clients

b. facilitate discussion of sensitive topics

When a client is suspected to have Tourette's disorder, the social worker's first referral should be to: a. a psychotherapist b. neurologist c. pathologist d. long-term treatment program

b. neurologist

A social worker has been contacted to evaluate a four-year-old boy. His teacher claims that he is bullying the other children and often makes menacing or sexual gestures toward the other children. The social worker is most likely to suspect: a. Attention Deficit Hyperactivity Disorder b. possible sexual abuse c. parental neglect d. conduct disorder

b. possible sexual abuse---not enough to substantiate conduct disorder but often children who act out sexually have a history of childhood sexual abuse

A middle aged women complaining of recent lack of motivation is seen by a social worker in a mental health agency. The worker would first rule out a. depression b. situational stress c. anxiety d. bi-polar

b. situational stress

A 28 Yr old client with a long standing history of drug use is referred to a social worker by a concerned relative. In the assessment interview, the client tells the social worker about frequent cocaine use. The social worker should FIRST: (A) conduct a family interview to evaluate the family's concern (B) begin treatment, focusing on the reason for drug abuse (C) refer the client to a substance abuse program (D) evaluate the client's motivation for change

c

The initial symptoms of schizophrenia MOST often occur between which of the following years of age? a. 6 to 12 b. 13 to 16 c. 17 to 45 d. 16 to 60

c. 17 to 45

a patient experiencing dysthymic disorder will: a. Vividly describe hallucinations that began at least three months ago b. Have associated eating disorders that began at least six months ago c. Experience chronic depression on most days for two years or longer d. Probably be drug dependent

c. Experience chronic depression on most days for two years or longer Dysthymia low grade, chronic depression.

The concepts of "pre-affiliation" (becoming acquainted), "power and control" (setting the roles), "intimacy" (developing cohesion), "differentiation" (independent opinion expression), and "separation" (moving to closure and termination) are all stages in: a. The lifecycle of a therapeutic relationship. b. General relationship cycles. c. Group development. d. Team cohesion.

c. Group development. During these stages, the social worker needs to: 1) facilitate familiarity and elicit participation; 2) clarify roles; 3) develop group cohesion; 4) support individual differences; and 5) foster independence. The use of a "Sociogram" (a chart or diagram depicting group member relationships) can aid the social worker in revealing, monitoring, and intervening (if necessary) in group member interactions and bonding.

John, a Japanese immigrant, comes into the office with his 12 year old child, Amanda. Amanda asks if she can be used as an interpreter due to the family's imminent need for housing services. The social worker's best course of action would be to... a. allow Amanda to interpret just for this session and then hire a professional interpreter for future sessions. b. refer John and Amanda to a Japanese Social Worker. c. Hire a trained interpreter and proceed as soon as possible. d. Try and communicate through an online interpretation device.

c. Hire a trained interpreter and proceed as soon as possible.

A social worker has a bad home visit where the clients were uncooperative and kept arguing with one another. Within the documentation, the social worker used derogatory language about the clients. When the case came up for review, this information was discovered by administration. What course of action should be taken? a. The social worker needs anger management counseling b. The social worker should be suspended without pay c. The social worker would be advised to review the NASW Code of Ethics d. The social worker should allow the clients to view the documentation

c. The social worker would be advised to review the NASW Code of Ethics Answer - C - An administrator dealing with this situation needs to express the importance of using accurate and respectful language in all communication about clients. The NASW Code of Ethics stresses that social workers should not use derogatory language in their written or verbal statements.

A family comes to an agency complaining of their child's behavior problems. As a result of the assessment phase, the social worker believes that the marital relationship is the primary problem. The social worker should focus on: a. the parents, confronting them with their relationship issues b. the child and the ways the parents should react to the behavior c. a discussion of the social worker's perception of the marital relationship d. developing the child's insight into behavior as a reaction to marital stress

c. a discussion of the social worker's perception of the marital relationship

Clients in a mental health agency group are encouraged by the group leader to socialize even when not participating in the group sessions. The group is probably: a. an analytically oriented group b. a behavioral group c. a supportive group d. a transactional group

c. a supportive group

A child was sexually abused and the family lost their child and the state took custody. They have been in therapy and the court thinks it is dafe for the child to be returned to the parents. The social worker is unhappy with this decision and does not believe is is in the best interest of the child. What should the social worker do? a. insist that custody not be granted b. let the child wait until he is 18 and then reunite with the parents c. abide by the courts decision d. request another judge

c. abide by the courts decision---must follow court orders

Slurred speech, an unsteady gait, impaired attention, sudden mood changes, and impared judgement are symptoms frequently found in a. anxiety disorder b. ADHD c. alcohol intoxication d. schizophrenia

c. alcohol intoxication

In working with an acutely depressed client, a social worker's main concern in a first interview is a. formulating the correct diagnosis b. referring the client to a psychiatrist c. determining suicidal intent d. selecting the best medication

c. determining suicidal intent

A social worker is co-leading a group for individuals that are substance addicted. Prior to the start of the group she is approached by one of the members. A male participating in the group who is gay complains openly that no one else in the group is gay. He is upset about this and feels he cannot relate what should the social worker do? a. tell him to keep his sexual preferences private b. assess his anger related to the subject matter outside of the group setting c. encourage him to voice his concerns to the group d. tel him that the purpose of the group is treatment and he will have to adjust.

c. encourage him to voice his concerns to the group

Supervision has generally been related to social work practice primarily as a means of: a. in-service education b. measuring worker effectiveness c. ensuring the delivery of ethical, quality, effective and efficient social work services d. social control

c. ensuring the delivery of ethical, quality, effective and efficient social work services

Mrs. W has a long history of alcohol abuse. Though she has made several failed attempts to quit, she is now convinced that alcohol is affecting her heart and liver and she decides to stop drinking. She refuses medical supervision. Several days after deciding to quit, she arrives at the clinic without making an appointment and demands to see her social worker. She is loud and demanding and seems disheveled and uncoordinated. She then abruptly leaves the clinic. The social worker might conclude that Mrs. W : a. is going through alcohol withdrawal and needs medical attention b. wants the social worker's help in managing her discomfort c. is intoxicated d. may be experiencing delusions

c. is intoxicated Signs of intoxication include: Exhibiting signs of irritability and extreme mood swings., Slurred speech, Lack of coordination, and Trouble with motor coordination or an inability to walk properly. A is wrong: Withdrawal symptoms can include sweating, tremors, sleep problems, rapid heartbeat, nausea and vomiting, hallucinations, anxiety, restlessness, and possibly even seizures

The developmental task for self evaluation and skill learning are associate with what life stage? a. toddlerhood 2-4 yr b. early childhood 4-6 yr c. middle childhood 6-12 yr d. early adolescence 12-18 yr

c. middle childhood 6-12 yr

The two most common defenses used by couples with marital problems are a. repression and denial b. repression and suppression c. projection and displacement d. projection and splitting

c. projection and displacement

In the first interview with a battered woman who wants to leave her violent husband, the social worker should be most aware a. that women often make false claims of violence to improve their situation in custody and financial battles associated with divorce. b. that family treatment should be encouraged as it can reduce the levels of violence c. that the woman may be in great danger, as men who batter are often enraged by the woman's effort to seek help. d. that court orders of protection, social services, and community supports are readily accessible and available to most battered women.

c. that the woman may be in great danger, as men who batter are often enraged by the woman's effort to seek help.

An adult client with no strong family ties is preparing for discharge from a psychiatric hospital. Which of the following social work activities will BEST contribute to the client's successful reentry into the community? A Helping the client locate appropriate housing B Ensuring that the client has achieved maximum hospital benefit C Helping the client to apply for financial assistance D Collaborating with community agencies

d

Common indicators of sexual, physical, and emotional abuse include all of the following features EXCEPT: A violence against younger children B sexualized behavior C isolation D stuttering

d

Mrs. W asks for help with her 6 yr old daughters frequent temper tantrums. The tantrums often occur when shopping. When the child sees something she wants she screams and kicks until her mother gives her the item. When asked how she responds, Mrs. W says she usually gives in to avoid being embarrassed. Which behavior principle is demonstrated? (a) The child is negatively reinforcing mother's compliance with the child's demands (b) The child is positively reinforcing mother's giving in (c) Mother is negatively reinforcing the child's behavior (d) Mother is positively reinforcing the child's behavior

d

The concept of secondary gain in the psychodynamic approach is MOST congruent with the social learning theory concept of: A punishment B extinction C discrimination D learning reinforcement

d

In the beginning of treatment for divorce adjustment, which of the following worker statements would be least effective in regard to initial problem assessment and identification? a. "Why have you come to see me"? b. "Tell me how you responded when you decided to divorce"? c. "After she told you, what did you do"? d. "How many times have you been divorced"?

d. "How many times have you been divorced"?--- focuses on history and not assessment

Autism is a spectrum disorder defined by a certain set of behaviors that children and adults exhibit in varying ways. The main symptom of autism is the abnormal or impaired development of communication and social interaction. Two people with the same diagnosis may display any combination of behaviors in any degree of severity. What criteria are needed on an assessment for an autism diagnosis? a. Failure to develop peer relationships b. Delay in the spoken language c. Lack of social or emotional reciprocity d. All of the above

d. All of the above

A 40-year-old male client explains that he is seeking care because he suffers from panic attacks. He notes that he gets chills, nausea, dizziness, and experiences fear about once a day. A social worker's FIRST step with this client would be to a. Teach relaxation therapy b. Set goals for reduction of anxiety c. Arrange for a doctor's appointment d. Assess the client's current situation

d. Assess the client's current situation

A client expresses her disappointment at not being called back after a job interview and vents her anger at the job interviewer. She informs her social worker of this exchange. The social worker's best response is: a. You have to be patient in a job search b. I can see that you're upset about this c. I wonder how you feel about this d. I guess you're hurt and angry about not getting the job

d. I guess you're hurt and angry about not getting the job A. is wrong because you are giving advice (FARM GRITS ROAD) B is wrong because the client "vents her anger." Do not assume what is not in the question. Nowhere does she say she is upset. C is wrong because venting does not always require further analysis, but it is important for SWs to clarify what the client means.

A social worker answered a call on a suicidal hotline from a call her who is sarcastic and hostile. The worker's best response is: a. I have no time for this b. I need to keep this line open, so please get to the point or hang up. c. I am going to hang up, so you can call back when you are serious d. I know it was hard for you to make this call, but could you tell me more about the reason for this call

d. I know it was hard for you to make this call, but could you tell me more about the reason for this call

A man dies and, after hearing of this, a relative living in another state experiences chest pains. Which of the following describes this reaction: a. Projection b. Reaction formation c. Sublimation d. Introjection

d. Introjection this is the unconscious adoption of the ideas or attitudes of others.

Which of the below questions is not evidence of discrimination in a job interview or job application process: a. Please send a photograph with the application b. What is your birthdate? c. What is your husband's occupation? d. List your previous employment

d. List your previous employment

When a client seems overwhelmed or uncertain how to share further, it can help to break down the concerns at hand into smaller, more manageable parts. This communication technique is known as: a. Fragmentation. b. Sequestration. c. Downsizing. d. Partialization.

d. Partialization.

A client is told by her doctor that she has breast cancer. She is distraught and says she doesn't know what to do. She has 6 children. Which of the following would you do? a. Refer her to a support group b. Refer her for further medical tests c. Encourage her to make provisions for her children d. Provide supportive therapy

d. Provide supportive therapy

Andrea is the mother of three children and has been receiving social work services for almost a year now. She feels there is no longer a need for the service and wants to see her records. The social worker is not sure how to handle this situation since some of the information found within may cause emotional harm to the client. The social worker tries to explain this to the mother, but eventually Andrea contacts an administrator. What kind of response would a supervisor give to this client? a. The client is not allowed to see any documentation recorded by a social worker b. The client will need to obtain a court order to secure the records c. The social worker should just hand over all records to the client d. The social worker should go over the records with the client to prevent confusion

d. The social worker should go over the records with the client to prevent confusion Clients should only be limited in their access to documentation when the information may cause harm to the individual. Administrators will normally look at the records first to take steps in protecting the confidentiality of other individuals that may be identified.

A client is in the hospital and an attorney calls requesting his records. What are your ethical obligations? a. To turn over the records b. To claim privilege c. To ask the client to sign a release form d. To maintain confidentiality

d. To maintain confidentiality

When faced by a hostile client in an agency setting, it is best for the social worker to a. suggest that the client's attitude is making the situation worse b. accept the client's hostility and talk about non-threatening topics c. set limits and structure on the interview session d. acknowledge the client's feelings and encourage discussion of them

d. acknowledge the client's feelings and encourage discussion of them

The contract is a basic concept in social work practice. Its major features do not include. a. mutual agreement b. differential participation in the intervention process c. reciprocal accountability d. free association

d. free association

During Mrs. S's first appointment in a family agency, the social worker encourages her to express her feelings about the recent placement of her child in a residential facility for the developmentally disabled. Mrs. S talks at length about her physcial health problems. The social worker should, a. use Mrs. S's remarks about her health to identify unresolved parenting issues b. refocus the interview on the child's behavior and the reason for the residential placement c. explore with Mrs. S how her health problems are related to the recent placement of her child. d. listen to Mrs. S's description of her health as a beginning of a relationship and means of assessing her self-perception.

d. listen to Mrs. S's description of her health as a beginning of a relationship and means of assessing her self-perception.

There has been a traumatic event in your town. You have been instrumental in providing services. You would like to publicize the work you did. You would: a. write your congressman and ask him to issue a public statement about your work b. call the newspaper and ask them to do a story c. write your local social work board d. nothing - this would be unethical

d. nothing - this would be unethical

A social worker meets with a refugee couple in the hospital soon after the birth of their baby. The infant has Down syndrome and cardiac problems. The couple asks that the child be allowed to die. The social worker should first: a. report the case ot child protection services becuause of the at-risk nature of the situation b. offer the parents the option of relinquishing the child for adoption c. encourage the parents to meet with other parents of Down syndrome children d. perform a biopsychosocial assessment including cultural, religious and familial factors.

d. perform a biopsychosocial assessment including cultural, religious and familial factors

A social worker meets a recently widowed 55 year old friend in the supermarket and talks with her briefly. She seems sad and says she has little appetite for food and reduced interest in her usual activities. She says she has a much diminished desire to visit with friends, though she is usually very social. The most probable DSM-IV diagnosis is. a. depression b. post-menopausal depression c. grief reaction d. there is no DSM-IV diagnosis

d. there is no DSM-IV diagnosis

Which type of interviewing question is MOST likely to generate client defensiveness? a. open ended b. closed c. leading d. why

d. why

While in their second session, a woman, who is having difficulty controlling her children, asks the worker if she has any children. The BEST response for the worker would be:(a) " Yes I have children, but I am wondering why you ask?"(b) "No, it is your children we are discussing"(c) Avoid the question and redirect it back as "It sounds like your children are causing you concern"(d) "I am not allowed to discuss my life with you"

(a) " Yes I have children, but I am wondering why you ask?"---Answer the question but always relate any further comment back to the client.

A couple present complaining of difficulty with intimacy. The social worker should FIRST ask: (a) what initially interested you in each other (b) how do you usually resolve problems (c) tell me about your sex life before marriage (d) how long have you been married

(a) what initially interested you in each other-this is how you start to address the problem solving process

R comes to group and appears withdrawn, non-verbal, and aloof. W, another group member is agitated by R and begins to aggressively tell R and the rest of the group, how R's behavior is not acceptable. The social worker should: (a) let the group members attack each other verbally (b) ask the group members to slow down and assess their feelings towards R (c) terminate the group immediately (d) let the group continue to maintain its own balance

(b) ask the group members to slow down and assess their feelings towards R---aggression is not conducive for group cohesion

L has a child who was recently diagnosed with a terminal illness. L. tells the social worker that her child is not ill and will not be returning to the clinic for follow-up. Which of the following defense mechanisms is BEST represented by L's response? (a) rationalization (b) denial (c) intellectualization (d) projective identification

(b) denial---Mother refuses to acknowledge the situation

A client, J, is absent from two consecutive group sessions without contacting the social worker. On J's return to the group at next session, the social worker should: (a) terminate J from the group (b) encourage the group to address J's absences (c) welcome J back to the group (d) ignore J's absences and proceed with the group session

(b) encourage the group to address J's absences

As a social worker, you are about to finish a session with a father and son. As you are preparing to close the session, you notice that the child has a cigarette burn on his arm. The first course of action for the social worker would be:(a) contact Child Protective Services about potential abuse(b) ask the father how the injury occurred(c) wait till the next session to explore the situation(d) ask the child in private how the burn occurred

(d) ask the child in private how the burn occurred----- If confronted the father may feel pressured to lie about the injury. The ideal situation is to ask the child alone. If that was not a choice, it would be better to either extend the session and discuss it or consider making it an agenda item at the next meeting or try to contact the child setting up a meeting without the father.

A family is seen in treatment, consisting of a father (age 45, a long distance truck driver), a mother (age 43, housewife), a son (age 22, living at home recovering from alcohol and drug problem), and a daughter (age 15, recently failing in school). The mother has made the appointment because the daughter is refusing to try to pass her subjects in school, and the usual methods of discipline and motivation are not working. Early in therapy, the worker asks the family to complete a family sculpting exercise, the primary reason for this is: (a) reduce isolation of the family members (b) help the father see he is absent in decision making (c) break up poor relations between mother and daughter (d) increase family awareness of communication styles/patterns

(d) increase family awareness of communication styles/patterns

A supervisor hears a supervisee telling a client in the lobby that they must arrive for appointments on time or they will no longer be treated. The supervisor should FIRST: (a) ignore it and allow the conversation to continue (b) immediately confront the supervisee and let them know this is inappropriate (c) do nothing as this may be a legitimate exchange (d) later in the day discuss your concerns with the supervisee

(d) later in the day discuss your concerns with the supervisee --- clarify first and address

Supervisors generally are required to provide a variety of services for those they supervise. Services included generally consist of all the following EXCEPT: (a) education (b) assistance with reinterpretation (c) active problem-solving (d) parental guidance when warranted

(d) parental guidance when warranted ----NEVER DO THIS

P is a 12 year old male child who suffers from Autistic Disorder. While talking about managing the child with the mother, the mother reveals that the child often exhibits abnormal social play and when he feels threatened, he will hit his head against the wall. The social worker CORRECTLY tells the mother the following: (a) this behavior may be reflective of a dysfunctional family pattern of learned behavior (b) this child needs immediate hospitalization to control these outbursts (c) the parents should not try to stop the child when he is hitting his head because it might cause the behavior to increase (d) unfortunately, these behaviors can be common signs and symptoms of the condition that will need to be addressed through behavior modification techniques

(d) unfortunately, these behaviors can be common signs and symptoms of the condition that will need to be addressed through behavior modification techniques

A client who has received services for several years in a dialysis unit appears for a routine visit. The nurse notices a markedly changed affect and refers the client to the social worker. When seeing the social worker, the client seems detached, self-absorbed, and tearful. The social worker should FIRST assess the client's: (A) life situation and recent changes (B) compliance with medical care (C) concerns about dying (D) feelings about dialysis

A

A firm, fixed idea that is not amenable to rational explanation and is held despite evidence to the contrary is BEST defined as a: (a) delusion (b) illusion (c) hallucination (d) denial

A

A hospital social worker interviews a couple whose six month old infant has recently been hospitalized for cystic fibrosis. The social worker notices that the parents are reluctant to touch the child. Based on this information, the social worker's FIRST intervention should be to: (A) explore the parent's reaction to the child illness (B) refer the couple to a support group (C evaluate the situation for out-of-home placement (D) provide the couple with information

A

A husband and wife who have eleven years of drug recovery present for therapy. The report that they are frequently arguing and connot understand why. The social worker should first: a. assess for relapse of one or both parents b. inquire about domestic violence c. work on communication techniques d. refer for substance abuse counseling

A

A 45 yr old recently divorced mail carrier complains his memory and logic skills have diminished. He is on a new delivery route and is embarrassed that he can't visualize the exact boundaries of the route or calculate his workload and schedules. These are important factors in his job and he is worried. He has developed a slight limp and tires easily. What would the social worker first consider?

A

A 67-year-old who has recently retired seeks therapy with a social worker because of confusion, irritability, and inability to concentrate. The social worker should FIRST: A refer the client for a comprehensive physical exam B assign the client to a support group for recent retirees C assure the client these symptoms are common following a major life change D explore adequacy of plans made for retirement

A

A Latino child has been referred to a mental health clinic because of disruptive behavior at school. The mother speaks limited English, is confused, and does not understand the referral or the school's concerns. In the referral to a Spanish-speaking social worker, the child's male teacher notes that the mother will not communicate with him and seems uninterested. To facilitate an assessment, the social worker's NEXT step is to: A arrange to go with the mother for a school conference B observe the child in a classroom setting C refer the child for a psychological evaluation D schedule an appointment for a family assessment

A

A PRIMARY strategy for social workers interested in community building, networking, and local level problem resolution is: A interorganizational collaboration B interagency consultation C an ad hoc agency committee utilization D force field analysis employment

A

A client has mutually established treatment goals with the social worker and is progressing well in treatment. She announced that she is also in therapy with another social worker. The social worker should: a. explore with the client why she sees the need to do that b. tell her that therapy can be less effective when going to two therapists at the same time c. ask her to contact the other social worker so that you can talk with her d. discourage her form doing this

A

A client informs the social worker that while receiving marital therapy from another social worker in the same agency, a business relationship developed between the therapist and the client's spouse. The social worker's FIRST course of action should be to: A discuss the alleged ethical violation with the colleague B assist the client in filing a complaint with the licensing board C contact the client's spouse to obtain more information D file a complaint with the agency

A

A client is receiving treatment at an in-patient facility for an alcohol problem and is given the medication Antabuse (Disulfiram). When the client takes his first drink, he reacts by vomiting and states that he will never drink again. In this example the medication is a: a. positive reinforcer b. negative reinforcer c. negative stimulus d. positive development

A

A client states "I am really upset". The social worker repeats the same words saying, "you are telling me that you are really upset". This is an example of: a. accurate reflection b. reframing c. projection d. validation

A

A mother has been found to have neglected her child. Child Protective Services have decided that it is in the best interest to remove the child from the mother's care. A family friend volunteers to take the child. Upon interview, it is established that this family friend has had charges pending against her for child maltreatment in the past. The social worker should FIRST: (a) place the child in an emergency foster care group home (b) immediately assess the placement situation for current problems (c) ask the child's mother which placement option she prefers (d) initiate paperwork for another more permanent foster placement

A

A social worker has been treating a client for two months in an agency setting and is experiencing intense sexual attraction toward the client. The social worker's BEST response is to: A consult with an agency supervisor about the case B discuss the attraction with the client C transfer the client to another agency worker D discuss the case with colleagues for feedback

A

A social worker is seeing an adolescent referred by his parents who are concerned about his angry outbursts related to an intense hatred for homosexuals. After several sessions with the social worker, the adolescent acknowledges having homosexual fantasies but continues to articulate his hatred for gays. Which defense mechanism is most likely being displayed? a. Reaction formation b. denial c. sublimation d. displacement

A

A woman does not like her neighbor referred to as M and is constantly yelling, arguing, fighting and finding fault with M. She believes all of the other neighbors in living near here fell the same way about M. She tells the neighbor M that the other neighbors do not like her as well. When discussion her intense feelings for M., the other neighbors are often strangely silent and do no voice the same concerns. What is the client most likely experiencing? a. projection b. projective identification c. displacement d. reaction formation

A

A worker believes that her supervisor is regularly abusing alcohol and is sometimes incapacitated at work. During one supervisory meeting, the supervisor is clearly intoxicated. Her speech is slurred and she is unable to respond adequately to questions. The odor of alcohol is on her breath. The social worker's best first step is to A. discuss the problem with the supervisor and ask her to seek assistance. B. discuss the supervisor's problem at the next staff unit meeting and urge the supervisor to seek help. C. report the problem to the administration. D. refuse to meet with the supervisor if she appears intoxicated at subsequent meetings.

A

A young man separates from his wife. He goes back to live with his parents saying that he cannot afford an apartment. He argues with his mothers incessantly and make many demands of her. He gives the impression that he has not separated from her. The social worker would first work with A. the man alone. B. the man and his mother. C. the man and his wife. D. his mother.

A

According to psychodynamic theory, when a client relates to the social worker as though the social worker is someone from the client's distant past, the client is experiencing:A transferenceB projectionC conversion reactionD introjection

A

An 80-year-old client has a double below-the-knee amputation which requires weekly medical attention. The client is living with her son but wants to relocate to an area with limited support services despite family wishes. She has requested assistance from the hospital social worker. The hospital social worker should FIRST: A consult with the medical team to determine the client's medical needs B meet with the family to explore their concerns C develop a plan with the patient to facilitate the move D provide the client with information obtained from agencies in the relocation area

A

In dealing with a client's guilt, the worker would A. accept and understand the client's guilt, allowing him or her to ventilate. B. always relieve the client's guilt. C. help the client see that guilt is very unhealthy and find ways to rid the client of guilt. D. help the client see what he or she has done.

A

In setting goals and objectives in the therapeutic setting it is MOST IMPORTANT for goals and objectives to be: a. relevant to the presenting problem b. mutually negotiated by the client and the social worker c. relevant to the area of expertise of the social worker d. concrete and easily addressable

A

J exhibits repeated muscle twitches that he reports are beyond his control. Also, when he gets excited, he has a very difficult time speaking and often repeats his word over and over again. This can be very frustrating to J's family and school teachers because often times, he curses repeatedly at other teachers, students, family, and friends. The BEST preliminary diagnosis for J is: (a) Tourette's Syndrome (b) Pica (c) PTSD (d) Petite Mal Epilepsy

A

L, a college student, sees a social worker for counseling. In the first session, L states that she needs a medication prescription to help calm her. L informs the social worker that she is having trouble sleeping and concentrating in school. She believes all of her mental and physical health complaints started after the recent separation of her parents. The social worker's FIRST consideration should be to establish: (a) if L has recently had a physical, and what is her general physical health (b) assess L for depression, as many times this can occur when in a new situation (c) assess for history of substance abuse or medication abuse to rule out possible misuse of the medication (d) assess for other symptoms of anxiety or stress

A

Of those listed below, the most important reason for goals to be set in therapy is: a. it can help the worker estimate what type and extent of helping services will be needed b. it can help in the establishment of roles c. it provides the content for the initial interview d. it is a general guideline for future intervention

A

Prior to the termination of treatment, a client states that his insurance has run out. It is your opinion, as the social worker, that services need to be continued. You should: (a) arrange a sliding fee scale (b) terminate therapy (c) discuss other options for therapy (d) offer a referral to assist the client

A

Self-disclosure by the therapist is best defined in the therapeutic setting as: a. the worker shares personal information b. the client shares personal information c. the worker requests the client to share personal information d. the client requests worker validation

A

The concept of gender roles is BEST defined as: A attitudes and behaviors attached to individuals because of their sex B an individual's sexual orientation and/or preference C biological characteristics that influence an individual's behavior D interplay of biological and sociocultural influences

A

The mental health agency learns that a local landlord refuses to rent out apartments to African Americans. The Board decides to take action on behalf of several clients who have been refused housing. The best strategy is to A. use existing anti-discrimination statutes and institutions to force landlords to comply with the law. B. organize a demonstration to protest the landlord's actions. C. sue the landlord for damages. D. publicize the landlord's behavior.

A

The most important consideration in choosing a goal with a client is A. what the client wants. B. what the agency wants. C. what the worker wants. D. what the client's parents want.

A

When addressing mental health conditions, completing a comprehensive diagnostic assessment is most helpful in the are of: a. assessment interpretation b. treatment regimes c. medication recommendations d. treatment planning

A

When addressing termination of the client in a planned short-term treatment context, it should FIRST be discussed with the client. A. In the beginning of treatment B. In the middle of treatment C. at the end of treatment D. generally, not discussed until termination

A

When selecting indicators of organizational performance, the MOST critical factor a program evaluator should consider is the: A ability of workers and managers to have some control over the performance being evaluated B length of time data related to the indicator has been collected C interest of board members in the information to be collected D potential public relations damage that could come from a negative result on the indicator

A

When treating persons who have had long periods of alcohol dependence and abuse, the worker might reasonably expect that A. there is a high likelihood of cycling between periods of relapse and recovery. B. once treatment has been fully implemented clients will be free of their addiction. C. the client will consistently lie about their alcohol use. D. their friends will be supportive and try to help them refrain from using alcohol.

A

When using the concept of self in the therapeutic concept it is best defined as: a. the way in which a social worker purposefully interacts with a client b. revealing of personal information from the social worker to help the client c. transference between the client and the social worker d. the disclosure of the information by the client to the social worker

A

Which of the following medications is primarily used in the treatment of psychosis? (a) haldol (b) xanax (c) wellbutrin (d) prozac

A

Which of the following social work activities would be considered a PRIMARY prevention effort to address the problem of child abuse? A Helping design legislation which protects the rights of children B Providing shelters for children who have been abused C Conducting treatment groups for abusive parents D Delivering crisis intervention in-home services for abusive families

A

Which of the following tasks is appropriate for a social worker utilizing a strategic approach to family therapy? A Designing an intervention for each specific problem B Limiting interventions to the immediate family C Assuming a nondirective role with the family D Completing an in-depth family history

A

n therapy a client begins to complain about her current boyfriend stating that he is often insensitive to her needs. The social worker asks the client if it is the same boyfriend she was discussing in the previous session and described him as very kind and considerate. The client agrees that it is the same person. The social worker comments directly to the client that she is describing her boyfriend in a very different way from last week. The technique the social worker is using here is referred to as: a. confrontation b. clarification c. reality testing d. interpretation

A

You are working with a client in a hospital setting. They indicate that want to run down to the cafeteria and get themselves a cup of coffee and asks you if they can get you a cup of coffee as well. The BEST thing to do is to... A) Decline the coffee but thank the client for their thoughtfulness. B) Accept the offer because it is "just a cup of coffee" and it is a token gesture of respect. C) Decline the offer but offer to walk down to the cafeteria with the client and get your own cup of coffee. D) Accept their invitation but offer to pay for your coffee

A) Decline the coffee but thank the client for their thoughtfulness. This is a clear example of what ethics professionals call the "slippery slope". The chances of you accepting a cup of coffee as a token gesture will probably have no impact on your relationship with the client. But, you have just crossed a boundary, no matter how small, and the relationship has changed in ways that you cannot predict. You do not know what "buying a cup of coffee means to the client"! In some cultures, providing food and drink is very important and sets up very clear roles. What happens the next time the client meets you and wants a cup of coffee but only has enough money for themselves. They then decide to forgo the coffee rather than embarrass themselves by not being able to but you one. Alternatively, they say, "Hey, I want a cup of coffee and I am broke today, since I got last time, how about you getting them this time?" Simple things can get complicated very fast when you do not maintain clear boundaries. Remember, boundaries are nothing more than rules you create about how to interact with other people. Make your rules and do not violate them and you will be much happier as a therapist.

When systematic desensitization is used to treat a phobia, the feared object or event is considered to be A) the unconditioned stimulus. B) the unconditioned response C) the conditioned stimulus. D) the conditioned response

A) the unconditioned stimulus. Systematic Desensitization ranks an encounter with a phobia from lowest to highest in fear response. Example for the fear of snakes. -- Lowest exposure: seeing a picture of a snake. -- Medium exposure: being in the same room as a snake. -- Highest exposure: touching the snake. > The snake is the stimulus. > The fear/emotional reaction is the response. This understanding takes out B and D. Now, unconditioned vs conditioned. A is correct because... > The unconditioned stimulus is one that unconditionally, naturally, and automatically triggers a response. --> seeing the snake elicits fear D is incorrect because... The conditioned response is the learned response to the previously neutral stimulus.

A mother brings in her three-year-old son for an evaluation because she is concerned he is stuttering After completion of the assessment the worker correctly tells the mother:A. This can be a normal problems that can occur at this age.B. That her child has probably developed this in regard to increase emotional stressC. This conditions is generally more common in girls than boysD. An immediate referral for a physical exam is required.

A. This can be a normal problems that can occur at this age.--- Not considered an issue until age 5

When interviewing a client whose speech patterns are different from the social worker's, the social worker's PRIMARY consideration should be that: A communication patterns vary due to economic conditions B speech is influenced by culture and experience C psychosocial conditions will cause variations in speech D speech is influenced by group taboos and norms and may indicate false beliefs

B

A social worker is leaving an agency and must terminate many cases. How long prior to leaving should the social worker tell the clients she is leaving the agency? A. About one month before the termination date. B. About two weeks before the termination date. C. When the client seems ready to accept the social worker's departure. D. About three months before the termination date.

A. About one month before the termination date. It is generally accepted that one month is enough time for the worker and client to resolve the termination process.

During a group session, a client shares that 7 years ago he set a building on fire, which resulted in the death of a security guard. The social worker may do all of the following EXCEPT: A. Ask group members how they want to handle the situation B. Ignore the statement because the situation happened 7 years ago C. Check out members' feelings D. Encourage the client to speak with the police

A. Ask group members how they want to handle the situation

A SW at an outpatient clinic is working with 38 yr old female who reports having difficulty sleeping for the past 2 weeks. The woman reports that she has been getting a lot of ideas of what she can do in her life and that she cannot keep up with all of them. The woman reports that she has spent all of her money on materials to advance her life and is now at risk for losing her house. The client has never been hospitalized and denies ever being on meds. What is the most likely diagnosis. A. Bipolar I B. Bipolar II C. GAD D. Schizophrenia

A. Bipolar I

A boy (9-year-old) tells the social worker during a session that a relative has sexually abused him. The boy immediately breaks into tears as he fears the social worker will report it. The first response of the worker should be to: A. Calm the child and ask for further clarification of the abusive situation B. explain to the child that the social worker has no choice and it must be reported immediately C. tell the child that it is for the best in the long run and he must trust the worker D. explain the legal requirements and what will probably happen tot he child as a result of current abuse

A. Calm the child and ask for further clarification of the abusive situation --- the case will have to be reported if abuse report is verified

In the counseling environment, which of the following is the MOST important: (a) considering the cultural values and ethnicity of the client (b) cultural sameness of therapist and client (c) open discussion of differences between the client and therapist (d) cultural similarity between the therapist and client

A. Considering the cultural values and ethnicity of the client

After repeated hospitalization for severe back pain, Harry is provided with prescription painkillers. His family tells his social worker that he has started to obtain duplicate, sometimes forged prescriptions for controlled substance to supplement his regular supply. Harry told his family that the doctor is not giving him enough drugs to control his pain and that he has been forced to "improvise." The social worker might suspect A. Harry has become drug dependent and needs help to control his need for painkillers. B. the doctor needs to be informed and asked to increase Harry's prescription. C. the family should be helped to control Harry's behavior. D. Harry's behavior will probably diminish after that back problem is cured.

A. Harry has become drug dependent and needs help to control his need for painkillers.Drug dependence occasionally follows on legally prescribed use of prescription painkillers. Harry's request for increased doses, suggest a medically induced drug dependence for which treatment is needed. Even if the back problem is cured, it is likely the drug dependence will continue.

When counseling an Asian family that is forced to seek treatment due to court mandate, the social worker should first consider: A. Possible resistance to treatment based on a strong cultural value to resolve problems within the family B. A lower incidence of mental health problems among Asian Americans C. The open nature of Asian family systems and realtionships D. Preference of outside intervention to analyze family difficulties

A. Possible resistance to treatment based on a strong cultural value to resolve problems within the family

A young woman with a history of drug abuse appears at a Mental Health Clinic for her fourth visit with a clinical social worker. Though calm and lucid at previous interview, the client is not agitated, anxious, sweating profusely and her hands are shaking. She seems unable to sit still and moves her legs continuously. She occasionally rises and walks aimlessly around the room. The worker might suspect A. a possible drug withdrawal syndrome. B. a pre-psychotic episode. C. intoxication. D. a drug overdose.

A. a possible drug withdrawal syndrome. The client exhibits classic symptoms of drug withdrawal. Intoxication is likely to produce either an exhilarated or depressed state, while a drug overdose is likely to immobilize the person or even cause them to enter a comatose state.

To diminish a client's resistance in a treatment relationship the worker would A. acknowledge the client's ambivalence about accepting help in solving his problem. B. increase the client's anxiety about problems to increase his motivation. C. be firm with the client and insist an answer to all questions. D. remind the client of the reason for seeking assistance and stress the importance of cooperation.

A. acknowledge the client's ambivalence about accepting help in solving his problem. The way to reduce resistance is to accept the client's feelings, to "go with it" rather than hammer against it, which is likely to strengthen rather than weaken resistance. As a practical matter, premature direct approaches to resistance may also threaten the client sufficiently to terminate a treatment relationship.

When referring a recent legal Latino immigrant client for public assistance, the referring social worker should A. coach the client about what to expect, facilitate the referral by calling the welfare department, and ask the client to call back after filing an application. B. determine eligibility before making a referral and then advocate for the client. C. discourage the client from applying since welfare reform makes it difficult to receive assistance, then refer the client for employment. D. suggest that the client bring a legal assistance lawyer with him.

A. coach the client about what to expect, facilitate the referral by calling the welfare department, and ask the client to call back after filing an application.Helping the client obtain services is a similar process for immigrants and for others. If there were language difficulties, the social worker might also have to make additional arrangements.

A supervisor works with a social worker whose learning pattern is that of an experimental-empathetic learner. The worker is intuitive and uses self appraisals quite well. To enhance the worker's learning the supervisor might A. offer repetitive experience with different cases. B. state concrete and specific expectations. C. help the worker conceptualize and critically evaluate the issues. D. make the associations between theory and practice.

A. offer repetitive experience with different cases. The key term is experimental. This type of learner is prepared to learn empirically from demonstration-effect-observation. Learning best from observation and testing is quite common and suggests that a supervisor permits some latitude to the supervisee to allow them to test hypotheses.

In working with a client, you become aware that she persistently behaves in ways to please or gain the approval of others. While this is not always problematic, you discover that she is obsessed with wearing the "right" clothes, living in the "right" neighborhood, and marrying the "right" person. At present, her finances are in a shambles as she tries desperately to "keep up with the Joneses," and her romantic life is suffering, as she only pursues relations that she believes others think are optimum, rather than judging relationships on more personally relevant values, such as her feelings for them, baseline compatibility, etc. Utilizing Kohlberg's Theory of Moral Development, specify the Level and Stage of moral development that applies to this individual: a. Conventional Level, Stage 3. b. Pre-conventional Level, Stage 1. c. Post-conventional Level, Stage 6. d. Conventional Level, Stage 4.

A: Conventional Level, Stage 3. Pre-convetional is concerned with punishment Conventional Stage is concerned with "good girl, bad girl" aka being right. This stage is formed in sections- - stage three: people value a supportive community and therefore have the desire to be a good, helpful member. - stage four: meet the goals of the society, maintain law and order.

You are seeing a 16-year-old youth who has, for the past year, been losing his temper frequently, is regularly argumentative with adults, often refuses to follow direct requests, is easily annoyed, and routinely uses blaming to escape responsibility. Approximately four months ago he was caught in a single episode of shoplifting. The most appropriate diagnosis for this youth is: a. Oppositional defiant disorder. b. Conduct disorder. c. Impulse-control disorder. d. Disruptive behavior disorder, not otherwise

A: Oppositional Defiant Disorder. ODD: o Patterns of anger, irritability, argumentative or defiant behavior, and/or vindictiveness o Unlike children with CD, children with ODD are not aggressive toward people or animals, do not destroy property, and do not show a pattern of theft or deceit The hallmark of Conduct Disorder is deliberate cruelty, and wanton disregard for others rights and property. This client lacks any pervasive and long-standing evidence in this regard.

In the process of referral, clients will MOST likely follow through to the new agency if: A they have a clear understanding of what the new agency has to offer B the option to follow through is left up to the clients C they are informed of the consequences if they do not follow through with the referral D they know that the agency is expecting them

Answer: A This is an unscored question. One of the basic roles performed by social workers is that of brokering services, i.e., referring clients to appropriate community resources. However, client follow through to actually connecting with the referral agencies requires knowledge of the resources, which in turn requires agencies to be skilled in the interpretation of these resources to clients. If the social worker is unclear or vague about the services a referral agency offers option (key A), the client is much less likely to follow through. Communicating a clear picture of an agency's services is prerequisite knowledge to strategies—options (B), (C), and (D)—that might encourage the client to request the services

A social worker is asked to help a family struggling after the recent loss of their home. A fire consumed the entire structure and all contents within. Tangible supports such as food and clothing have been donated. What other care would be most beneficial at this time? A. Informational and emotional B. Medical and social C. Independence and money D. Psychological and medical

Answer: A - Informational support can be provided by advice from friends and family. A social worker can also make referrals to agencies that provide information on housing. Emotional support helps reassure individuals that others can help them through this traumatizing time.

Strategies used to cope with anxiety can be overused, causing them to take on the appearance of needs. Karen Horney developed a well-known theory that results from basic anxiety caused by interpersonal relationships: indifference, erratic behavior, direct or indirect domination, lack of respect, disparaging attitudes, lack of guidance, absence of admiration, lack of reliable warmth, having to take sides in an argument, too much or too little responsibility, injustice, discrimination, over-protection, isolation, unkept promises, and hostile environment. This theory is called what? A. Neurosis B. Needs C. Abandonment D. Cognition

Answer: A - Neurosis is the theory that results from basic anxiety and can be classified into three categories. The first refers to needs that move you towards others, causing you to seek affirmation and acceptance. The second category addresses needs that move you away from others creating hostility and anti-social behavior. Lastly, needs that move you against others cause individuals to control other people in a domineering manner.

A client has had difficulty falling asleep for the past year. During an initial session with a social worker in a medical clinic, the client expresses feelings of hopelessness and a lack of desire for sexual relations. These symptoms are MOST characteristic of: A sexual aversion disorder B depression C anxiety D adjustment disorder

B

Schizophrenia typically occurs in young adulthood and is accompanied by significant occupational or social dysfunction. The diagnosis is based on self-reported experiences and observed behaviors. There are no medical tests to validate the existence of this disorder. What are some reported symptoms? A. Hallucinations, disorganized speech, paranoia, social isolation B. Restlessness, talkative, sadness, crying C. Violent outbursts, anger episodes, night terrors D. Alter-ego, neurological problems, weight gain

Answer: A - Schizophrenia is thought to affect cognition, usually contributing to chronic behavior and emotional problems. Bizarre delusions add to auditory hallucinations and disorganized thinking. Social problems occur such as long-term unemployment, homelessness, and poverty. The onset of schizophrenia generally occurs before the age of 19.

A social worker visits a home after a referral comes in about inappropriate disciplinary techniques being utilized by a mother of a nine-year-old boy. The child is described as out of control, and the parent spanks him five or six times a day. The mother has been heard screaming at the boy, and this has not helped the situation. Shortly into the initial assessment, it is discovered the child has been referred for ADHD testing by a counselor at school. The mother feels her son is simply defiant and angry because his father left the family two years ago. Should the social worker be concerned? A. Yes, the mother has unrealistic expectations of her child's possible disability. B. No, the mother is handling the situation and understands her son's emotional problems. C. Yes, the child should be referred to juvenile services for aggressive behaviors. D. No, the neighbors are just trying to start trouble for the family.

Answer: A - There are issues of concern that should be further investigated by the social worker. The child is not receiving adequate care for his disability. A child with ADHD may demonstrate behaviors of defiance, aggressiveness and temper tantrums. The nine year-old may be displaying behaviors related to his disability and the mother has unrealistic expectations.

Crisis is a normal part of life, and individuals try to maintain equilibrium by using coping mechanisms. Sometimes the usual techniques do not work when a problem is threatening. People in crisis may experience guilt, feelings of anxiety, helplessness, and fear. Which of the following criteria are used to determine whether a situation constitutes a crisis? A. An individual's view and response to an event B. How others view and respond to an event C. Instincts on what is threatening D. Conditioning as to what defines a crisis

Answer: A - When an individual sees an event as threatening with no way to cope, he or she may find themselves in crisis. This is a state of psychological disequilibrium with a number of indicators. Changes will be observed in energy levels, ability to concentrate, sleeping patterns, and social interaction with friends or relatives.

Tina has suffered with obsessive-compulsive disorder for years. She has tried medications and seen several psychologists with no relief. The recommendation is for Tina to attend therapy that will promote positive reinforcement, modeling, and social skills training. The idea is to analyze the information gathered from the patient and make temporary changes in her activities. What kind of therapy should she attend? A. Cognitive therapy B Addictions therapy C Behavior therapy D Anxiety therapy

Answer: C - Behavioral therapy can be used to treat many psychological conditions such as depression, ADD, ADHD, obsessive-compulsive disorder, and specific addictions. Therapeutic treatment varies with each individual's case. Information is obtained from the patient to identify the behaviors that cause stress, reduce quality of life, and have a negative impact. Training will occur for desensitization, environmental modifications, and relaxation.

Strategies utilized to cope with basic anxiety caused by interpersonal relationships can often be overused. When this happens, they take the appearance of needs as described within the neurosis theory. Which psychologist developed this idea? A. Sigmund Freud B. Erik Erikson C. Karen Horney D. Piaget

Answer: C - Karen Horney defines basic anxiety as the need to move towards others, move away from others or move against others. The needs apply when an individual seeks acceptance, creates a hostile environment, or tries to control others in a domineering manner.

Over a third of people in various countries meet criteria for major categories of this disorder at some point in their life. It is defined as a psychological or behavioral pattern related to disability or distress occurring within an individual and is not a part of their normal development. Psychotherapy and psychiatric medication are two major treatment options. What is being described? A Psychosis B Psychotic disorder C Mental disorder D Schizophrenia

Answer: C - Mental disorders are thought to be disorders of the brain circuits more than likely caused by developmental processes. There is a complex play of experience and genetics that vary with biological and environmental contexts. In some cases, involuntary detention and treatment will be ordered according to legislation. Those who suffer with this disorder also have to live with the stigma and discrimination.

Cassandra works as a case manager for Child Protective Services. A new family she has been helping has a parent that is starting to raise red flags. The mother is the sole caretaker for the children but is demonstrating some peculiar behaviors. She has not kept consecutive doctor's appointments, has difficulty utilizing public transportation and fails to follow through with basic directions. This parent is always accompanied by another adult when she leaves the home on an errand. When Cassandra starts questioning the mother on her background, it is discovered that she attended special education classes and did not graduate from high school. These factors should arouse concern in what area? A. Child neglect B. Child abuse C. Intellectual limitations D. Financial distress

Answer: C - Parents with intellectual limitations often demonstrate less positive child-rearing practices. They fail to recognize their children's needs and lack the skills to advocate for them. A social worker should help the parents become aware of eligible community resources and make referrals as deemed appropriate.

A client presents with numerous symptoms of depression. Her spouse of 22 years died recently. She is not sleeping or eating well and cries often. An additional symptom which would lead to a diagnosis of major depressive disorder rather than uncomplicated bereavement would be: A withdrawal from close friends B a sense of hopelessness C obsessive thoughts about the deceased D talking about the deceased at every opportunity

B

A family in therapy reports that they are having extreme financial difficulties because the primary person responsible for paying the rent on their apartment has just lost his job. Initially, the social worker could help this family the MOST by providing information about: a. a referral for family counseling b. a referral for financial services c. a referral for vocational rehabilitation d. a referral for employment opportunities

B

Calvin has disowned that part of himself that controls anger. His sudden outbursts and bad temperament are a natural part of Calvin's neural network. He found love with Rebecca who, unlike him, displays no anger. She rarely gets upset and does not ever raise her voice. This theory of opposites attract is described as: A. Relationship conflict B. Separation anxiety C Polarizaiton D Compulsive behavior disorder

Answer: C - Polarization is the conscious want of something one cannot have or a trait that one does not possess, especially those that are directly opposite to one's attributes. Polarization is a sign of disowning parts of neural networks causing internal conflicts between the conscious and subconscious. An example may be an individual who cannot express love. They will seek out someone who can compensate them in this area. Polarization dynamics can formulate around many emotions and role functions at any point in a relationship. For example, the pursuer-avoider is a common polarization dynamic occurring early in relationships. At the same time, the giver-taker framework typically emerges later, in the course of a long-term relationship or marriage. Polarization is a key element of attraction and partner selection, i.e., we may be unconsciously drawn to individuals who we psychologically sense can house and act on specific emotions or relational experiences more so than are we. Individual or couple therapy may be useful in drawing one's attention to what is projected onto one's partner and, in turn, recognizing what relationship patterns are the product of polarization. Ideally, all polarizations would be reduced or eliminated so that each partner could occupy, experience, and act on complex emotions wholly and entirely rather than splitting off and projecting onto our partner the ones that are personally more difficult to tolerate or resolve.

In Maslow's hierarchy of needs, the need for love and esteem is referred to as emotional dependency. There are five levels of emotional dependency, all necessary for children to grow into happy individuals. These levels are self-actualization, esteem, love, belonging, and: A. External fulfillment B. Emotional reciprocity C. Physiological dependence D. Psychological growth

Answer: C - The five levels of emotional dependency are: self-actualization, esteem, love, belonging, and psychological growth. All five needs have to be met consistently, or an individual will suffer from a lack of fulfillment. Without love and esteem, children will not grow emotionally. The result may be psychological problems later in life.

All initial assessments should be conducted by an experienced and qualified social worker. Before confronting the family, there should be a plan about who is doing what and when information can be shared with the parents. Which of the following should be part of an initial assessment? A. Verifying Medicaid coverage B. Observing the child with both parents present C. Seeing and speaking to family members as appropriate D. Allowing the referent to sit in on all interviews

Answer: C - The social worker should speak to family members as appropriate in their ideal language. Children should be interviewed alone with the consent of the parent or caretaker. If consent is not given, other means should be utilized. The referent should be involved as far as obtaining relevant information.

The disengagement theory views aging as a mutual withdrawal from family, support circles and friends. Societies around the world appear to endorse the idea of elders voluntarily slowing down. Sometimes this occurs when older adults lose their ability to drive, eat or bathe on their own. Which of the following factor most influences the extent to which an elderly person will interact with others? A. Support services B. Abundant relatives C. Health and mobility D. Friends of the same age

Answer: C -As the elderly lose the ability to be mobile within society, their large network of social circles start to grow smaller. Eventually, the only people within an older individual's life tend to be a few close relatives. An assessment will see this withdrawal as beneficial to the community and elderly.

Lisa is a 14-year-old adolescent who skips school on a regular basis. She normally misses at least two days per week and reports just staying home in bed. The teen reports being tired all the time and cannot function at school most days. A social worker will identify a target behavior by examining the antecedent, identifying the function and identifying a positive replacement behavior. What type of problem solving should be initiated by the social worker? A. Authoritative B. Self-centered C. Collaborative D. Independent

Answer: C -The social worker should initiate collaborative problem solving. The social worker will determine what the ultimate outcome should be with therapy, and the client will work towards that goal. If Lisa is given time to think about her actions, she may decrease the incidents. By examining the problem situation and developing a hypothesis, behaviors can be sustained.

The diagnosis of Attention Deficit Hyperactivity Disorder has been quite controversial since the 1970's. Boys tend to be diagnosed two to four times more frequently than girls and symptoms occur prior to the age of seven. This neurobehavioral development disorder is the most diagnosed problem in children, affecting what percentage globally? a 70% to 85% b40% to 60% c 10% to 20% d 3% to 5%

Answer: D - 3% to 5% of children are diagnosed with ADHD throughout the world. 2% to 16% of these are school-aged children. It is characterized by the co-existence of hyperactivity and attention problems. ADHD management normally requires a combination of medication with behavior modifications, counseling, and lifestyle modifications.

A social worker is confirming that an assessment plan flows naturally from child and family to service planning. To assess needs and recognizing positive strengths, she conducts face-to-face meetings. Other agencies may be involved including teachers, therapists, and other community support systems. What type of assessment encompasses all of the above? A. Cognitive behavioral assessment B. Family fundamentals assessment C. Community relations assessment D. Functional assessment

Answer: D - A functional assessment is one of the key factors in achieving the well-being, safety and permanence of a child. The family and child are engaged as partners in the entire process. Family assessments are needs-based, team-based and strengths-based. They make use of family systems relationships.

A client tells a social worker at the intake interview that he has little trust for the agency where the social worker is employed. He reports that he has had several bad experiences there and is not sure that social worker will really be able to help. From the choices below, which one would prove to be MOST effective in engaging the client and beginning a productive therapeutic relationship. a. complete a social history, including former agency service b. focus on what the client presents as the problem to be addressed c. ask the client to explain the problems with his prior experiences in the agency d. focus only on providing concrete services

B

Jessica has a huge fear of flying, but a promotion at work requires excessive traveling in this manner. Excellent career advancement and matching pay persuades Jessica to get over her fear. She visits a therapist who directs her to sit and watch a reenactment of another individual resolving this problem. A movie is observed where a young man slowly makes the transition from airport to actually boarding the plane. He displays great dread, but takes his time and talks himself through the entire process. Once on the plane, more fears are addressed until the man has successfully made his first flight and landed safely. What kind of therapy was being utilized? a. Copycat therapy b. Disassociation therapy c. Avoidance therapy d. Modeling therapy

Answer: D - Bandura conducted research on the modeling theory and found it to be quite successful. His theory determined if you could get someone with a psychological disorder to observe someone dealing with the same problem in an effective manner, the first person will learn through modeling the second. Watching t movie is a form of Symbolic Modeling

Five-year-old Jesse has been displaying inappropriate behaviors on the playground. He wants to take other children's toys, push them off the swings when he wants a turn, and knocks kids down to get to the slide. Jesse clearly does not like to share or wait in line during recess. His teacher started conditioning him by providing the opportunity of being first in line for recess when he displays appropriate playing behaviors the previous day. Eventually, Jesse found that he could get to the swings and slide faster if he behaved. Conditioning occurs through interaction with the environment. What theory of learning is based on the idea that all behaviors are acquired through this conditioning? Anti-social theory Communication theory Comprehension theory Behaviorism theory

Answer: D - Behaviorism can be studied in a systematic and observable manner without the need for internal mental states. Two major types of conditioning are classical and operant. Classical utilizes a naturally occurring stimulus paired with a response, while operant is the method of learning through rewards and punishments.

A social worker meets with a family to gather information that will help identify patterns of parental behavior, both positive and negative, over time. The family's strengths are distinguished to determine protective factors that aid in identifying resources to help the household meet its needs. Overall requirements are addressed that affect the permanency and safety of the children. Information gathered through other assessments is also used to help develop a service plan or plan for intervention. What kind of evaluation is being described? A. Initial assessment B. Safety assessment C. Environmental assessment D. Comprehensive family assessment

Answer: D - The comprehensive family assessment gathers all background information on its members and evaluates the home environment. Contributing factors are discussed, such as domestic violence, lack of employment, alcohol/drug usage, and educational background. A social worker will also collect information on who lives in the home and the parent's culture, health and finances.

A referral comes into the local Children's Protective Services in regards to a female infant. The concerns raised by the informant include a lack of nutrition and improper supervision. Neighbors hear the baby cry at all hours of the night for long lengths of time. The parents leave the home before dark and do not seem to reappear until dawn the next day. Groceries brought into the home are noticeably snack foods and alcohol. The social worker responding will need to complete what kind of assessment? A. Comprehensive family assessment B. Functional assessment C. Economic assessment D. Initial assessment

Answer: D - The initial assessment is the first priority with a new referral that may indicate a child's needs are not being met. This is brief and determines whether there are concerns of significant harm, neglect, abuse, and whether services are required. If a child is found to be in need, further assessments will be completed.

A 40yearold patient with a long history of substance abuse has been diagnosed with AIDS in the advanced stage and is being considered for discharge from a treatment facility. The patient's elderly parents have expressed willingness to have the patient return to live with them. In formulating discharge plans, the social worker should FIRST evaluate which of the following factors? A Availability and access to outpatient health services B The family's ability to provide care C Patient's likelihood of involvement in substance abuse D Availability of financial and vocational resources in the community

B

A child welfare worker observes red marks and welts on a 4-year-old Asian child's stomach and thighs. The child has recently been ill. The family are recent emigres and they seem stable and loving. The child seems well-cared for. There are no previous referrals for child protection. The worker should first A. refer the family for a child protective investigation. B. explore the possibility of a culturally specific practice that might have lead to the symptoms described. C. recommend ongoing supportive services. D. Do nothing.

B

A client diagnosed as schizophrenic begins to hallucinate while in treatment. The social worker's primary strategy is to likely involve A. engaging the hallucination to diminish anxiety. B. continuing to provide ego support. C. referring the patient to a psychiatrist for medication. D. offering insight and trying to interpret the client's hallucinations.

B

A client frequently arrives late for appointments with her social worker and sometimes fails to call in advance to cancel. When confronted, she minimizes the problem and accuses the social worker of having unreasonable expectations. The social worker's MOST appropriate response is to: A ask the client to sign an attendance agreement and pay for missed appointments B encourage the client to talk about her feelings and perceptions C transfer the client to another therapist D offer the client a different appointment time

B

A family report that they cannot seem to communicate effectively with each other without each discussion ending with a disagreement. The parents state that they just want help in learning how to communicate better with their children and each other. The BEST initial intervention for the social worker to employ is: a. practice role-playing of effective and ineffective communication patterns b. explore with the family to better define what is meant by "better" communication c. complete a psychosocial history on each family member d. ask the parents about their knowledge of child developmental phases

B

A patient who was readmitted to a psychiatric hospital complains to the social worker that his previous and current doctors rarely understood him or responded to his feelings. In such a situation, the social worker should A. explore the client's reasons for this feeling and offer to discuss them immediately with the doctor. B. explore the client's reasons for this feeling and help the client discuss them with his doctor. C. acknowledge that this is a common problem to minimize his hostility. D. tell the doctor about the patient's complaint so that the doctor can handle this with the patient directly.

B

A seven-year-old boy was caught playing doctor with a friend. It is also noted that he has been masturbating at home since he was 5 years old and at times he does this in front of other children. What is the best intervention? a. tell the parents this is normal for his age and to ignore the behavior b. tell the parents this is normal for this age and help the parents understand how to best deal with it c. immediately assess the child for possible sexual abuse d. tell the child this behavior is inappropriate and will not be tolerated.

B

A social worker at a suicide hotline receives a call at 2:00 am from someone who says he is depressed and is contemplating suicide. The social worker should A. keep talking to him on the telephone until a specific arrangement can be made with a therapist later in the morning. B. send a crisis worker immediately or convince him to go to the hospital emergency room. C. tell him no none is available and to call back in a few hours. D. tell him to go to a mental health agency in the morning.

B

A social worker has been asked to assist an elderly client in making alternative living arrangements. In the initial interview, the client repeatedly attempts to discuss past experiences. What is the social worker's MOST appropriate response to the client? (A) ignore the references to the past (B) facilitate discussion of the recollections (C) refer the client for psychiatric evaluation (D) administer a geriatric evaluation scale

B

A social worker instructs the parent to ignore the social implication of the child's behavior that has just wet his bed and focus only on the physical consequences of the behavior. The parent is told only to deal with the consequence the resultant bed wetting. What form of therapy is this most likely being used? a. psychoanalysis b. behavioral therapy c. rational emotive therapy d. task-centered therapy

B

A social worker is seeing a client that is very anxious about some recent events that are happening. The social worker listens to the clients and uses silence in the session to facilitate the intervention process. Silence is used to: a. help the client realize the social worker is empathetic to the situation b. help in relieving the client's tension in regard to the situation c. help to facilitate rapport with the client d. assure the client that the social worker is listening to what is said

B

A social worker that works daily with clients in crisis reports her supervisor that she is having a difficult time dealing with her clients. She feels that all of her clients' have problems that are too severe. She believes that most of her current clients are simply not responsive to her methods of intervention. She states that she often feels bored and distanced from her clients' situations and often gets frustrated or angry with them. This supervisee is most likely dealing with the issue of: a. job related stress b. counter-transference c. transference d. a depressive disorder

B

A social worker wants to show hospital administration that the groups she is running are effective. She selects one group that is designed to address depression and administers a standardized scale to measure each participants level of depression before and after participating in the group. This design is best termed a: A. true experimental design b. one group pretest posttest c. pretest/posttest control group design d. random group design

B

According to the DSM criteria, which of the following symptoms is most prevalent in persons with major depressive episode? a. inflated self-esteem b. sleep disturbance c. hallucinations d. negative view of the future

B

An anorexic teenage girl has made great progress in an outpatient treatment facility. Her social worker is informed of the pending discharge home. The social worker is concerned because both of the client's parents exercise and diet rigorously. The best course of action for the social worker: a. release the girl to a halfway house. b. conduct a family conference to discuss discharge strategy c. encourage the client to stay with relatives d. assist the client to confront her parent's behavior

B

At the end of the initial session with a client diagnosed with Borderline Personality Disorder, the client hugs the social worker. The BEST response to this action by the social worker should be: (a) tell the client immediately this is not appropriate (b) at the next session discuss the behavior and establish boundary rules to be followed in subsequent sessions (c) realize that this is an indicator that the client accepts the therapist (d) to avoid rejection, reciprocate by hugging the client back

B

During the fifth session of marital counseling a couple continues to display fighting behavior. They do not appear to be responding to the techniques being employed. Suddenly, in the middle of an argument the social worker stops them and says "No one can help the two of you, you might as well just agree to fight as often as you are together". This technique is: a. reframing the situation b. a paradoxical approach or directive c. prescribing a ritual d. using reinforcement for behavior change

B

If a supervisor believes a supervisee's case is being handled poorly, the supervisor should FIRST: (a) discuss it immediately with the client (b) discuss it immediately with the supervisee (c) report it to the ethics committee (d) set up an appointment with the client involved

B

In an initial session with a social worker, a client mentions having been hospitalized several times for depression. When the social worker attempts to explore the hospitalizations, the client becomes tense and guarded, saying "it's old history". The client refuses to give permission for the records to be released. The social worker should FIRST: (A) explore why this topic appears to be upsetting the client (B) acknowledge the client's right to decide about release of records (C) reassure the client that the focus will be on present issues (D) assess the client's current level of depression

B

In counseling battered women who have been involved in lengthy marriages, the social worker should keep in mind that many battered women A. do not care for the batterer and find it easy to decide to leave. B. believe that they can stop the violence by learning new techniques to control the behavior of their mate. C. trust the confidentiality and protection of the agency. D. do not experience ambivalence toward the batterer or the relationship.

B

Latin Americans tend to accept help from experts who A. sound knowledgeable and work for local institutions. B. speak Spanish, are knowledgeable about the area of concern and have a positive reputation within the community. C. are Anglo and seem confident about the problem and its possible solutions. D. have a reputation for knowledge about a subject and are also a relative or friend of the person in need.

B

M enters a crisis center telling the social worker that she wants to kill herself because she is flunking several college courses. During the initial interview, M appears lucid, has a supportive family, no history of previous attempts, and states that she does not have a plan to commit suicide. The social worker still has concern for the young woman's safety, so the worker should FIRST: (a) hospitalize her immediately (b) try to establish a "no suicide" contract (c) notify the proper authorities (d) start counseling process on how to address this problem

B

Mr. and Mrs. Beech continuously argue and bicker, though they agree in one area: Mr. Beech's beginning career as a college teacher. Both are pleased with his choice of the profession, the salary and the university that hired Mr. Beech. Mr. Beech is on the verge of an emotional collapse, however, due to his fears of starting a career and the stress involved in completing his dissertation and his work as an instructor. Mrs. Meech demands constant attention and interferes with his work. The social worker's primary focus in treatment would be to A. modify the attitudes that created the marital crisis. B. give support to Mr. Beech and help him work through the stresses involved in pursuing his academic career. C. examine and interpret the underlying causes of their marital conflict. D. help Mrs. Beech minimize her destructive attacks on her husband.

B

Mrs. K., age 35, has been in treatment for the past three months. She says she feels inferior and worthless when with family and friends. She avoids close personal relationships and fears dependence on others. She feels that most people are more in control and are "happy about their lives." Preoccupied and anxious about her failures and worries about the future, she constantly complains about chest and back pains that keep her awake at night. Medical evaluations are negative. She also experiences sudden mood shifts in which she appreciates her achievements, feels good and sees others as the problem. Mrs. K's condition could be diagnosed as A. overanxious reaction to adulthood. B. borderline personality disorder. C. narcissistic personality disorder. D. reactive depression.

B

Mrs. Taylor was referred to a social worker after her alcoholic husband was hospitalized following an acute episode. She felt there was no reason to see a social worker as she had consulted with several doctors and read popular books on alcoholism. She feels that her husband sees her as a mother symbol and blames her for all his problems. She offered him a psychology book to read that she thought would help him understand his behavior. Mrs. Taylor found passages from an abnormal psychology book that she thought described her husband. She read sections of the book to Mr. Taylor, especially chapters describing personality disorders, egocentricity and dependency. Mrs. Taylor's major defense mechanisms are A. reaction-formation. B. denial and intellectualization. C. obsessions and compulsions. D. compensation and displacement.

B

Rose, a crack cocaine and alcohol user, appears at the HMO clinic complaining of frequent stomach pains. Rather than getting the painkillers she wants, she is surprised to learn that she is at least nine weeks pregnant. In addressing Rose's pregnancy, the social worker interviewing her should first seek to A. refer Rose for psychiatric care. B. help Rose understand the health consequences of drugs and alcohol on her body and the developing fetus. C. discuss the pregnancy with Rose's family and ask for their help in forcing her to stop using drugs and alcohol, at least during her pregnancy. D. arrange admission to a half-way house.

B

The PRIMARY source of information for a social history on a client diagnosed with middle stage Alzheimer's disease is the: A individual client B client's family C referral source D attending physician

B

The best way to establish support for the funding of a community development program is to: a. gain support from community leaders b. help to establish grass-roots supports c. highlight a similar program that has been viewed as sucessful d. create a well formulated plan for intervention

B

The biological parents' rights have been terminated and the child is placed in foster care. Some months later, the parents contact the foster care social worker, requesting that their child be returned to their custody. The social worker should: A set up an agreement for changes needed before agreeing to reconsider reunification B advise the parents to talk to a lawyer about their request C inform the parents that there is nothing the social worker can do D request that the parents agree to a home study as a preliminary to reunification

B

The phase of the helping process which leads to subsequent implementation of interventions is: A exploration and summation B exploration, assessment, and planning C initiation of establishing rapport D clarification and analysis

B

The social worker takes the LEAST active role in treatment in which of the following perspectives: (a) Ego-psychology (b) Client-centered (c) Behavioral (d) Task-centered

B

The type of information that can be prosecuted under tort law is: a. confidential information b. privileged information c. essential information d. personal information

B

Using the technique of "modifying the environment" helps to diminish A. superego anxiety. B. objective anxiety. C. instinct anxiety. D. anticipatory anxiety.

B

When treating clients with a borderline condition it is generally recommended that the social worker A. urge the client to accept group treatment, preferably the Gestalt approach. B. allow the client to fell in control of the session. C. focus on the issue of dependency and the client's inability to choose life goals. D. confront the client on his or her use of diminishing defenses.

B

Which of the following statements about Tardive's Dyskinesia is untrue? A. It is caused by prolonged use of antipsychotic medication. B. It can always be reversed by discontinuing psychotropic medication. C. It causes involuntary facial, lip and tongue movements. D. It is a late-appearing condition causing gross motor problems involving jerky limb movements and finger motions.

B

You are working with Jodi and during your second session, she states that she is a left- handed individual and therefore is more sensitive, more creative and more emotional than many other people who are right handed. You ask her how she knows this and she says, "I just heard it and it fits me." You are aware that she may be correct according to a theory created by Nobel-prize-winners Roger Sperry and Robert Ornstein. This theory is known as... A) Brain Schism Theory B) Brain Lateralization Theory C) Inherent Lobe Theory D) Corpus Collosum Effect

B) Brain Lateralization Theory

You are working in a clinic and during a staff meeting a social worker discusses his client's behavior in terms of the "games" they are playing and the "scripts" they are enacting. You know this social worker has been trained in... A) Reality therapy. B) Transactional Analysis. C) Rational-Emotive therapy. D) Gestalt therapy.

B) Transactional Analysis. Transactional Analysis (TA) is a psychoanalytic theory and method of therapy wherein social interactions (or "transactions") are analyzed to determine the ego state of the communicator (whether parent-like, childlike, or adult-like) as a basis for understanding behavior.

Albert Bandura proposed Social Learning Theory. One of its main tenets is BEST described as... A) the acquisition of a new behavior requires cognitive insight. B) behavior is learned through a process called modeling. C) reinforcement is not required to learn new behaviors D) new behaviors are acquired through stimulus-response connections

B) behavior is learned through a process called modeling. Bobo doll experiment Albert Bandura believed that aggression is learned through a process called behavior modeling. He believed that individuals do not actually inherit violent tendencies, but they modeled them after three principles. Bandura argued that individuals, especially children learn aggressive responses from observing others, either personally or through the media and environment.

You are working with a counselor who is beginning a short-term group and they state they are not interested in their clients developing any insight into their actions. Which of the following types of therapy would BEST describe the overall concept of the group? A) psychoanalytical. B) behavioral C) Gestalt approaches. D) Adlerian

B) behavioral Behavior therapists make frequent use of insight, but avoid the term because dynamic therapists have formulated it in terms of the unconscious. Insight does not necessarily imply belief in the existence of the "unconscious mind." Behavioral insight consists of making the client aware of the antecedents and consequences of target behavior. Case studies are presented in which behavioral insight was involved in therapeutic change. Implications of behavioral insight for behavior therapy are discussed.

You are working with a clinical social worker who has been trained in structural family therapy. The new case you are working on is an enmeshed family of four, in which the mother is over-controlling, yells, nags and confronts her 14 and 16-year-old daughters. The FIRST thing a structural family therapist would do is... A) provide a paradoxical intervention by telling the mother to set aside an hour a day to nag her daughters. B) create a structural diagram of the family to determine interventions. C) help the mother develop insight into how her yelling elicits oppositional behaviors from her children. D) create a role play where the daughters treat the mother in the same fashion they are treated.

B) create a structural diagram of the family to determine interventions. Structural Family Therapy is concerned mainly with the WHOLE family structure/system. Notes Structural family therapy is aimed at establishing or reinstating appropriate boundaries and facilitating effective communication. The therapist seeks to change the structure of the family. Strategic family therapy is aimed at encouraging family members to re-conceptualize their problem and solve it as a unit. Diagrams that may be used include: - genograms are graphic portrayal of the composition and structure of one's family - an ecomap is a graphic portrayal of personal and family social relationships.

You are reviewing a chart of a patient who is being referred to you from a colleague. You notice in the chart a previous clinician has written, "Client has periods of time where they report depersonalization." You understand the BEST way to understand the client is to understand that 'Depersonalization' is characterized by A) severe free-floating anxiety. B) disorientation combined with a loss of ego functions. C) extreme avoidance and irrational fear. D) internalized voices which tell the person they are 'bad'.

B) disorientation combined with a loss of ego functions. Depersonalization as an isolated event occurs in many people without significantly affecting their functioning; it is considered a disorder only when it impairs the patient's daily activities, when it is not associated with some other mental disorder, and when the patient's perception of reality remains intact.

You have begun working with a 67 year-old male who has recently retired from a job at a manufacturing plant where he had worked for the last 26 years. His initial complaints are depression, isolation and a general feeling of uselessness. You ask about his interactions with friends and he says he has no-one but his wife of 35 years. He jokes that she is getting tired of him hanging around the house. Your BEST intervention would be to... A) Work with him to schedule a second honeymoon with his wife B) explore different possible activities which could help bring usefulness back into his life. C) locate some vocational training so he can go back to work and feel productive again. D) help him locate a support group in order to further his adjustment in retirement.

B) explore different possible activities which could help bring usefulness back into his life. This client is undergoing a very understandable grief process as the loss of his way of life. Our working situation structures our life. He has probably driven the same route, 5 days a week, for the past 27 years. Eaten at the same restaurants and talked with the same people. Now he has none of that. He feels anchorless and floating-free. You need to help him explore activities and ideas which can allow him to become part of something bigger than himself. Most people want to be part of something which means more than they do as an individual.

From the work of Freud, a group of psychoanalyst arose who would become known as the Neo-Freudians. When you are reviewing the differences between the Neo-Freudians thoughts and the Classical Psychoanalysis of Freud, the PRIMARY difference is that neo-freudians... A) were unconcerned with life experiences B) placed greater emphasis on the impact of social influences on ego functions C) focused on an individual's "innate wisdom" D) placed greater emphasis on instinctual and unconscious forces.

B) placed greater emphasis on the impact of social influences on ego functions Freud however placed greater emphasis on instinctual and unconscious forces.

You have a social work intern in your office and she has been reading about different therapies. She is slightly confused about Client-Centered Therapy and the work of Carl Rogers. She is unsure how a client centered therapist would approach the concept of client diagnosis. You know the BEST answer is... A) a task shared equally by the therapist and client. B) probably unnecessary and possibly detrimental. C) the initial step in therapy. D) a process ongoing throughout therapy.

B) probably unnecessary and possibly detrimental. Client-centered therapists rarely ask questions, make diagnoses, provide interpretations or advice, offer reassurance or blame, agree or disagree with clients, or point out contradictions. Instead, they let clients tell their own stories, using the therapeutic relationship in their own way.

You have taken a job as a prison social worker and one of your first clients of the day is a man serving a life sentence without the possibility of parole. He has served 8 years up to the point where you meet him. When you ask him why he requested an appointment with mental health he begins a long tirade about how everyone is against him. He tells you the dorm sergeant hates him and his fellow dorm members are always causing him problems. You have read his "prison jacket" and realize he has been sentenced for 1st Degree Murder. When you ask about the crime he was sentenced on he tells you it was not his fault. Upon further questioning, he decides to tell you his story. He went out one night to rob a local drug dealer because he needed cash. He took his brother's pistol. As he was robbing the dealer, the dealer pulled out a gun and shot him, wounded, but not incapacitated he fired back and killed the drug dealer. He was convinced that since the drug dealer shot him first, he was only acting in self-defense. He then stated that his lawyer "screwed him" because his lawyer refused to use self-defense as a legal defense for his actions. You realize the primary issue facing this inmate/client is his internalization of a specific criminal thinking pattern. You recognize this criminal thinking pattern to be... A) The "good person" stance B) the "victim" stance C) the "lack of time" stance D) the "unique person" stance

B) the "victim" stance Using the victim stance allows you to blame other people for what has happened to you. Your primary behavioral mechanism is to "point fingers at others" and "generate excuses" for your lack of success. The pay-off for this type of behavior is the ability to NOT ACCEPT responsibility for your life. There is no need to put in the hard work of actually determining why you are "where you are".

A client diagnosed with a character disorder begins to realize certain behaviors are no longer acceptable. The client then becomes depressed, experiences loss of appetite and has trouble sleeping. The social worker would see these symptoms as A. an exacerbation of the client's problem. B. the result of the worker's misinterpretation of the client's psychodynamic functioning. C. an indication of progress. D. the client becoming aware that the worker is not interested in offering treatment.

C

When referring a client in the community for service, the most important information for the social worker to discuss with the client is: A. the agency's reputation in the human service community B. Appropriateness of the agency service to meet the client's needs. C. Fees the agency will charge the client for services. D. Ease with which the client can reach the agency from home or work

B. Appropriateness of the agency service to meet the client's needs.-----All may be relevant, but appropriateness of the agency to address the problem is essential.

Some individuals suffer from a disorder that is characterized by extreme mood swings. Manic behaviors that may be damaging to a person's health will suddenly change to a manner of depression. This is a complex disease to diagnose since moods can last for weeks or months. What is the disorder being described? A. Post-Traumatic Stress Disorder B. Bi-Polar Disorder C. Dissociative Identity Disorder D. ADHD

B. Bi-Polar Disorder

A social worker meets with a 30 year old client who came to therapy after receiving an ultimatum from her spouse. The client reports experiencing periods of feeling 'on top of the world' followed by periods of severe depression. When the social worker inquires about what led to her spouse's ultimatum that she attend therapy, the client shares that last week she quit her job and maxed out her credit cards to start a new business venture that came to her in the middle of the night. The client shares that she hasn't slept in days, but feels like she can take on the world. What is the MOST likely diagnosis? A. Cyclothymic Disorder B. Bipolar 1 C. Bipolar 2 D. ADHD

B. Bipolar 1 The correct answer is B: Bipolar 1. The symptoms being described indicate the client is experiencing a manic episode. The symptom severity is beyond that of a hypomanic episode, which allows us to rule out Cyclothymic Disorder (A) and Bipolar 2 (C). Even though the client shares that she experiences periods of severe depression, remember that as soon as a manic episode is present, the diagnosis is Bipolar 1 (even when there are also depressive symptoms). The symptoms being described go beyond that of ADHD (D).

A belief that one's thoughts alone can result in the accomplishment of certain wishes is: a. Depersonalization b. Magical thinking c. Thought broadcasting d. Looseness of associations

Rationale: Basic recall item requiring knowledge of cognitions.

A social worker meets with a client who discloses they are HIV+ and are having unprotected sex with their long-term partner who does not know their status. The social worker and client discuss the importance of openness with the partner, but the client refuses, expressing concern that the partner will end their relationship. The social worker is concerned about the risk to the client's partner. What should the social worker do? A. Express their concerns to the client. B. Honor the client's right to self-determination while further exploring their concerns. C. Inform the partner without disclosing the identity of the client. D. Consult with a colleague.

B. Honor the client's right to self-determination while further exploring their concerns. The correct answer is B: Honor the client's right to self-determination while further exploring their concerns. It isn't our place to express our concerns to the client, as this is placing our values on them. C is breaking the client's confidentiality (not to mention that the long-term partner would likely know the identity of the client even if the social worker did not explicitly say it). And for purposes of the test, there is no reason to do D at this point. If the question stem said something like 'the social worker is unsure of how to proceed' or 'the social worker doesn't know if they have a duty to warn the partner' then consultation would be a good next step. But as it is written, there is no need to consult at this time.

A social worker is meeting with a 23 year old client. The client was referred by his doctor who suspects he may have Bipolar Disorder. The client shares a long history of vacillating between periods of severe depression and mania. After completing an assessment, what is the FIRST intervention the social worker should recommend? A. Refer to an MD for a physical evaluation B. Refer to a psychiatrist for medication management C. Refer to a support group for young adults with Bipolar Disorder D. Begin psychotherapy

B. Refer to a psychiatrist for medication management The most effective treatment for bipolar disorder is a combination of medication and psychotherapy, and clients typically need mood-stabilizing medication to control manic (or hypomanic episodes). While we typically do not start off therapy with medication referrals, this will be necessary for some diagnoses (such as Bipolar Disorder and Schizophrenia). The client was referred by their doctor, so a referral to an MD for a physical evaluation (A) isn't necessary. While we may do C, this wouldn't come before referring for medication management. We will certainly begin psychotherapy with this client, but should first discuss the need for medication management (psychotherapy will likely be more successful once the client begins on a mood stabilizer).

A child welfare worker has been assessing custody of a child in a divorce case. The guardian ad litem has also been making this assessment. MOST LIKELY, the guardian ad litem's assessment will be: A. Different B. Similar C. Biased D. Formulative

B. Similar

During the first session with a client who was referred after being arrested for armed robbery, the client stated that he was suffering from depression. The social worker did not assess any symptoms of clinical depression. The best preliminary diagnosis for this client is: a. conduct disorder b. antisocial personality disorder c. malingering d. conversion disorder

C

The parents of a four year old child are referred for social work intervention by a local physician. The parents cannot get the child to separate form them and attend nursery school. The parent's complain that they cannot seem to get the child to willingly leave their presence without the child throwing a tantrum. The most important are for the social worker to address during the assessment phase is: A. The parent's use of rewards and punishments. B. The parent's knowledge of the child's developmental processes. C. Ways in which the child is disturbing the parent's relationship D. The parents own experiences as a child

B. The parent's knowledge of the child's developmental processes.-----first determine what the parent believes to be normal responses for the age of the child.

Hassan, his wife and two daughters aged 11 and 13, moved to the United States from the Sudan seven years ago. Both parents sought and achieved citizenship. The year, prior to a planned brief visit to Khartoum in the Sudan, both daughters ran away from home several times and were returned by the police. When asked by a social worker whether their parents mistreated them, both girls said their parents loved them and took good care of them. After careful inquiry, the social worker learned that the girls did not want to return to the Sudan because the purpose of the visit was to perform clitorectiomies on both girls. What should the social worker do with this information? A. The social worker can and should do nothing. Female circumcision is an accepted cultural practice in North Africa and among many Islamic cultures, and the worker has no warrant to impose her values on the family. B. There is no question that this situation fits the criteria of child abuse. The worker should take immediate steps to involve the child protective agency and to prevent the family from performing the surgery. C. The social worker should suggest that the family enter family counseling to try and reach some accommodation that all could agree with. D. The social worker should suggest that the family have the procedure done in the US to reduce the girl's fear and to insure a better medical result

B. There is no question that this situation fits the criteria of child abuse. The worker should take immediate steps to involve the child protective agency and to prevent the family from performing the surgery.Clitorectomies are primitive cruel cultural practices that many human rights advocates believe should be banned. The social worker should treat this situation as a child abuse issue and involve the protective agency to protect the children. Immediate action is needed since the family plans to travel to the Sudan.

A social worker seeks to develop a relationship and case plan with a 15-year-old Black adolescent who is involved with a violent gang. His family has low expectations of him and focus on "staying out of trouble" as a primary goal. Some relatives have rejected him, telling him he will grow up like his father --- a chronic alcoholic and wanderer. Neighborhood adults have accorded him pariah status and are openly fearful, distrustful and occasionally hostile. He feels that police harass Black teenagers and do not provide protection. His school performance is poor and because of his attitude, teachers have low performance expectations for him. He does not participate in social agency recreational programs and view youth workers as simply another type of authority figure. The gang provides a sense of community and family and offers opportunities to display power and competence. In working with the teenager described above, the social worker should consider a plan that uses A. a family therapy approach. B. an ecological systems approach. C. a psychoanalytic approach. D. a problem-solving approach.

B. an ecological systems approach.An ecological systems approach is one which takes into account the social environment. A central feature of this adolescents life is his membership in the game. Thus, the most effective plan is one which recognizes and respects the centrality of this relationship.

The Jones family is composed of a mother, her live-in boyfriend, and seven children. The family receives financial assistance through Temporary Assistance for Needy Families (TANF). Mrs. Jones is depressed and complains of losing control of the children. There is continuous fighting among them. According to Minuchin's model of family treatment, the most helpful intervention would be A. sibling play therapy with each child to encourage them to verbalize their feelings. B. first working with the family's neutral subgroupings, then changing the subgroupings and actually manipulating them in relation to the whole family group. C. working with Mrs. Jones and her boyfriend. D. working with the adolescent subgrouping giving Mrs. Jones the most problems.

B. first working with the family's neutral subgroupings, then changing the subgroupings and actually manipulating them in relation to the whole family group.Sal Minuchin, a structural family therapist, believed that child therapy could not be practiced outside of the context of the family. Thus, an approach using the Minuchin model would require that at least initially, the practitioner will intervene with the whole family. None of the other choices includes the entire family as the focus on intervention.

An infant showing drug withdrawal symptoms at birth is read for foster care placement following medical treatment and court action. To prepare the prospective foster parents, the social worker should A. not mention the infant's drug background so as not to precipitate feelings of bias. B. inform them of the infant's drug background, treatments provided and answer any questions about the medical issues the child may present. C. inform them that the baby may have some medical problems and give them instructions about how to proceed. D. inform them of biological mother's drug addiction and that they should not worry about any potential medical problems.

B. inform them of the infant's drug background, treatments provided and answer any questions about the medical issues the child may present.This is both an ethical and a practice question. The foster parents have a right to full disclosure of the infant's condition. Moreover, in order for them to properly care for the child they need to have complete information. Holding back or minimizing is an ethical breach and is poor practice.

The Family Court refers Mr. and Mrs. Williams to a community service agency. During the initial interviews they are angry, have poor reality testing, and have difficulty restraining themselves from acting out. Their projections are easily accessible, but Mr. Williams exhibits disorganized thinking. the most appropriate recommendation of the social worker would be to A. see the couple conjointly, but on a short-term basis. B. refer them to a psychiatric clinic for possible medication and long-term therapy. C. see the couple individually for short-term treatment. D. provide no psychotherapeutic interventions since it might exacerbate the decompenstation.]

B. refer them to a psychiatric clinic for possible medication and long-term therapy. The combination of anger, poor reality testing, and acting out behavior suggests a more intensive therapeutic approach in mental health or psychiatric setting. The "service" agency would not seem to be a proper treatment venue for couples with serious emotional difficulties. Short-term family treatment and short-term individual treatment would not seem appropriate. Doing nothing is not an appropriate option.

After working with a 15-year-old girl with a substance abuse problem for several weeks, the child welfare social worker learns that the client is pregnant. She is unable to control her drug use, has left a rehabilitation program and does not wish to return and seems unconcerned with the potential serious health effects on her unborn child. The social worker's best strategy is A. suggest that the client reenter the drug rehabilitation program. B. seek to place her in a residential setting for the duration of her pregnancy, even if a court order is required. C. notify the client that she can be compelled to stop her drug abuse. D. discuss the problem with her obstetrician.

B. seek to place her in a residential setting for the duration of her pregnancy, even if a court order is required.The client's lack of concern, immaturity, uncontrolled drug use and failure to adequately care for herself during pregnancy require a dramatic solution. The worker should find a setting where her drug use can be controlled, at least while she is pregnant. A residential facility is likely to be the best setting, even if it requires a court order or a child abuse petition.

In human growth and development, psychosocial theory is similar to psychoanalytic theory, however, in psychosocial theory the emphasis is placed on: A. Language development B. Social development C. Intellectual development D. Cognitive learning

B. social development

Jane, a 14-year-old girl diagnosed with diabetes, reports to her clinical social worker that she and her boyfriend experiment with drugs. Jane is unsure about the names of the drugs she has taken at parties, but she does state that she is concerned about the possible side effects. The social worker's best option is to A. minimize Jane's concerns, but suggest that she not try any more drugs. B. support Jane's apprehension, suggest a referral for drug education and a discussion with her physician. C. discuss Jane's drug use with her parents and ask them to discuss the dangers with her. D. suggest that Jane get more information about the drugs she has taken.

B. support Jane's apprehension, suggest a referral for drug education and a discussion with her physician.Drug use by someone with a diabetic condition can be extremely dangerous. Jane should be helped to resist taking any unknown drugs and she should receive a full explanation of the interactive effects of narcotics and diabetes. Her physician should also be involved to reinforce the discussion.

An infant is assessed as low-functioning. The family's social worker should help the family understand that A. the major causes of mental retardation are found in the parent's genetic characteristics, not in their behavior. B. they can be helped to meet the child's needs and that community services are available to them. C. the child should probably be paced in a specialized facility. D. the parent's should consider specialized foster care.

B. they can be helped to meet the child's needs and that community services are available to them.B is the only concept that starts from a "strengths" perspective, emphasizing the possible and assuring that the family will be able to manage this crisis and make a reasonable adjustment. The other alternatives are negative and warranted by the description of the child's condition.

In the initial phase of treatment, the primary social work task is to A. establish a warm relationship with the client. B. understand by the client is seeking help. C. engage the client in treatment. D. gather information for a psychosocial diagnosis.

B. understand by the client is seeking help.The first task is always to understand why the client has sought assistance. During that process a warm relationship (A) can be established, the client can be helped to engage in treatment (C) and information for a psychosocial diagnosis (D) may be obtained.

Identify the missing step in Albert R. Roberts seven-stage crisis intervention model: 1) assess lethality; 2) establish rapport; 3) __________; 4) deal with feelings; 5) explore alternatives; 6) develop an action plan; 7) follow-up. The third step is: a. Evaluate resources. b. Identify problems. c. Environmental control. d. Collateral contacts.

B: "Identify problems" is the third step in the Roberts crisis intervention model.

You are called to see a young black man in his mid-twenties. Two adult sisters brought him for an urgent appointment. The young man is clean, neatly dressed in slacks, dress shoes, and a tweed sport coat. He is also calm, relaxed, and without any signs of agitation. The two sisters, however, appear disheveled, frazzled, and almost histrionic. They blurt out the he "has problems" and urge you to talk with him. Privately, he tells you that he is fine. Later, however, the ladies tell you he left home abruptly and traveled cross-country with no destination. He didn't sleep for three days (with them pursuing him), was spending money excessively and writing checks he couldn't cover. He ended up in a nationally famous amusement park at 3:00 a.m. (having scaled a fence), sitting on an empty roller coaster "waiting for the ride to start." When confronted, he admits all of this, but says he's now rested, and doing better. The most likely diagnosis would be: a. Brief psychotic disorder. b. Bipolar I, single manic episode, in full remission. c. Bipolar I, single hypomanic episode, in full remission. d. Cyclothymic disorder.

B: Bipolar I, single manic episode, in full remission. There is no evidence of frank psychosis, thus brief psychotic disorder can be ruled out. Hypomania does not appear appropriate, as the client's behavior would likely have resulted in hospitalization had anyone been able to evaluate him during his period of mania. Cyclothymic disorder does not appear appropriate, as the client's conduct exceeded the threshold severity for hypomania, and no information is provided regarding depressive symptoms (though he may well have them). Finally, the Bipolar I, single manic episode is identified to be in full remission, as the client's manic symptoms appear to have completely resolved.

A therapeutic approach that views the client from a social context, that sees behavior as derived from unconscious drives and motivations, that views disorders and dysfunction as emerging from internal conflicts and anxiety, and that seeks to facilitate the conscious awareness of previously repressed information is called a: a. Cognitive approach. b. Psychoanalytic approach. c. Gestalt approach. d. Behavior approach.

B: Psychoanalytic approach. This approach is built upon the concepts and theory of Sigmund Freud and others who have followed him. The approach is also sometimes called a "psychodynamic" approach.

Mrs. Wilson lives in a quiet neighborhood on the outskirts of town. Everyone on her block steers clear anytime she is outside of her home. Neighbors describe Mrs. Wilson as being moody and frequently placing herself in chaotic relationships with men. The ambulance can be seen at her home at least twice a month for self-injurious acts she has committed upon herself. Mrs. Wilson has cut her wrists and attempted suicide many times. What kind of disorder is Mrs. Wilson suffering from? Narcissistic personality disorder Alter-ego disorder Borderline personality disorder Impulse control disorder

Borderline personality disorder

A 33-year-old lawyer is convinced his promotion was refused because two senior associates dislike him. He was told he needs further experience, but that the firm would like to consider him for promotion in six months. He tells his social worker that he knows they turned him down because he is too aggressive and that they will deny his promotion again. The social worker's best response is: A. "Yes, you are probably too aggressive for them." B. "How could you believe something for which you have no evidence?" C. "You may be right, but they offered to reconsider you." D. "That is the way law firms are."

C

A 38-year-old client with a history of chronic alcoholism develops short-term memory loss. He compensates by inventing stories. He seems confused and disoriented. The most likely diagnosis is A. psychogenic amnesia. B. Alzheimer's Disease. C. Korsakoff's Syndrome. D. Tardive's Dyskinesia.

C

A child can walk well but still runs with an awkward gait. She enjoys playing and pushing large objects such as carts and wagons. She plays with other children, but is not able to play with them in a cooperative way. Her vocabulary is about twenty-five words and she is able to put two to three words together to express an idea. This child would be considered to be at the developmental age of: A nine months B twelve months C two years D four years

C

A child is referred to a school social worker for being truant from school. The social worker asks the parents to come in and at the intake interview it becomes obvious that there is a problem in the home that is disrupting all communication. The parents are being evicted from their apartment for non-payment of the rent. The FIRST step in the intervention process is for the social worker to: a. help the parents to refocus their attention to the child's behavior b. arrange a session for the child without the parents c. work with the parents on the eviction issue d. help the parents talk to the child about their concerns for eviction

C

A classroom teacher refers a sevenyearold child to a school social worker because the child consistently comes to school tired, unclean, and unable to concentrate on lessons. The social worker's FIRST action should be to: A make a referral to child protective services B refer the child for academic testing C meet with the parents and the child D refer the child to the school nurse

C

A client complains that if the social worker really cared about him like his mother does, then the social worker would not charge him for treatment. This is an example of A. counter-transference reaction. B. sublimation reaction. C. transference reaction. D. subjective reaction.

C

A client has been evicted and is in need of immediate housing. The client's social worker owns an apartment building and has an available unit. The social worker should: A allow the client to temporarily rent the unit the social worker owns at a reduced rate B refer the client to several apartments, including the one owned by the social worker C refer the client to apartments owned by anyone not personally known to the social worker D refer the client to an apartment owned by a close friend of the social worker who will give the client a reduced rat

C

A client who is unable to tolerate frustration had a tendency to discharge tension by action is said to A. act-in. B. act-out. C. decompensate. D. sublimate.

C

A common psychiatric diagnosis found among South East Asian clients, particularly Cambodians is A. Psychosis. B. Major Depressive Disorder. C. Post Traumatic Stress Disorder. D. Phobias.

C

A couple consults a social worker because their child is disobedient. The social worker is aware that the couple has little understanding of child development and ageappropriate behavior. To most effectively serve this family, the social worker should FIRST: A let the parents know that their expectations are inappropriate B discuss behavioral techniques to assist the parents in achieving their goal C acknowledge the parents' concern and explore with them possible methods of intervention D recommend parenting classes for the parents to increase their understanding of child behavior and parent effectiveness

C

A couple expressing extreme anger at each other come to a social worker for marital counseling. The social worker should FIRST: (A) explore the goals the anger is masking (B) clarify whether each partner is willing to hear the other (C) acknowledge the anger each partner has toward the other (D) use the interview to probe for positive feelings each has for the other.

C

A criterion that is not a measure of a client's motivation is A. the level of discomfort. B. The level of hope. C. ego strength. D. ability to see himself/herself as able to change.

C

A family comes to an agency complaining of their child's behavior problems. As a result of the assessment phase, the social worker believes that the marital relationship is the primary problem. The social worker should focus on: (E) the parents, confronting them with their relationship issues (F) the child and the ways the parents should react to the behavior (G) a discussion of the social worker's perception of the marital relationship (H) developing the child's insight into behavior as a reaction to marital stress

C

A family seeks counseling help after the accidental death of one of their two children. The social worker's genuine concern and "over involvement" with the family is noticed by the social worker's supervisor. The supervisor should A. commend the social worker for being so dedicated to his work. B. arrange for an immediate transfer of social work services for this family to a more experienced social worker. C. involve the social worker in assessing the effect his subjective reactions may have on the quality of his work and on his clients. D. thoroughly explore the social worker's inner conflicts regarding the nature of this tragedy and suggest the necessity for an immediate resolution.

C

A major difference between clinical social work and psychiatry is A. the use of verbal techniques. B. the use of nonverbal techniques. C. the ability to prescribe medications. D. a focus on intrapsychic issue

C

A man is seen in the ER with complaints of visual hallucinations, confusion and restlessness. He also has chills, dilated pupils and nausea. He says "nothing is wrong, I just need sleep". Which of the following substances is most likely the cause of the condition? (a) alcohol (b) marijuana (c) cocaine (d) barbiturates

C

A mother and her 15yearold pregnant daughter seek consultation for the daughter's unplanned pregnancy. The mother wants the baby to be released for adoption because she is in ill health and cannot help her daughter raise the baby. The social worker's FIRST response should be to: A encourage the young woman's mother to consider other options B help formulate an adoption plan C meet with the young woman alone to explore her feelings and wishes D suggest that the mother participate in the treatment sessions

C

A newly arrived immigrant family has a 14-year-old daughter in junior high school. The school is encouraging her to become involved in after-school activities. The girl tells the school social worker that she is not permitted outdoors after school. The social worker should FIRST: A encourage the parents to allow the student greater freedom B tell the student that she can stay out during daylight C facilitate interactions with peers during school hours D refer the student to a support group

C

A nine-year-old student was referred to the school social worker because of aggressive behavior on the playground. In the first telephone contact with the child's mother, the social worker learns that the child has become more aggressive at home since the mother's remarriage three months ago. In this situation, the social worker's FIRST task should be to: A help the teacher to design a behavior modification program B intervene with the child on the playground C meet with the parents for an assessment interview D place the child in a peer support group

C

A social worker meets with a couple who refuse to provide appropriate medical care to their child with a life threatening illness because of their religious beliefs. The child is receiving some care within the guidelines of their church's ministry but the child's condition is deteriorating. How should the social worker respond to this dilemma? A Take no action on the basis of client confidentiality B Not report the situation because the family's right to self-determination restricts the worker's actions C Report the situation to child protective services D Continue to strongly encourage the family to take a different action in the best interests of the child

C

An eightyearold was brought to a family service agency by her parents because of enuresis and poor school performance. After the initial assessment, the FIRST course of action for the social worker should be to: A obtain a complete social history B recommend psychological testing C refer the child to a physician to rule out organic problems D request school records

C

An elementary school teacher has placed a disruptive child away from all activities for a predetermined, though short, period. This is an example of a A. positive reinforcement. B. negative reinforcement. C. time-out strategy. D. vicarious reinforcement.

C

M. a social worker in a human service agency has recently been tasked with conducting a cost- benefit analysis for the agency and outlining the services that were provided. In beginning this project , the most difficult task for M. to complete is: a. determining the actual services that the agency provides b. creating a control group to use in the analysis c. operationally defining the outcomes that are to be studied d. clearly identifying what therapeutic techniques were used

C

N is an 8 year old child who was the victim of severe physical, emotional, and sexual abuse. Based on this abuse, he has been placed in a therapeutic foster care home. While placed in this new foster home, a 15 year old child has reportedly sexually and emotionally abused N. what is the FIRST thing that the social worker should do to assist N? (a) ask the foster parents to take disciplinary action against the 15 year old (b) remove the child immediately from the home and place him in another foster home (c) visit the home immediately and assess the situation (d) initiate disciplinary action against the 15 year old removing him from the foster home

C

The purpose of feedback in a social work interview is to: A allow the social worker to confront distortions in the client's thinking B summarize the main points of the interview C let the client know the social worker understands the issue D give the social worker the opportunity to self-disclose for the benefit of the client

C

When N. a social worker uses the art of empathy in the therapeutic setting, she is: a. feeling sorry or distressed for the client b. showing compassion and/or mercy for the client c. perceiving, experiencing and responding to the emotional state of the client d. addressing the emotional crisis a client is exhibiting

C

When working with a client, concreteness is often sought as part of the therapeutic intervention. From the choices below which one should NOT be used to illicit concreteness in the therapeutic setting? a. to focus the client in the here and now b. to help establish a clear direction for intervention c. to avoid any emotionally charged topics d. to clarify what the client is saying

C

Which of the following is an example of triangulation? a. the child becomes the parent b. a couple, where one of the partners is having an affair c. when one parent tries to secure the allegiance of the child to his/her views against the other parent d. a relationship where the parents join together

C

When addressing a toddler during the interview, the health care provider should: A) ask the child, before the caretaker, about symptoms. B) use nonverbal communication. C) use short, simple, concrete sentences. D) use detailed explanations.

C Use short, simple, concrete sentences

You have gotten into a discussion about ego defense mechanisms with another social worker. They present you with a sheet and ask you which of the following defense mechanisms is NOT described appropriately. A) A woman who hates her mother goes out of her way to always buy her pretty things: Reaction formation B) A man with a high level of aggression becomes a local butcher: Sublimation C) A husband who is cheating but accuses his wife of having an affair to hide his behavior: Displacement: D) A young wife who goes home to visit her parents for 3 weeks and begins to act very child-like: Regression.

C) A husband who is cheating but accuses his wife of having an affair to hide his behavior: Displacement: A husband who is cheating but accuses his wife of having an affair to hide his behavior is projection, not displacement. When people project, they identify their negative emotions, beliefs, or traits in someone else. Displacement involves an individual transferring negative feelings from one person or thing to another. An individual may "take out" their anger on others.

You are working with the son of an elderly parent. They are concerned about the father's driving and would like to pursue the necessary legal guardianship in order to keep him off the road. Of the following four circumstances he could describe regarding his father, which on would be the WORST reason for pursuing guardianship in regards to the driving issues. A) His father displays the inability to take care of his basic needs. B) His father has gotten lost while driving in familiar neighborhoods. C) His father is a danger to self or others D) His father displays advanced signs of dementia and is no longer able to manage his financial affairs

C) His father is a danger to self or others this is an issue of crisis intervention. There is no indication his danger extends to his car as there is no indication of a decline in the judgment or spatial skills needed to drive and navigate.. This is a phrase which is normally used in relation to suicidal ideations or self-hard behaviors. It will often require an involuntary commitment. But it has no impact upon his driving.HINT: When answering the questions on the test, it is important to use only the information given in the question, and not to read anything else into it. The moment you hear yourself saying, "In this situation it could be..." you are in trouble.

John has withdrawn from his social circles and has begun to display rather peculiar habits when his friends come over to visit. All of his friends notice small behavior changes, which include odd statements, reference to things that "cannot be" and a comment regarding his beliefs that his dog may be listening into his telephone conversations. He is not known to use any drugs and has had no medical history. Because John is mild mannered and pleasant, his friend have felt his behavior is odd but not a major concern. When Cecelia went to visit him yesterday, he had not bathed in several days. His bird feeder was removed from the backyard and was sitting on the table. When she asked about it, he smiled pleasantly and said in a calm tone of voice, "The birds have been gathering around the bird feeder in order to spy on me and watch what I do during the day. So, I took down the birdfeeder so they can't come around anymore." He mentioned that he was happy because his brother started to call him again and they had a great conversation on the telephone that lasted for about an hour this morning. Cecelia knows that John had only one brother and he died in a car crash several years ago. Cecelia reported this to her friends and is very concerned. She calls a social worker she knows from work and they recommend John be evaluated for a psychotic disorder. Her social worker friend tells her that it is possible John is suffering from Schizophrenia. Cecelia goes online and learns the following about the onset of schizophrenia: A) It affects about 5% of the population and once treated with medications the person should have no further problems. B) It affects about 5% of the populations and most people continue to have symptoms throughout their lives. C) It affects about 1% of the population and treatment is often provided using neuroleptic medications, which can cause Tardive Dyskinesia and may have to be taken on a lifelong basis. D) It affects about 1% of the population and can cause some incapacity of social and work functioning, but the functioning incapacity is seldom profound.

C) It affects about 1% of the population and treatment is often provided using neuroleptic medications, which can cause Tardive Dyskinesia and may have to be taken on a lifelong basis.

You are working for an agency that takes walking referrals. In the middle of your shift you have a walk in referral, concerning a female who is a battered woman. She presents as scared and paranoid. Of the following actions, which intervention should you complete first ... A) Immediately begin assessment to begin case planning B) Assist the woman and identifying life patterns which should lead to abuse C) Obtain referrals for immediate safety D) Report the abuse to law-enforcement

C) Obtain referrals for immediate safety When you were dealing with the victim of domestic violence, the first thing you should do is ensure immediate safety. Immediate safety not only is required for the victim, but also of your agency and the people currently in your agency. When you are dealing with the domestic violence issue you don't know if the batter is following the victim and may storm into your agency. Safety is of the utmost concern.

You are working with a Strategic Family Therapist who uses a number of techniques derived from the communication model in family therapy. One of the techniques in this model is called "prescribing the symptom". The BEST way to understand and describe this technique is ... A) relabeling a symptoms in order to change its meaning B) redefining a symptom in order to create systemic confusion. C) a type of paradoxical intervention to overcome resistance

C) a type of paradoxical intervention to overcome resistance

We are moving into a more evidence-based world, and there is a movement to accept and provide therapy under practice theories which have evidence to validate their treatment approach. You have been asked the following question by your supervisor. Of the following practice theories, which one has the most empirical evidence to show it is successful? A) psychoanalytic B) social learning C) cognitive therapy D) gestalt

C) cognitive therapy Cognitive behavior therapy (CBT) is a type of psychotherapeutic treatment that helps patient's understand the thoughts and feelings that influence behaviors. CBT is commonly used to treat a wide range of disorders, including phobias, addiction, depression and anxiety.

You have just begun working at an agency with a lengthy history. It has been in the community for many years. You attend your second weekly meeting and are very surprised at the interactions. It appears to you that the entire agency is caught up in a very solid "groupthink" atmosphere. You realize you need to begin to minimize the harmful effects of this groupthink. The BEST way to accomplish this task and increase the group's ability to make better decisions is to ... A) assist the group with exercises to increase group cohesion and interdependence. B) ask the group leader to present a his favorite solution at the start of meetings. C) encourage constructive dissidence and structured criticism. D) ask the group to make more risky decisions

C) encourage constructive dissidence and structured criticism. Groupthink is a tendency by groups to engage in a concurrence seeking manner. Groupthink refers to a deterioration of mental efficiency, reality testing, and moral judgment that results from in-group pressures. Groupthink occurs when group members give priority to sustaining concordance and internal harmony above critical examination of the issues under consideration. According to Irving Janis, the pioneer of Groupthink, it is a quick and easy way to refer to a mode of thinking that people engage in when they are deeply involved in a cohesive in-group, when the members' striving for unanimity override their motivation to realistically appraise alternative course of action.

You have been working with parents who have an authoritative style of parenting. They encourage their children to be independent but still places controls and limits on their actions. You know that specific parenting styles will lead to specific outcomes. The BEST way to describe the children produced by the authoritative style is... A) dependent, passive, and submissive. B) disobedient, aggressive, and rebellious. C) independent, self-confident, and self-controlled. D) moody, creative, and independent.

C) independent, self-confident, and self-controlled. Authoritative parents will set clear standards for their children, monitor the limits that they set, and also allow children to develop autonomy. They also expect mature, independent, and age-appropriate behavior of children. Punishments for misbehavior are measured and consistent, not arbitrary or violent. Authoritative parents set limits and demand maturity, but when punishing a child, the parent will explain his or her motive for their punishment. They are attentive to their children's needs and concerns, and will typically forgive and teach instead of punishing if a child falls short. This is supposed to result in children having a higher self esteem and independence because of the give-take nature of the authoritative parenting style. This is the most recommended style of parenting by child-rearing experts

As a new social worker you find yourself being told by your supervisor you need to be more confrontational. This seems at odds with the social work mission, until you supervisor explains the primary purpose of confrontation is to: A) demonstrate accurate understanding B) help a client change her view of a problem C) make a client aware of inconsistencies D) help the client identify alternatives to her present behavior

C) make a client aware of inconsistencies Client's often have inconsistent views and values concerning a specific problem. This is a HUMAN problem we all fight with. Conflicts between what a client 'thinks' should be the solution and what the actual solution will be is the bread and butter of therapy. Just because we know we should do something does not mean we will act in the appropriate manner. (Think about cigarette smoking as an example)

You are discussing behavioral therapy and family systems theory with another social worker. They indicate they see a number of systems at play in the family which causes information to be used by the family to change or stay the same. They are interested in an appropriate nomenclature for this system. They ask you what they would call the process which acts to correct a family system in trouble and help restore it to a previous state of equilibrium. You know the BEST answer is... A) a feedback loop. B) positive feedback. C) negative feedback. D) neutral feedback loop

C) negative feedback. This concerns the concepts of entropy... Entropy is also defined as disorder in a system If a system has overall a high degree of negative feedback, then the system will tend to be stable. A negative feedback loop has been likened to a homeostatic system, in which the feedback loop provides information that returns the system to some preset level and reduce deviation causes to the system.

Your supervisor at a small mental health agency makes a referral to you for individual counseling. During the assessment, the client lets you know they have been a friend and acquaintance of your supervisor for more than 15 years. After you have completed your fourth session with this client, your supervisor asks you to see the client's file since she was the person who made the referral to you. Your BEST response is to... A) give your supervisor the client file. B) give your supervisor the file 'facesheet' only C) refuse to provide the file until the client signs a release of information D) terminate services with the client and refer to another agency

C) refuse to provide the file until the client signs a release of information Ethical dilemma question... This is what is known as a 'rock and a hard place'. Your supervisor should have disclosed her relationship with the referral up front, so you could determine if you wanted to take it. Your supervisors' request to see the client file is completely inappropriate. The reason the information should have been given up front is simple. You may report to your supervisor with questions and concerns, but on this particular case, your supervisor should have laid out an alternate supervisory chain for you. They should recuse themselves from the case and leave all information alone.

You are working with an agency whose mission statement is to help individuals recently released from a psychiatric inpatient program find work and adjust to life in the community. They provide an array of services. However, the level of prevention they operate on is BEST described as... A) primary prevention. B) secondary prevention. C) tertiary prevention. D) non-crisis intervention.

C) tertiary prevention. 1. Primary intervention is a program that seeks to stop problems before they start. 2. Secondary intervention is defined as locating an at-risk group and providing them services before they become involved in trouble. 3. tertiary intervention. All interventions which occur after the problem in an attempt to keep it from happening again or ameliorating the problem to reduce the bad effects are tertiary prevention programs.

You have been asked by a social work student about the origin of anxiety according to Sigmund Freud and his psychoanalytic theory. Your understanding of Freudian theory allows you to explain the BEST answer is ... A) emotional reactions to strangers during infancy. B) the need and ability to repress id impulses. C) the process of resolving conflicts between the id, the superego, and reality. D) the process of freeing yourself from symbiotic relationships.

C) the process of resolving conflicts between the id, the superego, and reality.

You are working as a discharge planning social worker for the local hospital. You have been assigned the case involving a 53 year-old man and his 78 year old mother. After a medical exam, the physician states that the mother's memory is impaired and she is incontinent of both bowel and bladder, he also states that according to her history she has fallen three times in the last 9 months, the latest fall resulting in her current hospitalization. The physician has suggested that she be placed in a nursing home. You are discussing this with the son and ask questions about support structures which he pleasantly avoids answering. He listens politely and nods occasionally. He then thanks you and tells you his mother will be fine living at his home. What is your BEST analysis for the son's response? A) the son is unwilling to discuss his family business with you. B) the son does not agree with the physician and wants to continue to care for mom C) the son is in denial about the status of his mother's health and condition. D) the son believes the doctor is trying to take away his responsibility to care for his mother.

C) the son is in denial about the status of his mother's health and condition. The key to understanding this question is in the sentence which deals with the mother having fallen three times and the third fall being responsible for her current hospitalization. The son may feel very obligated to attend to his mother's needs or he may be unable to understand her needs, but continued falls will quite likely result in her death. His refusal to answer questions is an aspect of his denial. If he answered your support questions, he would have to begin justifying how he was able to take care of her. Given the limited information, you need to assume denial in deciding how to continue approaching the son.

You are seeing a 23-year-old female client who appears rather manipulative. She lives at home with her mother and father and does not work or attend school. She makes the following statement during your second session, "If you really cared about me like my mother cares, you would not charge me for treatment." The client has just performed a .... A) Displacement reaction B) counter-transference reaction C) transference reaction. D) sublimation experience.

C) transference reaction. The client has just used you as a movie screen and projected her mother onto you and is now transferring her emotions and desires onto the projection of her mother. As a therapist, you must withdraw the screen so the projection fails and therefore the transference is unsuccessful. This will cause the manipulation to fail and you should see the client attempt to use other mechanisms. Remember, the client is using this mechanism (set of behaviors) because it is very functional for getting their needs met. You have to assume that they are in therapy because they are running into situations where there old mechanisms no longer work as well, if at all. This is the time to explore and learn to use new mechanisms.

In working with a client who exhibits a narcissistic personality disorder, the social worker frequently becomes 1) the object of the client's wishes to be mirrored; 2) an object of the client's anger; 3) idealized by the client to fulfill functions that have remained unfulfilled by the client's parental figure; 4) angry because of the client's distorted projections. A. 1 and 4 B. 1, 2, and 3 C. 1 and 3 D. 4 only

C. 1 and 3Self-psychology theory holds that normal development involves having one's goodness and uniqueness mirrored back by the parents and feeling a sense of trust and safety in the presence of perfect parents. Narcissistic personality disorders originate from problems in this area of development. The transference will be in the form of seeking mirroring from, or idealizing the worker.

Amanda states that she cannot stay in her current marriage of 15 years since her husband's abusive behaviors are escalating. She reports that since he has lost his job, he often takes things out on her. She states she has a plan for leaving this abusive relationship because although she has allowed this abuse in the past she believes the potential that he will harm her or her children is too great. This is an example of: A. Assimilation B. Modification C. Accommodation D. Application

C. Accommodation----Needs to modify the relationship to protect her children

Alysha meets with a social worker and discusses self-perceived problems. She reports complex mood swings that last for weeks and sometimes months. Alysha's main concern is that her manic behaviors may one day become life threatening. When the client is in a more sullen mood, she feels sad but somewhat safer. When completing a full assessment, what might the social worker diagnose Alysha as having? A. Dissociative identity disorder B. Manic depressive disorder C. Bi-polar disorder D. Post traumatic stress disorder

C. Bi-polar disorder Manic depression is no longer used as a term.

A social worker is meeting with a 55- year old lesbian who has a history of depression and substance abuse. The client says she feels "pretty good" about most things in her life, but wants to have a lont-term relationship and has never achieved this. She reports feeling disconnected from the gay community because she is politically conservative and "doesn't fit in". The social worker should FIRST:A. Recommend that the client join a group for gay singles in the area.B. Make a psychiatric referralC. Complete a biopsychoscial assessmentD. Explore the disconnection between the client's sexual orientation and political beliefs.

C. Complete a biopsychosocial assessment.------ Social worker needs to know more about the client's overall functioning and history before further explorations or referrals can be made. A, B, and D are all good interventions, but should be done after the assessment is complete.

While worker with a 20 year old client you discover that her Uncle sexually abused her for the last five years. She is concerned he will do it to her nephew as well. As a social worker you are ethically bound to:A. Report the case of sexual abuse because it first occurred whenever she was a minor.B. Report the case of sexual abuse because it is currently going onC. Do not report current case, only report this case if you feel a cild is currently in dangerD. urge the client to confront the abuser so it won't happen to someone else

C. Do not report current case, only report this case if you feel a cild is currently in danger------Report only in you feel a child is in danger

The Sustaining system consists of A. The individual's family B. The "helping" professions C. Institutions of the larger society D. A physiological system that sustains life

C. Institutions of the larger society

A social worker in a residential facility meets with an adult client with Down's Syndrome. The client shares that when she met with her psychiatrist last week, he told her she looked pretty and touched her breasts. What should the social worker do FIRST? A. Meet with the psychiatrist to determine what happened B. Provide psychoeducation on appropriate vs. inappropriate touch C. Make a mandated report to adult protective services D. Report the psychiatrist to their medical board

C. Make a mandated report to adult protective services The correct answer is C: make a mandated report to adult protective services. This falls under reportable abuse or neglect, and the exam wants to make certain that you know to take this as seriously as any other disclosure. You do not want to dismiss a client's report due to having Down's Syndrome or any other intellectual developmental disorder. A is incorrect because we don't go to the suspected abuser when someone shares something reportable. B is incorrect because there is no reason a psychiatrist should touch a client's breasts. D is something we may do, but it wouldn't come before making the mandated report.

Older teens and young adults self-report symptoms of this serious mental illness, usually before the age of 19. Contributory factors include environment, genetics, and psychological and social processes. Symptoms appear as auditory hallucinations, bizarre delusions, paranoia, and disorganized speech. This disorder mainly affects cognition, but also contributes to emotional and behavioral problems. What illness is being described? A. Dissociative identity disorder B. Bi-polar disorder C. Schizophrenia D. None of the above

C. Schizophrenia Answer: C - Schizophrenia tends to cause other conditions such as anxiety disorders and major depression. Substance abuse is at 40% with social problems that lead to poverty, unemployment, and homelessness. The average life expectancy of individuals with schizophrenia is 10 to 12 years less than those without the disorder.

A clinical social worker wants to hire a bookkeeper to file insurance reimbursement. One of the applicants turns out to be a former client. Since therapy was terminated over a year ago, the social worker decides to hire the client. This situation is:A. ethical since termination of therapy was over a year agoB. ethics is not an issue here, if the applicant is competent she should be hiredC. This could be unethical because hiring a former client would create potential for a dual relationshipD. Not to hire the applicant would be seen as discrimination C. This could be unethical because hiring a former client would create potential for a dual relationship

C. This could be unethical because hiring a former client would create potential for a dual relationship

Joan tells her social worker that when her husband Mike comes home from work after having a few drinks, he is often verbally and physically aggressive with her and their children. Mike's drinking has increased and he is clearly using alcohol regularly. He has been arrested twice for driving while intoxicated. Joan is obviously fearful and asks the social worker for assistance. What should the social worker do first? A. Ask Mike to come in for treatment and work with him around his alcohol problem. B. Suggest family treatment to encourage Mike to seek help. C. Work with Joan to help her protect herself and the children, either through the child protective agency, a shelter or through a court order of protection. D. Suggest a medical examination for Joan and the children.

C. Work with Joan to help her protect herself and the children, either through the child protective agency, a shelter or through a court order of protection.The first priority in this type of situation is to insure the safety of Joan and the children. Mike's problem with alcohol is secondary to the potential for abuse that he presents. Moreover, in this scenario, the primary client is Joan.

If a women admits during a counseling session that her husband is abusing her, the social worker is obligated to do the following: A. report the incident to the proper authority with or without client permission B. report the incident with client permission only C. assist the woman to become aware of services available to her in regard to the abuse D. nothing is required of the social worker, it is up to the woman to ask.

C. assist the woman to become aware of services available to her in regard to the abuse ----reporting is not warranted

A young, married woman is admitted to a city hospital in acute cardiac decompensation. While there she writes a letter to the medical social worker in the voluntary hospital where she was previously treated and asks for a transfer to that facility. The work er receiving the letter should A. notify the patient that this is not the worker's function. B. ignore the letter, since the patient is in treatment. C. contact the social worker in the city hospital and ask him or her to see the patient. D. arrange for a transfer, as per her written request.

C. contact the social worker in the city hospital and ask him or her to see the patient.The client's request reflects an attachment to her previous social worker. However, she needs to be in a medical facility at this time. Her former social worker should contact the social worker in the current facility. This is a response which is caring and respects the history of the relationship between the client and the former social worker. The current social worker would let the client know that her previous worker called. This could allow the client to feel that there was a connection between her previous facility and her current one. Such a connection can feel holding and safe to clients.

As the social worker develops an understanding of the basic behavior of the client, he should A. continue to observe the client, keeping this knowledge in mind. B. share this information with the client. C. direct the client toward awareness without interpretation. D. gauge when the client is ready to both share and contribute to understanding his own behavior and this discuss it.

C. direct the client toward awareness without interpretation.The goal of clinical work is to promote insight so the client may change behavior. Answer A suggests that the worker should keep the information to himself, which has no clinical impact. It is important that the worker do the least intrusive intervention first, and encourage the client to discover the insight themselves. Therefore it is not useful for the worker to inform the client of the clinician's observations (C). Answer D states that the worker influences the conversation in ways that encourage the client to discover.

Because of an unexpected upsurge of migrant workers during the summer the director of a small social agency must cancel August vacations. He expects that the decision will be met with resistance and dissent. To engage staff effectively he should A. send a memorandum to the staff describing the change and the reason. B. hold a general staff meeting where the issue is noted as an item. C. discuss the issue with subordinates and allow for questions, alternate possibilities and objections. D. discourage dissent within the informal structure by asking the supervisors and informal leaders to meet with the director.

C. discuss the issue with subordinates and allow for questions, alternate possibilities and objections.Any policy that affects people's lives and is simply dictated without opportunity for discussion is likely to stir opposition. Opening the issue for discussion creates opportunities for change and provides staff with a sense of participation. Even if the outcome is the same, staff will feel they had an opportunity to represent their interests and to offer alternatives.

A breach of ethical conduct may exist when a social worker A. uses a client's first name B. exchanges information about local restaurants in the area C. exchanges social work sessions for yard work to be completed by the client D. discusses what happened at a local sporting event

C. exchanges social work sessions for yard work to be completed by the client ---- exchanging services are not considered acceptable practice

J, a 28 year old male client, discloses to his social worker in the second interview that he has had a sexual relationship with his previous social worker. He states that he is no longer in therapy with her but that he still sees her on occasion in a nonprofessional sense. The current social worker's BEST ethical course of action is:(a) report the other social worker to the governing board of social workers(b) do nothing in terms of reporting it, and respect client confidentiality(c) explain the options available to your client if he chooses to report it(d) give the client the number of the licensing board and ask him to call

C. explain the options available to your client if he chooses to report it--explain options and let the client choose for himself

Mrs. Taylor was referred to a social worker after her alcoholic husband was hospitalized following an acute episode. She felt there was no reason to see a social worker as she had consulted with several doctors and read popular books on alcoholism. She feels that her husband sees her as a mother symbol and blames her for all his problems. She offered him a psychology book to read that she thought would help him understand his behavior. Mrs. Taylor found passages from an abnormal psychology book that she thought described her husband. She read sections of the book to Mr. Taylor, especially chapters describing personality disorders, egocentricity and dependency. Initially, treatment with Mrs. Taylor would involve A. telling her that her approach has merit and suggests sensitivity to her husband. B. reflecting that her approach is overly intellectualized and futile. C. exploring her feelings about her personal and family situation. D. assuming that she is reluctant to be engaged and attempt to deal with resistance.

C. exploring her feelings about her personal and family situation.The first step is to offer the client an opportunity to talk about her feelings. Only when there is evidence of resistance is it useful to respond. According to theories about working with family members of people with compulsive behaviors, it is important to begin to challenge the idea that it is their job to save the addicted person. Therefore, the worker should focus on the family member's own feelings and encourage them to feel that their experience is important.

If an attorney requests the record of a client and the social worker has received a verbal okay to release the record from the client the social worker should: A. Release the record as requested B. wait until the record is subpoenaed to court C. get the client's consent in writing D. assert professional privilege and refuse to release the record as you believe it might be harmful to your client's case.

C. get the client's consent in writing ----Always get consent in writing

A board function that is inappropriate, except in rare instances is A. fund-raising. B. policy formulation. C. monitoring day to day activities. D. establishing new organizational goals.

C. monitoring day to day activities. Board functions can vary but it is highly unusual for a Board to intrude on the domains of the executive director. Day-to-day program operations are a clear boundary that Boards should observe. The creation of board policy directions, planning, fund-raising, and future directions are clearly within the parameters of Board responsibility.

A client at a mental health clinic who has been diagnosed HIV positive and now has AIDS has stopped working. As the illness progresses, he is uncharacteristically depressed and apathetic. He has memory lapses, has difficulty concentrating, appears confused and withdraws socially from friends and family. At the most recent interview the social worker noticed that he walked with a slight shuffle. The social worker might attribute these changes to A. growing depression associated with the client's life situation. B. diminished mental and physical capacity engendered by the client's growing physical weakness. C. physical and mental symptoms associated with the physiological effects of HIV. D. a consequence of being shunned by his friends and family.

C. physical and mental symptoms associated with the physiological effects of HIV. While all the responses are, to some extent factors in the treatment of HIV positive clients, approximately 50% of AIDS patients experience some form of neurological impairment. The particular clusters of symptoms noted in the stem of the question suggest HIV encephalopathy. the most vivid clue is the changed walk, in combinated with emotional symptoms.

Fred, age 35, is participating in his third family counseling session with his wife, Norma. Her major complaint concerns Fred's drinking. Fred feels his drinking habits are within normal limits. He has a responsible job that requires considerable travel and is home several evenings and on weekends. He wants to relax and feels that drinking 8 to 10 beers after working all day isn't unreasonable. Fred says that he rarely has more than two drinks daily at business lunches, and perhaps one more with his colleagues after work. In working with this couple, the worker should A. help Norma adjust to Fred's habits as his drinking is not excessive. B. suggest that Fred take Norma out more often to soften the tension between them. C. suggest that Fred confront his excessive drinking as it seriously affects his health, work and family life. D. ignore the drinking as the problem is Norma's lack of acceptance.

C. suggest that Fred confront his excessive drinking as it seriously affects his health, work and family life.Fred's drinking is excessive. He is consuming the equivalent of 10 shots of whiskey daily and is in denial about his abuse of alcohol. Whether in a family or individual treatment, his alcohol abuse is a major presenting problem.

In order to encourage reluctant patients from third world countries to adopt Western health methods and reject unhelpful traditional medical practices, a social worker should first A. take a tour of the health facility with the patient. B. arrange for the client to be seen in the home first. C. try to convince the oldest family member. D. try to convince the youngest family member.

C. try to convince the oldest family member. A marked characteristic of traditional societies is a hierarchical family structure and respect for the elderly. If an elderly family member can be convinced that modern scientific medicine is effective, it is likely that other family members will be easier to convince.

In statistical research, a "Type I Error" (also called an "alpha error," or a "false positive") refers to: a. Failing to reject the null hypothesis when the null hypothesis is false. b. A failure to randomize research participants, thereby potentially introducing bias. c. Rejecting the null hypothesis when the null hypothesis is true. d. Assuming a normal statistical distribution when it is skewed.

C: Rejecting the null hypothesis when the null hypothesis is true. A failure to randomize research participants will potentially introduce bias, and may provide grounds upon which to invalidate a study, but it is not a type I error. Assuming a normal statistical distribution when it is skewed will violate the assumptions necessary to apply a proper statistical model to the analysis of data.

The following criteria are all used to distinguish substance abuse from substance dependence except: a. Symptoms of substance abuse are usually less severe than those of dependence. b. The problematic effects of abuse are usually limited to family, finances, employment, and legal issues (e.g., driving under the influence), while dependence also involves significant physiological problems. c. Substance abuse typically involves narcotics, while dependence typically involves non-narcotic drugs. d. Abuse is typically limited to recreational use, while dependence involves the need for increasing doses for the desired effect and withdrawal symptoms of not used regularly.

C: Substance abuse typically involves narcotics, while dependence typically involves non-narcotic drugs. - dependence implies an addiction in which the user cannot stop using the drug. - abuse may occur — even more frequently than it should. However, a person who abuses drugs may do so in isolated instance.

During the initial session, a woman who is pregnant with her fifth child states that she wants to relinquish her parental rights by putting the child up for adoption. She and her husband are no longer together, but he agrees with the decision. Currently, she has lost her medical insurance and is in financial crisis. The best course of action for the adoption agency social worker is: a. encourage her to find other family members to take the baby b. help her to choose new parents for the baby c. help her with options and resources beyond the agency d. let her know the agency's philosophy

D

A social worker has begun work with a new client. In the first session the client tells the worker that she had a bad experience with a previous worker. She felt betrayed because the social worker revealed information to her husband without her knowledge. The worker should first A. reassure the client that she won't reveal anything to anyone unless authorized by the client. B. tell the client that with today's computerized systems, there are no secretes that are secure. C. reassure the client that the other worker probably had the client's best interest in mind. D. ask the client to tell her more about what had happened with the other social worker.

D

A social worker in a neonatal intensive care unit is approached by a hospital volunteer asking which infants may be available for adoption. The social worker's BEST response is to: A take the volunteer to view infants being placed for adoption B refer the volunteer to a lawyer representing the parents seeking to place an infant C introduce the volunteer to parents seeking to place an infant in an adoptive home D decline to share such information with the volunteer

D

A social worker on a home visit meets with N., a 75-year-old female, N. reports that she has no family or friends to assist in her care. She also reports that she feels helpless to defend herself and believes that neighborhood children are stealing her belongings from her yard. Her home is cluttered, but the woman seems capable of finding things and noting where they were left. She appears somewhat malnourished; although she has food in the refrigerator. The neighbors have complained and have repeatedly called the police reporting that the woman is terrorizing the neighborhood children by screaming at them from her yard. Based on the current information what is the BEST plan for the social worker to address this situation: a. arrange for home care b. call Adult Protetive Services c. help the woman to develop a plan to live alone d. do a complete assessment of the woman's competency to complete her own ADL's and live alone

D

A social worker on an interdisciplinary team at a residential center for children should make it clear to the residents that confidential disclosures are: A treated differently at times B maintained between the resident and the social worker C never shared with anyone D shared with the treatment team

D

An eight year old boy was sent to the social worker because he soiled his pants again. This is an example of: a. primary incontinence b. enuresis c. pica d. encopresis

D

As treatment progresses with Ira, a computer programmer who abuses cocaine and alcohol, the social worker realizes that he is highly unmotivated, but unable to fully stop. Which cognitive behavioral-approach might the social worker use to help him? A. Emphasize insight-based therapy to help Ira understand the underlying psychodynamic processes that promote his addiction. B. Suggest that Ira participate in a partial hospitalization program. C. Suggest that Ira participate in a family treatment program. D. Find out which situations seem to be associated with substance use and help Ira learn to avoid these.

D

During Mrs. S's first appointment in a family agency, the social worker encourages her to express her feelings about the recent placement of her child in a residential facility for the developmentally disabled. Mrs. S talks at length instead about her physical health problems. The social worker should: (A) use Mrs. S's remarks about her health to identify unresolved parenting issues (B) refocus the interview on the child's behavior and the reason for the residential placement (C) explore with Mrs. S how her health problems are related to the recent placement of her child (D) listen to Mrs. S's description of her health as a beginning of a relationship and means of assessing her self-perception

D

During a treatment group for first time offenders, aged 12 to 14, one group member starts bragging about how he stole several items from a neighbor's house. The other members of the group appear to be listening intently to what this group member is saying. The BEST course for the social work group facilitator to take is: a. after group report the adolescent's behavior directly to his parents b. individually, after the group session, confront the adolescent about his behavior c. tell the adolescent you must report this behavior to the juvenile authorities d. encourage the adolescent in the group setting to openly possible motivation for such behavior

D

During an interview with a client who has failed to meet the goals of a contract, the client blames the social worker for the failure. This statement is an example of: A rationalization B reaction formation C compensation D projection

D

In assessing risk factors for alcohol dependence among adolescents, the social worker would be most interested in A. the youth's involvement with friends who engage in deviant behavior. B. the youth's school performance and social relationships. C. the youth's developmental age. D. a biological family history of alcohol abuse.

D

In establishing a working alliance with the client, the social worker should A. establish the conditions for proceeding. B. subtly try to encourage a positive relationship. C. consider clinical objectives and get the client to agree to them. D. allow for the client's ideas and goals, understanding that conditions change as clinical work proceeds.

D

In evaluating the appropriateness of social work services from a majority social worker to a minority client, the social worker should consider all of the following EXCEPT: a. comfort level of the client with self-disclosure to the social worker b. decision that race can be a barrier to effective practice with the client c. social worker's ability to convey credibility in offering services to the client d. client's access to natural helping networks as an adjunct to social work intervention

D

In reacting to a client's presentation of material in a psychotherapeutic or clinical situation, the social worker should primarily A. deal solely with the concrete data presented. B. expand the date by interpreting on a deeper level. C. reflect back to the client using the same language and words where possible. D. abstract the basic theme of the communication and reflect it in a broader perspective.

D

In the beginning phases of establishing a professional relationship with a client, a social worker must convey a sense that the social worker's predominant feeling is A. a neutral attitude. B. a sense of objective observation. C. healthy skepticism. D. positive acceptance.

D

In the first therapeutic session, a mother tells the social worker that her baby constantly eats dirt and other things off the floor when crawling about. She states that she knows the baby is not hungry because it also happens after the baby has been fed. In assessment of this child, the social worker is most likely to suspect that the child suffers from: a. Rumination disorder b. Atypical Eating Disorder c. Childhood Schizophrenia d. Pica

D

In the initial session with a couple both state the reason for coming in as sexual relationship difficulties. The female partner stated that she no longer feels emotionally supported by her partner and has lost interest in having sexual relations with him. The male partner does not understand why his mate feels thi way reporting that his behavior has not changed in the ten years they have been married. Both individuals are very active in their careers and made the decision not to have children early in their marriage. The social worker should FIRST focus on: a. career patterns and choices b. focus directly on sexual relationship issues c. address the parenting decisions made earlier in the marriage d. focus on clarifying relationship issues

D

In working with a client over time, a social worker should communicate an attitude of A. concern about the client's moral responsibility for his behavior. B. openness, affection and permissiveness. C. accepting the client unconditionally. D. acceptance for the client with positive though conditional regard.

D

One disadvantage of self-help programs is that A. the participants are never fully equipped to offer the services they claim to provide. B. community groups involved in self-help rarely acquire the necessary financial resources. C. their scope is too limited to be of true service to the community. D. they rarely influence the larger institutional system.

D

The difference between "acute stress disorder" and "post-traumatic stress disorder" in DSM-IV is A. the length of time following trauma and length of time symptoms are experienced. B. the degree of trauma. C. that more somaticizing accompanies accompanies acute stress disorder. D. that PTSD has more flashback symptoms and occurs over a period of at least one month.

D

When a child is learning a new behavior all of the following factors are considered important ingredients EXCEPT: a. the cumulative effects of learning behaviors on overall development b. the environment conditions and circumstances that surround the behavior c. the current circumstance that surround the learned behavior d. the influence of innate learning capabilities

D

When a social worker is employed by an agency as a consultant, which of the following is NOT an accurate depiction of this role? a. the clients that are served by the agency will ultimately benefit from this consulting relationship or service b. the consultant will bring forth specialized knowledge and skill c. the consultant indirectly affects the direct skills of agency providers d. a consultant does not have to be sanctioned by agency administration

D

When faced by a hostile client in an agency setting, it is BEST for the social worker to: (A) suggest that the client's attitude is making the situation worse (B) accept the client's hostility and talk about nonthreatening topics (C) set limits and structure on the interview session (D) acknowledge the client's feelings and encourage discussion of them

D

When looking directly at the measurement of cost-effectiveness in a human service organization the primary focus should be on: a. creating an agency budget documenting materials and resources b. the relationship between what was expected to be the cost and what actually was c. economic benefits of the program in terms of the community d. calculating the cost of the program and comparing it to alternate means of providing patient services

D

You are working as a social worker for a school district. You have been given a case where the principal of a high school has refused a parent's request to place her son in a special education class. The reason stated by the principal is a report by the student's teacher which states that he is "lazy" and "refuses to do his work" and that he is not "a special needs kid". Your best chance of resolving this situation is to... A) discuss the student with the teacher in order to convince her the student needs to be placed in a special education class. B) talk with the mother and try to convince her that her son does not need a special education class and simply needs to adjust his attitude. C) review the file and come to your own conclusion, then submit a report to the principal. D) Arrange for an educational evaluation to determine the educational functioning level of the student and make your report based on the results.

D) Arrange for an educational evaluation to determine the educational functioning level of the student and make your report based on the results.

You are a non-hispanic social worker treating a Hispanic family. This family recently immigrated to the United States and has been here for about 4 years. At every session, they arrive 10-15 minutes late. You are beginning to question their commitment to therapy and find this late arrival behavior irritating. The BEST way to response to this behavior is to... A) realize that being late is a form of resistance and discuss their resistance with them. B) realize that perception of time and 'lateness' may not be understood the same by their culture C) consider a referral to a social worker with a similar cultural background. D) Discuss this behavior with them and explain your feelings about it and its possible impact on therapy.

D) Discuss this behavior with them and explain your feelings about it and its possible impact on therapy. Regardless of the cultural content of this interaction, the behavior (arriving late) is problematic. It reduces the time the family has in therapy and increases the length and the cost of therapy. There are many reason for clients to come late repeatedly. Often what we take for granted in our daily work is very hard for them to process. Direct discussion and a re-negotiation of expectations is appropriate here.

You have been working with a client for six sessions and they are covered by BCBS Insurance. They have elected to pay in cash rather than use their insurance benefits. Their insurance company sent you a letter requesting information about the client's progress and the current focus of therapy. Your best response is to... A) throw the letter in the trash and forget about it. B) send the insurance company a complete copy of the file. C) verify the validity of the letter and then copy and send the file. D) Send the requested information only with the client's written authorization.

D) Send the requested information only with the client's written authorization. While you could send the information with the client's verbal authorization, it would not be wise to do so. If the client later denied giving you authorization, it is a hearsay issue and cannot be resolved. A piece of paper with a client's original signature on it, in your file is a tremendous stress reducer in the event of litigation or complaint. I usually ask my clients to sign in BLUE INK, because it does not photocopy well and is easier to prove as genuine. Even if the client signs a release at the beginning of therapy, I would encourage discussing it with them prior to release and then letting them determine if release is still their preferred choice.

You have been asked to consult with another clinical social worker on a client. The social worker feels a diagnosis of Social Phobia is appropriate over a diagnosis of Agoraphobia. You know that the difference between these two diagnoses is BEST described as ... A) agoraphobia involves distress over the possibility of experiencing severe anxiety or a panic attack. B) agoraphobia involves a fear of acting in a way that will bring about humiliation or embarrassment. C) agoraphobia involves a fear of numerous social situations, while social phobia entails a fear of a single circumscribed social situation. D) agoraphobia involves a fear of places or situations, while social phobia entails a fear of people.

D) agoraphobia involves a fear of places or situations, while social phobia entails a fear of people.

You are working with a younger social worker and have been asked a question regarding personality disorders. Of the following personality disorders, which is four to five times more likely to be diagnosed in male patients than in female patients. A) borderline B) narcissistic C) obsessive-compulsive D) antisocial

D) antisocial

From the viewpoint of the Freudian Psychoanalyst, the purpose of interpretation as an analysis technique is to ... A) is appropriate only during the final "working through" phase of therapy. B) Useful in therapy with person's suffering from schizophrenia C) help the client develop transference D) helps the client understand the causes of his or her behaviors and beliefs.

D) helps the client understand the causes of his or her behaviors and beliefs.

You are seeing a female client in her thirties. Therapy is just beginning and you feel she is experiencing episodic depression. Using Aaron Beck's approach to the treatment of depression, the BEST response would be to A) arrange depression-inducing situations into a hierarchy. B) record his or her "automatic thoughts." C) imagine depression-inducing situations in exaggerated forms. D) instruct her to practice mood repair strategies.

D) instruct her to practice mood repair strategies. Aaron Beck was regarded as the father of cognitive therapy/ CBT

You have recently attended an in-service which talked about the psychologist, Margaret Mahler, you came away from it with the understanding that her concept of "individuation" results in ... A) achieving a unity among the psychological functions. B) the ability to establish relationships with others. C) incorporating the unconscious aspects of the personality into the conscious aspects. D) recognizing the individual existence of self and others.

D) recognizing the individual existence of self and others.

You have been asked to consult on a case with another social worker. They are seeing a male client and they are concerned about the possibility of suicide. Given your knowledge of the risk factors which are associated with suicide, which of the following would cause you the MOST concern for suicide potential... A) the client is between the ages of 17 and 30 B) the client has expressed a desire to kill himself but has no plan C) the client has expressed a desire to kill himself, has a plan as well as goals for his future. D) the client has expressed a desire to kill himself, has a plan and has no goals for the future.

D) the client has expressed a desire to kill himself, has a plan and has no goals for the future.

You have been asked to complete a behavioral assessment on a 12-year-old client. You know a behavioral assessment is different from a standard assessment because different areas are evaluated. Which of the following would NOT be an assumption during a behavioral assessment? A) the assessment would focus on behaviors repeatedly throughout the course of the intervention. B) the assessment would focus on specific aspects of the individual's behaviors. C) the assessment would focus on directly observable behaviors. D) would view outward behaviors as signs of an individual's underlying characteristics

D) would view outward behaviors as signs of an individual's underlying characteristics A behavioral assessment is concerned with the actual observable behaviors which are being manifest by the client. Behaviorism does not even deal with any underlying characteristics in the personality structure which may be affected by or affecting the behaviors.

A 35-year-old woman meets with a social worker for an intake session. The client reports that she hasn't been able to eat or sleep for several weeks, hasn't been going to work, and has been crying 'non stop' since an unexpected break up with her partner. She says 'I feel like my entire life and future are ruined.' What should the social worker do FIRST? A. Refer the client to an MD to rule out any medical issues B. Explore what led to the unexpected breakup C. Empathize with the client's sadness over the break up D. Ask the client if she's having any thoughts of suicide or self harm

D. Ask the client if she's having any thoughts of suicide or self harm The client has experienced a significant loss that has led to her feeling like her life and future are ruined. While it doesn't use the word 'hopeless' this is precisely what the client is conveying. Because of this, we need to first and foremost assess for suicide risk. When there are warning signs for risk of harm to self (or others), we must assess this first. After that, we could do A due to the physical symptoms, and then could move on to do C and B. When there are risk factors present in the question, we must start there and rule out any risk of harm to self before moving forward with anything else.

A social worker is working with a low-income family that has two young children. The children often miss school because of respiratory illness and other health-related problems. When asked about this, it appears that the mother and father also have had repeated health concerns. The family does not have health insurance but have been getting some services from a local health clinic. Upon investigation it becomes evident that the family lives in an apartment complex where the previous tenant was accused of making meth in the kitchen. The social worker should first: A. find out what other tenants think about the building B. arrange for the entire family to have a medical check-up C. encourage the family to relocate D. Contact the state health department to check the apartment.

D. Contact the state health department to check the apartment.---Once the apartment is confirmed suitable to be lived in other avenues can be explored.

The goal of the state when working with a family via dependency case is? A. Create an environment that promotes self esteem for the children B. Keep kids & their families together C. Provide an education for the child D. Establish a safe and permanent home ASAP

D. Establish a safe and permanent home ASAP

You are a social worker in an outpatient setting. A new client arrives for her first session with you. The client has noticeable hair loss and tells you that she is very tense and is having problems in her relationship with her boyfriend. Your first diagnosis would likely be A. Borderline personality disorder B. generalized anxiety disorder C. Schizophrenia D. Trichotillomania

D. Trichotillomania

When treating an impulsive and problematic child, the worker's most important task is to A. provide a positive antidote to the child's negative experiences. B. provide the child with painful experiences so that a weak ego can be strengthened with each new mastery of pain. C. provide a warm, protective environment in which the child can freely express himself or herself. D. accept impulsive statements and actions and help the child find constructive way to express them.

D. accept impulsive statements and actions and help the child find constructive way to express them.Children are often referred to treatment because of difficulty managing impulses. By accepting the child's impulses, the worker is indication to the child that s/he is acceptable. By helping the child learn other ways to express self, the worker is teaching the child that there is a difference between feelings (impulses) which are acceptable and some behaviors which are unacceptable. In this process, the child may well experience a positive antidote to prior negative experiences (A) and the child may feel protected (C).

An intake worker in an adolescent shelter is assigned a 14-year-old girl who has run away from her family. She does not wish them to know her whereabouts. When asked about her reasons for leaving home, she says that she has problems with her father, but does not volunteers any additional information. The worker would first A. notify the parents that she is at the shelter and request that they come in for a family interview. B. interview the parents. C. tell her that when she is ready to talk, a social worker is available. D. allow her to discuss taboo subjects.

D. allow her to discuss taboo subjects.A child who leaves home and presents at a shelter may have many reasons. However, a teenage girl who reveals "a problem with her father" may be unwilling to discuss embarrassing or threatening facts such as sexual exploitation. In this case, the worker would not initially contact the family, nor react with a neutral response. An attempt would be made to help the child express her concerns by letting her know she is protected, and that the worker will not be shocked by the situation.

A therapy patient with a history of drug abuse seems euphoric, excessively restless and far more talkative than usual. He speaks rapidly, relating many ideas quickly and without explaining their relationship to each other. He seems suspicious of everything the social worker says and relates a grandiose scheme for marketing a new product that he says he can develop. The social worker might suspect A. a psychotic episode. B. alcohol intoxication. C. dissociation. D. amphetamine intoxication.

D. amphetamine intoxication.The behaviors described are among the signs of amphetamine use and are similar to those of cocaine. Grandiosity, euphoria and high energy are often accompany amphetamine use. Other behaviors include hyper vigilance, impaired judgement, heightened sensitivity and changes in sociability. The symptoms usually disappear within 24 hours.

Marie and Tony are about to be married. They see a social worker to resolve relationship and communication problems that concern Marie more than Tony. After several sessions, the social worker comes to the conclusion that Tony has an unacknowledged alcohol problem. After coming to this understanding, the social worker should A. meet with Tony privately, confront him with the problem and offer him a referral for treatment. B. meet with Mary privately and form an alliance to address Tony's drinking. C. say nothing about the alcohol problem since this was not the reason they came into treatment. D. encourage discussion of Tony's alcohol problem at the first appropriate moment during a joint counseling session.

D. encourage discussion of Tony's alcohol problem at the first appropriate moment during a joint counseling session.This is a treatment question that frequently arises. Though the agreement between the clinician and the couple is to work on communication issues, one partner's alcoholism is a serious impediment to the marital relationship that is likely to grow worse over time. The clinician should permit the issue to come to the surface and then seek to motivate the client to seek help.

The diagnostic criterion for substance abuse does not include A. a pattern of pathological use. B. impairment in social and/or occupational functioning due to such use. C. duration of disturbance of at least one month. D. guilt concerning excessive use.

D. guilt concerning excessive use. While guilt over the alcohol or drug may use may be present, it is not essential for the diagnosis. The remaining three responses must be present for a DSM-IV diagnosis.

Michael finds it easy to give up alcohol, and he's done it hundreds of times in the last twenty years. Finally fed up by Michael's alcoholism, Barbara, his wife, has told him she will obtain a divorce unless he stops drinking completely. She wants him to move out of the house until he's totally sober for three months, then she'll consider taking him back. The family social work counselor should A. urge Barbara to soften her stance and give Michael a chance to shed his dependence of alcohol while at home since a separation will add to the stress and inhibit Michaels' recovery. B. point out to Michael that if he is sincere he can convince Barbara to relent. C. support Barbara's firmness as this may help Michael to fully confront the damage he is doing to himself and his family, and motivate him to change. D. help Michael confront the situation that he now faces and assist him in finding the treatment resources that he needs.

D. help Michael confront the situation that he now faces and assist him in finding the treatment resources that he needs.Barbara's firm stance may actually help Michael and the worker would not interfere in her decision. Since this is a family treatment situation, the best response for the worker is to help Michael as he addresses his denial. The worker would also assist him in sorting out how he can keep his family together and in finding the programs and service he needs.

A director of a social work agency might use the following problem solving tasks to address changing agency policy EXCEPT: A. defining the problem B. evaluating the probable impact of planned changes C. receiving input from those familiar with the policy D. implementation of the agency plan

D. implementation of the agency plan ---this is not a problem solving task

A social worker in a community agency is treating a married woman. She complain that her husband is a compulsive gambler. He owes money to bookies and has received physical threats. Her discussions always emphasize her husband's behavior. The social worker would A. call Gamblers' Anonymous. B. ask that the husband come in to see the social worker. C. refer the wife to a credit agency as a community resource. D. seek to help the woman understand that she is the client and the primary person to be treated.

D. seek to help the woman understand that she is the client and the primary person to be treated.Of all the choices, this is the best one since it recognizes that the woman is the client, not the husband. There may be reasons that the woman has difficulty focusing on herself, but the worker must always be clear about who the client it. The best place to start is with the person in the office.

A social worker that primarily works in his own private practice has been appointed as a board member in a neighbor community health center that provides counseling services. All of the following roles are appropriate as a board member excpet: A. serving as a chairperson of an assigned board committee B. reviewing and helping to interpret data about agency utilization C. helping to establish job descriptions for agency personnel D. serving as a referral source for difficult clients within the agency

D. serving as a referral source for difficult clients within the agency ---- Never serve as a referral source, this is not ethical

If a social service program is evaluated on achieving its objectives and whether outcomes are due to problem interventions, example of A. a formative evaluation B. a productivity evaluation C. an outcome evaluation D. MBO- management by objectives

D. serving as a referral source for difficult clients within the agency---- Never serve as a referral source, this is not ethical

The client's transference reactions usually relate to A. the client's tendency to observe his feeling in other people. B. the tendency of present events to color past memories. C. the client's ability to transfer learned experiences form one situation to another. D. the tendency to distort present perceptions due to transferred inferences from past, primary experiences.

D. the tendency to distort present perceptions due to transferred inferences from past, primary experiences.Answer A would seem similar to projection, while B is overly general. C is not responsive to the question. A transference reaction occurs when a client develops beliefs about the therapist based on their experience with a significant authority figure in the client's past. Transference reactions are often encouraged in insight-based therapies.

The most important contraindication for marital therapy is A. the fragility of one marital partner's defense. B. paranoid reactions by a marital partner. C. one partner's acting out behavior, e.g., infidelity, noncompliance with agreements. D. the therapist's inability to prevent a marital partner from using sessions for aggressive, hostile and destructive purposes against his or her mate.

D. the therapist's inability to prevent a marital partner from using sessions for aggressive, hostile and destructive purposes against his or her mate.It is well understood in couples work that when there is a threat of, or actual violence against one person by a partner, the therapy will be ineffective and will continue to place one person in a dangerous situation.

Name the four classic diagnostic "A's" of schizophrenia: a. Awareness, ambivalence, autism, and associations. b. Agitation, awareness, associations, and autism. c. Affect, anxiety, ambivalence, and awareness. d. Affect, associations, ambivalence, and autism.

D: Affect, associations, ambivalence, and autism.

An early cognitive theorist, who worked directly with Freud, established a theoretical orientation that differed from Freud's in three key features: 1) an individual's personality is best perceived as a whole, rather than as having hierarchical segments or parts; 2) social relationships drive behavior more than sexual motivations; and 3) current beliefs and thoughts play a far greater role in human behavior than is suggested via psychoanalytic theory, which is based largely in the unconscious and in past experiences and beliefs. The name of this theorist is: a. Lawrence Kohlberg. b. Anna Freud. c. Albert Ellis. d. Alfred Adler.

D: Alfred Adler. The sum are greater than the parts! Adlerian theory also includes a biological view, largely absent in Psychoanalytic Theory, recognizing that hormonal changes, physical illness, chemical imbalances, and neurological disorders can dramatically influence capacity and behavior. It is important to note, however, that Alder still locates false beliefs, irrational thoughts, and misconceptions in the unconscious mind.

You have been contacted by a couple to assist them with issues of marital discord. They have been married about six months. The wife presents as vulnerable, tearful, and anxious, and the husband presents as angry and overwhelmed. The wife openly claims that "he has never loved me," and expresses anger that he married her without "the proper feelings." The husband responds that he has "done everything possible" to "prove" his love (to the point of near bankruptcy and jeopardizing his employment with frequent absences), but nothing is sufficient. During the interview, you discover that she has had many short-term relationships in the past, that she has a history of suicide gestures and "fits of rage." Further, she frequently demands a divorce and then begs him to stay, is routinely physically assaultive, etc. The most likely diagnosis is: a. Intermittent explosive disorder. b. Histrionic personality disorder. c. Paranoid personality disorder. d. Borderline personality disorder.

D: Borderline personality disorder. Individuals with this diagnosis will exhibit: frantic efforts to avoid real or imagined abandonment; unstable and intense interpersonal relationships (especially extremes of idealization and devaluation); an unstable sense of self; extreme impulsivity (e.g., spending, sex, drug use, reckless driving, binge eating, etc); recurring suicidal behavior (gestures or threats, or self-mutilating behavior); affective instability due to reactivity of mood; chronic feelings of emptiness; intense anger (e.g., frequent displays of temper, recurrent physical fights); transient, stress-related paranoid ideation; or severe dissociative symptoms. However, as with all Axis II disorders, this diagnosis cannot properly be made during a first contact, but must be substantiated over a course of clinical contacts sufficient to compel the diagnosis to be made (DSM-IV).

Self-Psychology, as postulated by Heinz Kohut, acknowledges that personality is partly formed by social structure. A cohesive self is achieved by incorporating the perceptions and functions of healthy significant others and objects into an internalized self structure through a process called: a. Empathic mirroring. b. Rapprochement. c. Differentiation. d. Transmuting internalization.

D: Transmuting internalization. Empathic mirroring is the process by which the mother demonstrates ("reflects") care and understanding of the child, in turn helping the child to develop a self-identity. Rapprochement is a term from object relations theory, indicating the need for an infant to seek independence while still retaining security. Differentiation is a substage in object relations theory, where an infant begins to look at the outside world, as opposed to the inward focus common to infants younger than five months of age.

A school age boy is being beaten almost daily by his father who has a drug and alcohol problem for which he received no treatment. The boy solves his school conflicts by punching and kicking younger children. This behavior is BEST described as: a. Identification with the abuser b. Hostility toward peers c. Conduct disorder, group type d. Passive-aggressive personality disorder

Rationale: Basic recall of cycle of abuse concepts and dynamics. Option D is not reasonable given the information in the stem and is not a current personality disorder. He may be hostile towards peers (B) but it is because of his identification with the abuser. Option C and D are not indicated and do not BEST explain his behavior. Option A is the BEST descriptor of what is occurring.

The personality disorder that is characterized by a restricted range of emotional experience and expression is: a. Antisocial b. Schizoid c. Histrionic d. Dependent

Rationale: Basic recall of knowledge of personality disorders.

A social worker at a senior health center is working in a government program that provides services to all seniors. What is this approach to the provision of services called? a. Incremental b. Institutionalized c. Preventive d. Residual

Rationale: Basic recall of policy definitions.

In a clinical setting, information gathered from an ecomap is MOST often used to: a. Identify a family's evolutionary patterns b. Assess family strengths and weaknesses c. Assess intrafamilial communication patterns d. Identify systems that are part of a family's life

Rationale: Basic recall of the uses of an ecomap (D). Other tools are used to determine options A, B, and C.

A social worker meets with a group of concerned citizens who want to stop youth drinking and driving. At an initial meeting, it is suggested that if the youth can find jobs then they will not drink. Another suggestion is that the parents should become more involved in the youths' lives. They also want the police department to be more accountable for what happens with the youth. What should the social worker encourage the group to consider FIRST? a. A meeting with the police department to learn about policies and procedures b. Recommending that the licenses of youth who drink and drive be revoked c. A joint meeting with the youth to understand their concerns and issues d. Collection of all the statistics about youth and crime for the town

Rationale: Citizens have several competing suggestions for dealing with this problem and need to make effective plans, but these plans will only be effective if they are responsive to the youths. A joint meeting with the youths FIRST (C) is the only option that would help the group develop a meaningful plan. Collecting all crime statistics (D) is not pertinent to the issue of concerns of youth drinking and driving. Both options A and B are premature.

During an initial therapy session with a social worker, a client indicates that she wants to disclose certain information but is reluctant to do so because it involves others in her family. In response, the social worker should: a. Explain that the social worker is ethically mandated to maintain confidentiality b. Discuss the issue of confidentiality and the factors that would cause disclosure c. Assure the client that she does not have to share any information until she is ready to do so d. Quickly establish rapport with the client to increase her level of trust

Rationale: Client is ready to share but is reluctant about how it might impact others in her family. Discussing confidentiality and factors that would cause disclosure (B) recognizes that there may be times when information needs to be disclosed. This is ethical to do in an initial session and is part of the informed consent process. Option A suggests a blanket statement that does not educate the client about those situations in which disclosure may be necessary. Options C and D are incorrect because both do not address the client concern.

an agency social worker becomes aware that a co-worker frequently consumes alcohol on their lunch break. When this occurs, the co-worker does not appear intoxicated. This individual is responsible for driving foster children to and from specialized services throughout the day. What is the FIRST action the social worker should take? a. Discuss the drinking directly with the co-worker b. Inform the co-worker's supervisor of the behavior c. Continue to observe and document the co-worker's actions d. Consult the state social work regulatory board for direction

Rationale: Code of Ethics specifies the area of responsibility to colleagues and suggests that talking with the co-worker (A) is always done FIRST. The next action would be to inform the supervisor (B) but it would not occur first. Option C potentially places clients at risk, while consulting the regulatory board (D) is a step that would be taken later if necessary.

A social worker has been treating a client for two months in an agency setting and is experiencing intense sexual attraction toward the client. The social worker's BEST response is to: a. Consult with an agency supervisor about the case b. Reveal the attraction to the client c. Transfer the client to another agency worker d. Work toward ending the client's services at the agency

Rationale: Consulting with a supervisor (A) is the BEST response because it assists in helping to understand and address the situation. Options B and D are clearly unethical. Transferring the client (C) may be done later but it is unclear if this action is required at this point and there is not enough information in the stem to conclude this.

A 35 year old individual has recently experienced the death of a parent and the loss of a job, and has increased consumption of alcohol. Which of the following areas should be assessed FIRST by the social worker? a. Financial resource availability b. Potential for suicidal behavior c. Need for grief counseling d. Extent of substance abuse

Rationale: FIRST, the social worker would act to determine the client's safety. Theonly option that addresses the client's safety is (B). Options A, C, and D may beappropriate later, once the safety of the client has been addressed.

A family services agency has a high number of clients who do not keep appointments. The agency's social work administrator conducts a client satisfaction survey, which reveals that clients believe the agency is not welcoming to their children. To address the clients' concerns, the administrator should FIRST: a. Create specialized activities for children b. Evaluate the need to change agency programs c. Obtain consultation to develop an action plan d. Form a committee of clients and staff to identify solutions

Rationale: Forming a committee (D) engages the clients and staff in formulating solutions. FIRST engaging those who are directly involved is an important consideration. After forming the committee, options A, B, or C may follow, but would not be done first.

A doctor notifies a hospital social worker that a child with a chronic illness has not kept any appointments in the past year and must be evaluated four times a year. The doctor states that if the child does not come for the appointments, it will be considered medical neglect by the parents. The social worker learns the mother has a physical disability and lives over an hour away from the medical center. The mother asks if the child can be treated at a local medical center instead. What is the MOST appropriate action for the social worker to take? a. Insist that the child go to at least four medical visits per year b. Explore an alternative treatment schedule with the doctor c. Empathize with the mother about the limitations of her physical disability d. Determine if the child can receive health care services near the home

Rationale: Goal is to find a feasible way for the child to receive medical treatment. Hence A, B, and C are incorrect because none of them would remove the barriers to the child's medical treatment. Finding health care services near the child's home, (D), could potentially remove one of the perceived barriers.

A social worker meets with a 35-year-old American man who is anxious because his wife is pressuring him to move his mother into a nursing home. He prefers that she live with them. The social worker asks the man if he has any siblings and what his birth order is. This information is important to the social worker's understanding of the man's: A. cultural perspective B. support system C. financial options D. family dynamics

Rationale: It is always important for a social worker to understand the cultural context of client situations. Information about siblings and birth order can reveal much about the impact of culture on a client from the broadest perspective (keyA). This information is less likely to provide an understanding of a client's support system (B), financial options (C), or family dynamics (D).

In a routine review of case records at a mental health center, a social work supervisor discovers that case managers routinely refer all Native American clients to either Alcoholics Anonymous or Al-Anon. In discussing this practice with the case managers, the supervisors learns that the referrals are made without obtaining information about the clients' use or abuse of substances. How can the supervisor effectively address this practice? a. Reprimand case managers providing client services based on stereotyping b. Educate the staff about services provided by Alcoholics Anonymous and Al-Anon c. Provide staff training about culturally competent social work practice d. Allow no referrals to be made without approval from the supervisor

Rationale: It is important to provide educational interventions (C) before moving to more restrictive supervisory practices. Options A and D are more restrictive and may be done later, after the provision of base line training first. While staff members may benefit from being educated about AA and Al-Anon (B), it does not address the concern of stereotyping that is occurring.

Privileged communication is BEST defined as: a. Ethical standards that dictate the social worker cannot reveal client information under any circumstances b. Advance directives, such as living wills and enduring powers of attorney, which protect client wishes and decisions c. The privileges granted the client in communication with the social worker d. Legal rights that, under certain circumstances, protect clients from having their communications revealed in court without their permission

Rationale: Item requires recall of the definition for privileged communication as well as reasoning. Privileged communication is a legal concept, therefore D is the correct answer. It is neither an ethical standard (A), nor advanced directive (B). Option C is incorrect because it only states a selected aspect of the definition.

A school social worker working with an adolescent girl is concerned about the girl's weight loss and withdrawal from her peer group. Upon questioning, the girl expresses fear of gaining weight, and states that she exercises three hours a day and has stopped menstruating. What initial diagnosis should the social worker consider? a. Obsessive-compulsive disorder b. Anxiety disorder c. Bulimia nervosa d. Anorexia nervosa

Rationale: Only anorexia nervosa (D) includes all of the conditions described by the client in the stem. Basic recall item.

A rural social worker has recently been assigned to work with a person convicted of child molestation. The social worker is not knowledgeable or skillful enough in the treatment of sex offenders to provide effective services. What should the social worker do in this situation? a. Conduct a review of the research literature to identify an evidencebased approach b. Consult professional colleagues about the situation and follow their recommendations c. Suspend the client's treatment until the social worker receives adequate education and training d. Find and use the supervision of an expert in treating sex offender

Rationale: Option A is not correct because simply being able to identify an evidence-based approach is not sufficient to equip the social worker to use the approach. Option B is incorrect because there is no indication that the worker's professional colleagues have the necessary expert knowledge and skills in this area of practice. Postponing services, (C), is not appropriate. Finding and using supervision of an expert (D) is an appropriate resource that might be available to a rural social worker.

A social worker can BEST enhance a family's ability to understand each other's views by requesting that they: a. Clarify each family member's statements b. Identify weaknesses in the other family member's arguments c. Focus on the identified client d. Remain silent while the other family members are talking

Rationale: Option C suggests scapegoating one identified family member. Additionally, there is no indication that there is one identified client. Option B would invite additional conflict and is not the best option. Option D encourages listening skills, but this is no guarantee that members will comprehend what is being stated. The BEST answer is A, as it requires the social worker to restate the family's viewpoints in an attempt to help everyone understand their meanings.

.Which of the following activities is LEAST effective for the social worker in engaging clients in the beginning phase of treatment? a. Considering options for problem resolution with the client b. Exploring the client's perceptions of the problem c. Reducing the client's anxiety by providing support d. Clarifying the services that can be offered to the client

Rationale: Options B, C, and D are all aspects of the first steps of engagement and discussion of the services the social worker provides. Option A occurs later in the problem solving process, making it the LEAST effective activity in engagement.

A social worker in a youth recreation program conducts an initial interview with a teenager. The social worker concludes that the presenting problems involve the teen's need to get a job to meet personal needs. What should the social worker do NEXT? a. Contact the teenager's parents b. Consult with the supervisor c. Discuss planning with the client d. Provide needed assistance

Rationale: Planning should be discussed with the client NEXT (C). Options described in A and B may or may not be done later, option D does not address the presenting problems.

A nursing home social worker has a client who is elderly and mentally alert. The client declines a medical procedure that most likely will prolong life. An adult child tries to persuade the social worker to change the client's mind about the decision. The social worker should: a. Respect the client's capacity to make the decision b. Explain to the client the treatment choice that has been made c. Allow the client's family member to make an appropriate choice d. Challenge the adult child's decision only if it seems to violate ethical principles

Rationale: Question addresses the role of self-determination in social work practice. In the absence of a legally appointed guardian, the client's wishes must be respected (A).

A client begins seeing a social worker at the encouragement of family members. Family members are concerned about the client's well-being after losing a long-term job seven months ago. The client reports being frequently irritable and unable to focus on tasks. The client has also been experiencing nausea, excessive perspiration, and insomnia. The client is MOST likely experiencing which disorder? a. Posttraumatic stress b. Generalized anxiety c. Acute stress d. Dysthymia

Rationale: Requires knowledge of how various disorders are defined. Generalized anxiety (B) reflects the client's behaviors, given the history provided. Options A, C, and D can be ruled out for various reasons - posttraumatic stress (A) is a response to an event beyond normal experience; acute stress (C) refers to an immediate response to a grave threat; symptoms associated with dysthymic disorder (D) must be present for longer than two years.

Research findings from a study of cancer patients in one hospice agency can be generalized to: a. All hospice patients b. All cancer patients c. The general population d. The population studied

Rationale: Research findings from an isolated study of one population in one agency can only be extrapolated to that particular population (D). The variability of both the particular group being studied and the setting for the study makes generalization beyond the target population reliable.

A social work administrator is responsible for record keeping at a large agency. Which approach is MOST appropriate for maintaining client records? a. Maintain all records for seven years following termination b. Keep records indefinitely to assure their availability c. Seek permission from clients for destruction of their records d. Store records for the number of years required by law

Rationale: Social workers are required to follow the laws of the jurisdictions in which they practice. Such statutes vary in their requirements. Hence, the only option that is correct is to maintain files for the length of time required by law (D). Options A, B, and C might not apply in all locations.

A social worker meets with a 75-year old client whose wife died six months ago. In the Intake interview, the client reports losing weight, having very little energy, and having difficulty getting motivated to do things. What should the social worker do FIRST? A. Complete a suicide risk assessment of the client. B. Explore available family and community resources C. Suggest that the client schedule an appointment with a doctor. D. Refer the client to a local bereavement group.

Rationale: The client's self-report describes depression and suicide risk that warrant further assessment (Key A). Option (B), (C), and (D) are all plausible but not the FIRST action the social worker should take.

A client with whom a social worker has worked for several years suddenly dies. The social worker is very distressed by the death. During supervision, the supervisor encourages expression and ventilation of feelings. The supervisor is providing: a. Education b. Consultation c. Therapy d. Support

Rationale: The correct answer is (D) because ventilation is one of the techniques used in providing support.

Client feedback that indicates a social worker's interpretation is on target is BEST displayed when the client: a. Argues with the social worker b. Changes the subject c. Expresses anger at the social worker d. Discusses related material

Rationale: The effectiveness of a response can be assessed through client reaction. Continuing to stay on target in the discussion (D) indicates a good progression.

A social worker is working with a young child who is a victim of sexual abuse. The social worker has been using play therapy techniques for several sessions, but the child is becoming more aggressive and continue to exhibit disturbing behaviors such as bed-wetting and masturbation. The child's foster parents feel that the child is not making progress in therapy and have asked the foster care worker to assign a new social worker. What should the social worker do FIRST? A. Tell the foster care worker that the child is making progress as quickly as can be expected B. Transfer the child to someone who specialized in child cognitive-behavioral therapy. C. Encourage the child to talk freely about feelings and the details of the sexual abuse. D. Educate the foster parents about the effects of child sexual abuse.

Rationale: The social worker should FIRST help the foster parents understand the long-term impacts of childhood sexual abuse and the unpredictable treatment progress (D). Option (A) does not provide enough meaningful information to the foster care worker. There is no need to transfer the child (B) because play therapy is indicated for this age group. Encourage a young child to talk to about the abuse (C) is often counterproductive and contraindicated.

Over a period of several months a social worker thoroughly prepares a client for joining a group, including establishment of a contract for three months of participation. After one group session, the client informs the social worker that she will not return to the group. What should the social worker do FIRST? a. Remind the client of her commitment to three months of group participation b. Terminate the client from all services c. Discuss the client's failure to integrate with the group d. Encourage the client to discuss her reasons for leaving

Rationale: Unscored question. The FIRST thing that a social worker should do when a client decides to discontinue therapy is to try to determine the reasons (D). Options A and C seem to blame the client and may do more harm than good. Option B is an extreme reaction as it terminates the client from all services, not just group.

A school social worker meets with a teenager who is increasingly frustrated and disengaged in school. The student appears to be bright, but unmotivated, and does not submit completed homework assignments. Meetings with the student have not resulted in changes in the teen's behaviors. What should the social worker do NEXT? a. Consider an alternative educational program for the student b. Arrange for individual tutoring sessions for the teen c. Refer the teen for evaluation by an educational psychologist d. Create a behavioral self-monitoring plan with the student

Rationale: Unscored question. The fact that the student is noted as bright, but unmotivated, may indicate a learning problem. Of the choices listed, (C), is the correct answer because the social worker has not been successful in helping the teen, and the problem may be more than behavioral (D).

.An adoption social worker is involved in permanency planning for siblings and wants to place them in the same home. One of the siblings is receiving therapy and has severe behavioral problems that may make adoption difficult. In considering whether the siblings can be placed together, what should the social worker do FIRST? a. Plan for long-term placement based on the children's preference b. Request a recommendation from the child's psychologist about placement risks c. Allow the children to remain in foster care until they can be adopted together d. Make the decision to separate the siblings permanently to facilitate adoption

Rationale: Unscored question. This question is asking for the FIRST thing to be done in this situation. Obtaining a recommendation from the child's psychologist about placement risks (B) will allow the social worker information to consider in taking appropriate action.

At an intake session, a client requests medication for sleep. Since becoming unemployed the client has used alcohol as a sedative. The client is worried about finances and is afraid of being evicted. The social worker should FIRST: a. Refer the client for evaluation for sleep medication b. Refer the client to public assistance c. Obtain more information about the client's use of alcohol d. Refer the client to the local unemployment office

Rationale: While options A, B, and D may all be appropriate actions for the social worker to take, none would be the first. As part of the assessment process, the social worker needs to FIRST further explore the issue of alcohol use (C).

Tourette syndrome is not the direct physiological effects of a substance or a medical condition. The age of onset is usually between 7 and 10 years of age, with some children showing signs at an earlier age. Research indicates that 200,000 Americans have the most severe form of Tourette syndrome and one in one hundred have a milder version. What are the characterizations of this disorder? Voluntarily hitting other children for attention Repetitive, involuntary movements or vocalizations Cruelty to animals and aggressive behavior Depression, sadness and low self-esteem

Repetitive, involuntary movements or vocalizations

A social worker meets with a 32-year-old woman who's wife encouraged her to seek treatment. The client shares that she was recently laid off from work due to budget cuts and feels uncertain about her ability to find employment again anytime soon. She expresses concern about being a financial burden to her wife, who she's always shared expenses with. The client shares she is struggling to find purpose in her day-to-day activities while out of work and is starting to feel hopeless about the future. What areas should the social worker assess FIRST: A. Risk of harm to self and job history B. Risk of harm to self and medical concerns C.Job history and social supports D. Job history and coping mechanisms

The correct answer is A: to assess risk of harm to self and job history. Because the client expresses hopelessness, we need to explore any risk of harm to self. The client was laid off and expresses concern about her job prospects, so we also want to explore her job history to gather information that may be leading to her feeling this way. B is incorrect because no medical concerns or symptoms are discussed in the question stem. C and D are incorrect because, while important, they do not address potential risk of harm to self which must take priority when present.

A social worker meets with a client at a domestic violence shelter who has spent the past week at the shelter with her two young children. During the session, the client shares that her spouse has probably cooled down by now and that she plans to return home that day. While safety planning, the client assures the social worker that the kids are not being impacted by this because they are never in the same room where the violence occurs. What should the social worker do? A. Accept the client's decision. B. Inform the client that a CPS report needs to be made. C. Provide psychoeducation on the impact of domestic violence on children. D. Encourage the client to stay another night at the shelter. (scroll for answer and rationale).

The correct answer is C: to provide psychoeducation on the impact of domestic violence on children. Even when children are not physically impacted by domestic violence, they are still impacted. Sharing information on this is important, given that the client has indicated they are not impacted by the domestic violence because they aren't in the same room. This suggests the client is missing information about the impact of domestic violence on children. A is incorrect because while we may ultimately do this, it isn't the NEXT thing we would do. It ignores what the client has just said about the kids not being impacted. B is incorrect because on this national exam, we don't report domestic violence unless the children are being physically harmed (or neglected). There is no reason to do D at this time.

A competent college student has been waking very early, has lost interest in eating and socializing, and is not certain about her continued ability to handle her studies. These symptoms MOST likely indicate: A depressive disorder B anxiety disorder C anorexia nervosa D insomnia

a

A pregnant 17yearold has been referred to a school social worker. The client is upset, confused, and uncertain about her options. After several appointments with the client, the social worker recognizes that her own values may be interfering with her relationship with the client. The social worker's MOST appropriate action is to: A discuss the value dilemma with her supervisor B refer the client to another social worker C refer the client to a family planning agency for services D arrange a family therapy session to discuss the pregnancy

a

Which of the following treatment paradigms uses the technique of "time out"? A Social learning B Psychodynamic C Humanistic D Systems

a

In comparing models of social work clinical practice, probably the most significant difference is related to A. contracting. B. interpretation. C. clarification. D. confrontation.

a . contracting. The contract is theory-based and will be different depending on what is understood as the issue or focus. The other responses are tools for working with clients and are similar in all theory-based practice.

Encopresis is defined as: a. The voluntary or involuntary passage of stool in an inappropriate place by a child over the age of four. b. The voluntary or involuntary passage of stool in an inappropriate place by a competent adult. c. Deliberate fecal incontinence only in a child over age four. d. Involuntary fecal incontinence only in a developmentally delayed adult.

a. The voluntary or involuntary passage of stool in an inappropriate place by a child over the age of four. Most cases of encopresis are the result of chronic constipation

There has been a public emergency in a small town. First responders are called out to care for the injured. The social work administrator in a neighboring town calls his team of social workers in from their weekend off. What is the administrator's duty? a. To ensure his staff provides professional services to the greatest extent possible b. Place the social workers on call in case their services are needed c. Prepare the agency for a rush of welfare applications d. Contact local service agencies to let them know the team is not available

a. To ensure his staff provides professional services to the greatest extent possible Administrators should secure proper training for social workers to respond in a case of emergency. Professional services need to be provided to the greatest extent possible. Victims may require help with trauma, loss of housing, or loss of a loved one.

While many variables affect suicidality among teenagers, which item below has been a particularly powerful predictor: a. a recent parental divorce b. a transfer to a new school c. a history of occasional drug use d. a change in family economic status

a. a recent parental divorce Family dynamics and the stability of internal structure are the highest predictors of suicidality.

A schizophrenic client takes prescribed medications intermittently. After an argument, his partner kicks him out of their apartment and asks the social worker to hospitalize him because his condition is deteriorating. The client sometimes experiences hallucinations, but says he wants to try to live on his own. He refuses hospitalization and says he plans to stop taking his meds. What is the best course of action: a. acknowledge the right to refuse medication and treatment and seek to maintain a relationship with the client b. seek an involuntary commitment c. continue to insist that the client take the medication and note that previous episodes of not taking meds have led to hospitalization d. suggest that his partner care for him for a short period until he is more stable

a. acknowledge the right to refuse medication and treatment and seek to maintain a relationship with the client

The view that learning takes place through observation and that new responses are acquired through the process of imitating models is associated with (a) cognitive behaviorism (b) operant conditioning Brooks 2006 4 (c) social learning (d) classic conditioning

c

When conducting an assessment of the risk factors of child abuse for a child of a different ethnic background, a social worker must FIRST: a. be aware of how personal cultural biases affect the social worker's ability to deal with issues of diversity b. realize that assessment models are not a substitute for clinical judgment or experience c. use a strengths' perspective rather than a deficit model to form a partnership with the child's family d. understand the need for appropriate eye contact, tone of voice and question techniques during the assessment

a. be aware of how personal cultural biases affect the social worker's ability to deal with issues of diversity

In preparing for an interview with a client who is receiving ongoing services, the MOST important thing for the social worker to do is to: a. clearly identifying the purpose of the upcoming interview b. review previous case notes in the client's folder c. determine if there are new agency policies that pply to this client d. mentally review progress the client that has made toward goal attainment

a. clearly identify the purpose of the upcoming interview

A social worker facilitates a group of young adults. One member rudely disagrees with the social worker's comments. The social worker is annoyed by this behavior. His best response is to: a. communicate the feelings of annoyance b. speak to the person after the group c. allow the group to respond d. do nothing until it happens again

a. communicate the feelings of annoyance

A social worker works with a clientele with varied substance and chemical dependencies and decides to establish a new group focused on drug and alcohol rehabilitation. In the first stages of the group, the worker might expect: a. denial, rationalization, projection b. low self worth, acceptance, projection c. surrender, delusions, rationalization d. denial, rationalization, compliance

a. denial, rationalization, projection A is correct because it is most likely to see these traits during the early process of sobriety. B is wrong because these traits are likely to be seen in later steps of rehab - it takes time for acceptance to occur. Read carefully! C is wrong because "delusions" are not associated with rehab -delusions are a sign of psychosis!! If psychosis is present, social workers must refer to a psych institute prior to working on other goals. D is wrong because "compliance" is not seen in the first stage of rehab.

The social work supervisor, in reviewing a supervisee's recorded case file makes this comment about the case recording method being used: "This type of case recoding stresses the individual social worker's interpretation and tends to be very long and not uniformed in its presentation; that is probably why it is rarely used today". What recording method was used? a. diagnostic summary recording b. problem oriented recording c. SOAP note d. current process recording

a. diagnostic summary recording

The social worker supervisor, in reviewing a supervisee's recorded case file makes this comment about the case recording method being used: "This type of case recording stresses the individual social worker's interpretation and tends to be very long and not uniformed in its presentation: that is probably why is=t is rarely used today". What recording method was used? a. diagnostic summary recording b. problem oriented recording c. SOAP note d. current process recording

a. diagnostic summary recording

a social worker discovers that a nurse is giving more pain medication to a patient than the doctor has prescribed. What should the social worker do? a. discuss the issue with the nurse b. report the nurse immediately to the medical board c. immediately tell the nurse's supervisor d. tell the doctor

a. discuss the issue with the nurse

A 17 year old boy worries about undressing in the locker room at school. The boy has no genital or underarm hair and believes his genital s are underdeveloped. The adolescent is referred to the school social worker . The social worker would first a. encourage the youth not to be ashamed b. suggest the youth see a physician who specializes in this issue c. refer the youth and his family to family therapy d. discuss the issue with the physical education teacher

a. encourage the youth not to be ashamed

You decide that one of your clients could be best helped by a treatment intervention that is outside your scope of practice. What would you do? a. end treatment with the client b. seek consultation c. tell the client you are not familiar with the needed intervention d. get training in this intervention

a. end treatment with the client

The primary purpose of a service plan is to a. establish goals b. assess the home situation c. monitor behavioral changes d. evaluate client progress

a. establish goals

A generalist social worker is most concerned about: a. focusing on the hierarchy of client needs and the way they interact with the client's problems b. her/his relationship with the client c. the client's emotional well-being d. the way a person's spirit is connected to the client's physical well-being

a. focusing on the hierarchy of client needs and the way they interact with the client's problems

A social worker is interested in seeing the extent to which current clients are satisfied with a new relapse prevention program. The social worker distributes a client satisfaction survey to those in the program. The social worker then collects the surveys and analyzes the results that are presented to a management team in the agency. The social worker is conducting which type of evaluation? a. formative b. summative c. periodic d. exploratory

a. formative

A single mother with 3 children is unemployed, has no support or family who live in the area, no services for the children, and no resources for herself. The mother states she would like a support network. What type of services would you provide for her? a. respite services b. referral to unemployment c. referral to a support group d. referral to parenting classes

a. respite services

In the initial session with a social worker, parents of a 6-year-old boy describe their child's behavior. The parents state that the child startles easily, and he cries excessively when he has to leave them. This behavior intensifies when he has to leave them and go to school. He also spends a great deal of his playtime building and later destroying houses that he makes with wooden blocks. From the choices below, which provides the best preliminary assessment toward understanding the child's behavior? a. separation anxiety b. stranger anxiety c. child abuse d. anxiety disorder

a. separation anxiety-----stranger anxiety usually is in infants, the other deciding factor here is him crying with the most when the parents are trying to leave him

A single man, aged thirty, comes to an outpatient medical clinic for a physical checkup to decide whether he can work. He has a shattered index finger on his left hand because of an injury. The surgeon recommends an operation on the damaged finger and refers him to social services for help in planning for the operation. The social worker's first step in dealing with the patient is to A. refer the patient to the psychiatrist for evaluation of his readiness for surgery. B. establish a relationship with the patient to find out what surgery means to him. C. investigate community resources about helping the patient with his surgical plan. D. help the patient handle his anxiety about the operation by describing his procedure.

b

A social worker makes home visits to a young mother with her first baby. On the first visit, the four-month-old baby smiles, laughs, and responds to the worker's approaches. During a visit when the child is eight months old the baby turns away when the worker approaches, looks distressed, and begins to cry. The social worker should interpret this change of behavior as MOST likely due to: A conditioned fear response B developmentally normal stranger anxiety C child abuse D normal temperamental irritability

b

An elderly client is blind and confined to bed. Assistance with daily living skills by a neighbor has allowed the client to remain at home. However, the social worker is suspicious that the neighbor is taking the client's money and providing less than adequate care. The client wishes to remain at home, continuing the arrangement with the neighbor. The home health staff is concerned that interference in the situation will damage rapport with the client. The social worker's FIRST responsibility is to: A discuss the situation with the agency's lawyer B report to adult protective services C confront the neighbor about the suspicions D work with the client to accept alternatives

b

Which of the following assumptions is not accepted by a social worker engaged in family treatment (a) each person strives for a sense of relatedness and closeness with others (b) treating the scapegoated family member will help the rest of the family (c) family members explain their own sense of worth on the basis of the behavior of others (d) family boundaries are the means for individuals protect differences and maintain identity

b

A clinic forms a treatment group session of young adults. 4 men and 4 women. All 8 clients are present for the first group session. The social worker introduces himself and the members introduce themselves. Then everyone looks expectantly at the social worker. There is silence. The social worker's most appropriate response is to a. comment that they are clearly expecting her to start things b. ask everyone to say something about the problems that have brought them to this group c. not break the silence; wait for the group to start d. Ask, "where would you like to start"

b. ask everyone to stay something about the problems that have brought them to this group.

Mrs. Lopez, age 76 lives with her adult daughter. She was recently released from the hospital after hip replacement surgery. She refuses to enter a nursing home although her daughter works and cannot care for her full time. What is the most important task of the medical social worker at this time. a. give the family phone numbers for community resources and tell them to make their own arrangements to assure self-determination b. assess the strengths of the family for coping with a disabled parent and the potential goodness of fit with home care and visiting nurse agencies c. insist that Mrs. Lopez enter a nursing home d. ask the doctor to insist that Mrs. Lopez enter a nursing home.

b. assess the strengths of the family for coping with a disabled parent and the potential goodness of fit with home care and visiting nurse agencies.

The social worker suspects that a client on her unit is having sexual intercourse with another client on her unit who is HIV positive. It is against agency policy to disclose information to clients on the unit about other clients on the unit. What should the social worker do? a. tell the client that the individual he is having sexual intercourse with is HIV positive b. be sure the client is aware of HIV and the risk of having relations with someone who is HIV positive c. tell the agency administrator about the situation d. violate agency policy to protect your client

b. be sure the client is aware of HIV and the risk of having relations with someone who is HIV positive

A social worker maintains client records in a safe environment and only releases information as provided by law. The intent of these procedures is to protect a. the client's right of due process b. client confidentiality c. agency record requirements d. self-determination by the client

b. client confidentiality

A social worker is working in a community-based agency. She wats to help start a new program in the community. The first step she should take to start the process is: a. contact residents to gain interests b. complete a needs assessment c. speak with her supervisor about implementation d. post flyers and set up a community meeting

b. complete a needs assessment

During an intake interview, a client reports that she is extremely depressed and has self-destructive thoughts. She has had prior suicide attempts, but tells the social worker not to worry as she won't "do it again." The social worker should FIRST: a. create goals with the client b. conduct a safety assessment c. call 911 d. begin therapeutic interventions

b. conduct a safety assessment

A client discusses with the social worker a fight she had with her husband . The client explains how angry she was with her husband but the husband abruptly left the house and ended the fight. The client reported she was still furious with him. Since she had to hurry and get her daughter ready for school she wildly brushed the child's hair until the child complained it hurts. the mother is expressing her anger with the father while combing the child's hair roughly. This is an example of: a. sublimation b. displacement c. reaction formation d projection

b. displacement-- mother puts anger into the daughter's hair rather than husband

A fifteen year old girl has been sent home from school on three occasions for drinking alcohol at school. Each time, the mother becomes angry and threatens to punish the girl but is convinced by the father to give her another chance. Which of the following BEST characterizes the father's behavior in this situation? a. triangulation b. enabling c. advocating d. manipulating

b. enabling

A social worker has been asked to assist an elderly client in making alternative living arrangements. In the initial interview, the client repeatedly attempts to discuss past experiences. What is the social worker's MOST appropriate response to the client? a. ignore the references to the past b. facilitate discussion of the recollections c. refer the client for psychiatric evaluation d. administer a geriatric evaluation scale

b. facilitate discussion of the recollections

The positive cultural value that is expressed in the ideals of "machismo" is: a. masculinity b. family provider and protector c. power and control d. cooperation

b. family provider and protector

J. a 40-year-old male reports feeling depressed. Which is least likely to be a behavioral manifestation associated with his depression? a. decreased productivity and fatigue at work b. fear or belief in having a serious illness c. difficulty falling or staying asleep d. appetite changes resulting in weight loss

b. fear or belief in having a serious illness

A female client is having difficulty eating and sleeping. She cannot complete her work because she is plagued by constant worrying. What is the most likely diagnosis? a. bipolar disorder b. generalized anxiety disorder c. major depressive disorder d. adjustment disorder

b. generalized anxiety disorder

Which substance is the result in the user having blood shot eyes a. heroin b. marijuana c. cocaine. d. alcohol

b. marijuana

In a multi-disciplinary organization the best way for a supervisor to obtain desired outcomes is to: a. review salaries after establishing criteria for doing so b. meet with individual employees to establish rapport c. establish a motivational incentive system d. clearly establish goals and organizational objectives

b. meet with individual employees to establish rapport ---everyone is from a different discipline with a different perspective

In a multi-disciplinary organization the best way for a supervisor to obtain desired outcomes is to: a. review salaries after establishing criteria for doing so b. meet with individual employees to establish rapport c. establish a motivational incentive system d. clearly establish goals and organizational objectives

b. meet with individual employees to establish rapport---everyone is from a different discipline with a different perspective

Empirical research on diagnostic patterns has historically demonstrated that African Americans with the same symptom presentation as Caucasians will often be diagnosed: a. less severely b. more severely c. the same d. the research is irrelevant

b. more severely

A client in a state of crisis is most likely to evidence a. reliance on prior patterns of coping b. preoccupation with the precipitating stress c. resistance to change d. reflection on early experiences of stress

b. preoccupation with the precipitating stress

A recent social work graduate displays excellent relationship skills and a strong commitment to clients. The worker knows she has difficulty in translating her insights and goals into specific treatment plans. The worker might ask the supervisor to: a. point out problems and correct her when she loses sight of the goals b. provide concrete support and detailed help in areas that require improvement c. pair her with a more experienced worker d. provide written guidelines on good practice

b. provide concrete support and detailed help in areas that require improvement

A social worker consistently exceeds job standards. To help the social worker maintain the level of performance, the supervisor should: a. put a letter of commendation in the social worker's personal file b. provide the social worker with feedback on an ongoing basis c. arrange a merit increase for the social worker d. recommend the social worker for promotion

b. provide the social worker with feedback on an ongoing basis

In a clinical setting the most likely statistically significant program evaluation design would be: a. experimental b. quasi experimental c. single subject d. type 2

b. quasi experimental Clinical settings are likely to use ABAB designs for program evaluation because it is most convenient. It allows SWs to use data from their clients without having a separate control group; therefore, it is less time consuming, cost effective, and most accessible.

A child, who no longer characterized objects as all good or all bad, appears to demand an increased amount of attention from the primary character is: a. object constancy b. rapprochement c. individuation d. symbioses

b. rapprochement-this is a sub-phase within Mahler's stages of human growth and development. Exactly when the concept of rapprochement occurs is highly debatable but generally is considered to occur at approximately age 5 or

In the initial phase of treatment, the primary task of the social worker is to: a. establish a warm relationship with the client b. understand why the client is seeking help c. engage the client in treatment d. gather information for psychological diagnosis

b. understand why the client is seeking help

A social worker in private practice has a first session with a teenager who was referred for recent poor school performance. He is withdrawing from friends and family and has been found hiding under his bed. The parents have found marijuana in his room. He told the social worker in the first session that he has been hearing voices. What should the social worker do first? a. recommend substance abuse treatment b. refer the teenager for a substance evaluation c. refer the teenager to a psychiatrist for a mental health assessment d. assess the teenager for depression

b. refer the teenager for a substance evaluation ----these sound like drug related behaviors, so an evaluation would need to be first, before the referral for treatment

You have been seeing an elderly couple for one month. The 70 year old husband recently had a stroke. The wife tells you that sometimes he hits her. She rolls up her sleeve and shows you large bruises. You should: a. consider that the stroke caused his violence b. report elder abuse c. refer the couple to a doctor before reporting abuse d. call adult protective services and ask if you should file a report

b. report elder abuse Duty to protect the vulnerable. ALWAYS act when there are signs of abuse.

A family has just relocated from a homeless shelter to permanent housing. The mother is in recovery from alcohol abuse. The father has been violent with the mother and is unemployed. The 8 year old changed schools 6 times in the past year. The family's records describe the family's problems and note the "contract" to which the parents have agreed and the services that will be provided. The social worker in the permanent housing program's first task is to: a. assure that the mother continues in an AA program b. review and prioritize the family's problems and reinforce the family's commitment to addressing them c. help the father find a job d. accompany the parents to school with the child

b. review and prioritize the family's problems and reinforce the family's commitment to addressing them

A social worker is making little progress with a client after 4 weeks of treatment. What might the social worker do? a. refer the client to another social worker b. schedule a case consultation c. continue with treatment, it's too early for progress to occur d. confront the client about their lack of motivation

b. schedule a case consultation Case consultations provide a forum for learning about new resources, the criteria and practice of different programs, increasing skills in risk assessment, and making informed decisions within a team

a 30 year-old divorced male tells his social worker that he is going to commit suicide and clearly outlines a concrete plan to complete this. What should the social worker do first? a. complete a no-harm/no-risk agreement b. seek hospitalization c. complete an assessment d. maintain confidentiality

b. seek hospitalization----if ideation and intent can be clearly assessed, seek admission first

The research design most often used by social workers to improve their practice is: a. experimental design b. single-subject design c. longitudinal design d. quasi-experimental design

b. single subject design

When interviewing a client whose speech patterns are different from the social worker's, the social worker's PRIMARY consideration should be that: a. communication patterns vary due to economic conditions b. speech is influenced by culture and experience c. psychosocial conditions will cause variations in speech d. speech is influenced by group taboos and norms and may indicate false beliefs

b. speech is influenced by culture and experience

The policy of Permanency Planning refers to a. children who are at risk of removal or are removed from their own homes b. the development of a plan through which abused and neglected children will be assured of a stable family situation throughout childhood c. a stable foster care plan for children removed from their homes d. the use of adoption for children at risk

b. the development of a plan through which abused and neglected children will be assured of a stable family situation throughout childhood

A client (4 year old male) presents for treatment with his mother (24) and father (30). The child is having temper outbursts and although previously potty-trained is wetting himself during the day while he is playing with the other children or watching television. When asked why he does not go to the bathroom, the child has no reply. The parents state that the child's behavior is causing them to argue over the proper way to handle the situation and several other relationship issues. The worker suggests a "time out" to handle the temper outbursts. To complete this method the worker would most likely suggest that: a. the parents agree not to argue in front of the child regardless of how bad the tantrums are b. the parents remove the child from what he is busy doing after each outburst, place him in a chair for no more than 4 minutes , explaining concretely why he is being placed there c. the parents punish the child immediately by scolding him and send him to his room d. the parents tell the child they will isolate him from other children unless the behavior is modified.

b. the parents remove the child from what he is busy doing after each outburst, place him in a chair for no more than 4 minutes , explaining concretely why he is being placed there

An American Indian couple was referred to therapy because of the male partner's drinking. Throughout the session, the couple seem to be responding but never looked directly at the social worker. The social worker correctly interprets this as: a. they were no cooperating b. they were showing respect to the worker c. they didn't want to admit there was a problem d. they didn't want to discuss the situation

b. they were showing respect to the worker---May feel the matter should be discussed in a tribal group setting instead of with an outsider

The most significant clinical purpose for maintaining social work records in a family agency is: a. to facilitate worker accountability for treatment quality b. to assess client progress continually and to revise and update treatment planning c. to provide documentation that protects the agency and the worker from malpractice suits d. to provide continuity of care and to simplify transfer if the social worker leaves

b. to assess client progress continually and to revise and update treatment planning While the other three are not wrong, B is the most client focused. Read the question carefully!

In the social work/client relationship, acceptance is defined as: a. a neutral stance b. unconditional positive regard c. agreement with client's values d. an expression of approval

b. unconditional positive regard

A Native American Indian woman has decided that she wishes to give up her child for adoption. She feels that she can no longer handle the child and that she herself needs drug rehabilitation. She was a member of a Native American tribe but is no longer living near her reservation, nor does she want contact with them or her family. The first action for the social worker to take in this case is: a. Help the woman find immediate placement for the child b.present the case to the Native American tribal government for action c. find a concerned friend or family to take the child d. get the client into inpatient treatment for her substance problem and arrange temporary placement with the child with a local agency

b.present the case to the Native American tribal government for action -- always go to tribe first

A 45-year-old client has made frequent references to death and violence. The MOST accurate indicator of impending harm to self or others is: A the client's continuous catharsis of extreme hostility B a history of violent behavior C expression of a preferred method or plan D excessive crying

c

A DSM-IV-TR diagnosis of major depression, single episode, is indicated by all of the following symptoms EXCEPT: A flat affect B feelings of hopelessness C hallucination and hearing voices D suicidal gestures

c

A social history with a 52-year-old female admitted following a suicide attempt reveals that she had been sexually abused for six months by a relative when she was 12. In completing the social history, the social worker should NEXT: A take a complete sexual history B obtain confirmation of this abuse from other family members C determine how the abuse contributes to the client's presenting problem D acknowledge but give little weight to the information since it is not directly related to the immediate problem

c

A social worker is consulting with the parents of a young adult who has been recently diagnosed as having bipolar disorder. The social worker should FIRST: A obtain a family history of the client B identify programs available for the client C provide information about the disorder D develop ways to maintain the client's independence

c

A social worker is emotionally drawn to a client and begins to think about the possibility of dating the client. The first thing the social worker should do is: a. terminate with the client b. be honest and tell the client c. process these thoughts d. tell her supervisor

c

A tenyearold child is referred to a school social worker for irritability, poor concentration, and low selfesteem experienced during the past year. Which diagnosis would be appropriate? A Oppositional defiant disorder B Communication disorder C Dysthymic disorder D Separation anxiety disorder

c

An 18-year-old single mother has a 20-month-old child placed in foster care for six months because of neglect. The mother has followed her reunification plan, and the foster care plan is to return the child to the home. The foster care social worker learns two weeks before the hearing that the mother is several months pregnant. The worker's BEST option is to request that the: A mother regain legal and physical custody and close the case B child remain in foster care until after the birth of the baby C physical custody be returned to the mother and the family situation be monitored by the agency for six months D court date be postponed until the baby is born

c

An African American client receives successful treatment for depression from a non-minority social worker and is functioning much better. However, the client advises the social worker that she has other underlying problems the worker cannot assist her with because the therapist is not African American. How does the worker respond? a. refer the client to someone else b. ask her what the underlying problems are c. discuss with the client, why she does not believe the current social worker can help d. tell the client after she discusses the issue, that she will be given the referral

c

In facilitating a hospital support group for bereaved parents, a social worker can discuss a personal experience with the death of a child to: A create empathy with the participants B receive help from the group C create a sense of shared experience D establish a role as expert on grief issues

c

In the hospital, a team of professionals' works together to address the needs of the client that they are serving. Members of the team include a physician, a nurse and a social worker. All professionals on this team share many of their duties and responsibilities for client care and service position. For example, often times the nurse does counseling in regard to family issues, and the social worker monitors medication compliance. The term that BEST describes this type of teamwork is: a. multidisciplinary b. quality assurance c. inter-disciplinary d. cooperative

c

J. is suffering from Schizophrenia . From the list below which would be considered the BEST recommended course of treatment to start the client on in the initial stage? a. substance abuse treatment b. psychoanalysis c. pharmacological treatment d. insight oriented therapy

c

Parents of a 4 yr old are referred to a social worker after a physical exam reveals no physical problem preventing the child from being toilet trained. The parents reveal that the child has not been able to separate from them to attend nursery school and he sleeps with them most nights. The most important area to focus on in the assessment phase is the: (a) parents use of rewards and punishments with the child (b) early developmental and family history of each parent (c) parents understanding of the child's developmental processes (d) ways in which the child interferes with the parents own relationship

c

The 2 most common defenses used by couples with marital problems are (a) repression and denial (b) repression and suppression (c) projection and displacement (d) projection and splitting

c

You have just had your first session with a 24-year-old college student. She is seeing you following the break-up of a two-year relationship, which occurred without warning about six weeks prior to this visit. As she explained it, "He met someone else and just moved on." She has been having trouble sleeping and concentrating on her studies since that time. Today she presents as dysphoric and tearful, but is affectively expressive and responsive to humor and other interactive stimuli. The university she attends is a considerable distance from her family and friends, leaving her with limited support during this difficult time. The most appropriate diagnosis would be: 1. a. Primary insomnia. 2. b. Major depression. 3. c. Adjustment disorder with depressed mood. 4. d. Acute stress disorder.

c. Adjustment disorder with depressed mood. —> struggle with change. Criteria for this disorder includes a time-limited nature, usually beginning within three months of the stressful event, and lessening within six months-either with removal of the stressor or through new adaptation skills. Adjustment disorder is a "sub-threshold disorder," allowing for early classification of a temporary condition when the clinical picture remains vague. While the patient does have insomnia, it arises from the stressful loss and not as an independent condition. Many of the essential criteria for a major depression are absent (weight loss, psychomotor agitation, blunted affect, etc), although without successful treatment this condition could emerge. The diagnosis of acute stress disorder is not appropriate as the precipitating event did not involve threatened or actual serious injury or death.

A couple expressing extreme anger at each other come to a social worker for marital counseling. The social worker should FIRST: a. explore the goals the anger is masking b. clarify whether each partner is willing to hear the other c. acknowledge the anger each partner has towards the other d. use the interview to probe for positive feelings each has for the other

c. acknowledge the anger each partner has towards the other

A family is seen in treatment, consisting of a father (age 45, a long distance truck driver), a mother (age 43, housewife), a son (age 22, living at home recovering from alcohol and drug problem), and a daughter (age 15, recently failing in school). The mother has made the appointment because the daughter is refusing to try to pass her subjects in school, and the usual methods of discipline and motivation are not working. From a family therapy perspective the son's drug and alcohol addiction problem is related to: (a) the mother's over-concern for the children b. is providing a crucial role model for the daughter c. another way in which the family system maintains homeostasis d. is the actual problem that needs to be addressed

c. another way in which the family system maintains homeostasis

A child has a severe learning disability and is unable to speak. The child is being placed in a foster home. He was living with his parents who have now been placed in jail pending criminal charges. In order to facilitate temporary placement for this child the first step is the assessment process is: a. determine past behaviors competed in his previous home environment b. determine what tasks he was able to perform for himself in his previous foster home c. assess the current situation by examining how the child responds to verbal and non-verbal cues d. place the child with a neighbor familiar with his condition

c. assess the current situation by examining how the child responds to verbal and non-verbal cues

Which of the following drugs is a stimulant? a. Heroin B. PCP c. caffeine d. sedatives

c. caffeine

A 26 yr/old female client shares with her male social worker that she is becoming attached to him and often has sexual fantasies about him. She also shares that she would like to see him socially. What should be the social worker's first responsibility? a. discuss the client's feelings about her sexual fantasies b. immediatly refer the client to a female social worker c. clarify the professional nature of the social worker-client relationship d. discuss the client's pas relationships.

c. clarify the professional nature of the social worker-client relationship.

Case management is used for the purpose of: a. improving overall client functioning b. providing client support c. coordinating resources d. gaining access to resources

c. coordinating resources

A healthy couple has been referred to you because the woman is unable to have orgasms. The doctor that referred them states there are no obvious medical problems. You would first: a. explain female anatomy b. begin exploring the couple's relationship c. find out what the couple has tried thus far to address the problem d. do a genogram to explore family of origin issues

c. find out what the couple has tried thus far to address the problem RU SAFE question A is wrong because - it is not in our scope of practice - we have not assessed the situation to understand why this is happening B may seem correct initially, but this is making A LOT of assumptions... maybe it is not a relationshional issue... we must assess first. D is wrong because this is not a family origin issue.

An adolescent has been referred to a social worker because of school-related difficulties. During the intake interview with the family, the parents become angry and begin verbally attacking the teen. The social worker's most appropriate action is to a. allow the parents to continue to vent their anger b. stop the interview and see the teen alone c. focus on the parent's expression of anger d. ask the teen to leave the room.

c. focus on the parent's expression of anger

Data collection in the engagement phase of social work practice means that social workers: a. identify clients' financial and insurance resources b. seek out information on any legal or criminal problems c. interview clients and fill out forms during the initial interview d. read documents, records and other written materials

c. interview clients and fill out forms during the initial interview

Data collection in the engagement phase of social work practice means that social workers: a. identify clients' financial and insurance resources b. seek out information on any legal or criminal problems c. interview clients and fill out forms during the initial interview d. read documents, records and other written materials

c. interview clients and fill out forms during the initial interview Engagement is collaborative. Think - engaging with clients to gather information.

Dysfunctional families that try to maintain the appearance of open, reciprocal relationships with each other are best described by the term: a. enmeshment b. mystification c. pseudo-mutuality d. disoriented

c. pseudo-mutuality

When working with a client who repeatedly wets his bed, the worker prepares the parents to implement a behavioral treatment plan that will result in the child reacting differently just because he knows he is being watched. This is an example of: a. the "Rosenthal Effect" b. experimenter expectancy c. reactivity d. stimulus generalization

c. reactivity ----when people know they are being watched they may act differently

The long range focus of community organization techniques is to: a. develop trust b. alleviate anxiety c. reduce dependability d. arouse anger

c. reduce dependability

A 28 year old client with a long standing history of drug use is referred to a social worker by a concerned relative. In the assessment interview, the client tells the social worker about frequent cocaine use. The social worker should first. a. conduct a family interview to evaluate the families concern b. begin treatment, focusing on the reason for drug abuse c. refer the client to a substance abuse program d. evaluate the client's motivation for change

c. refer the client to a substance abuse program

The youngest child of a single parent is hospitalized for the second time in four months because of lead poisoning. The child is now ready for discharge, but the doctor refuses to go ahead with the release, citing an unsafe home environment. When interviewed by the hospital social worker, the parent says that the landlord will no have their apartment repainted or make other necessary repairs. The parent wants to move, but is unable to afford living arrangements that will also allow four children. After reporting the situation to child welfare authorities, the social worker should a. advise the parent to withhold rent until repair work is done. b. advocate for the child to return home with the parent c. refer the parent to a resource that can help locate suitable housing d. report the landlord's lack of cooperation to the housing authority

c. refer the parent to a resource that can help locate suitable housing

a man who lives in your neighborhood phones you expressing concerns about his son's conduct at school. The boy has skipped classes and his grades have declined this year. You and the man have several friends in common and you see him frequently at parties. The man asks if you would see his son for therapy. Your best course of action is to: a. see the son for therapy after you clarify issues of confidentiality with his parents b. see the son once to assess his needs and then refer him to another social worker c. refer the son to another social worker d. agree to see the son for therapy

c. refer the son to another social worker

A man in the army is referred to a social worker for alcohol abuse. He and his wife have been having many arguments about his drinking, which have escalated into physical abuse. What you consider for an intervention? a. assess for alcohol abuse and continue therapy b. refer to a marriage/family therapist c. refer to an alcohol treatment center d. refer to domestic violence classes

c. refer to an alcohol treatment center FARM GRITS ROAD Do not treat alcoholics

A social worker who attempts to impose their judgement on clients is most likely to elicit the client's a. acquiescence b. cooperation c. resistance d. appreciation

c. resistance

Establishing rapport with a client is essential in the initial stages of counseling. All of the following techniques are effective except a. moderate levels of self-disclosure b. admitting your mistakes c. sharing personal experience d. responding to a client's nonverbal messages

c. sharing personal experience

The unique focus of social work is on a. psychological aspects of life b. spiritual aspects of life c. social functioning of individuals, families, and groups d. ethical standards for practice

c. social functioning of individuals, families, and groups

In a social service agency, the designing of training programs and the implementation of staff development rests with the: a. support staff b. general administration c. supervisor d. company executive officer

c. supervisor ----person who serves in this role is expected to do this

A child who has been sexually abused asks her social worker to keep the abuse a secret. The social worker would: a. agree to keep the secret b. expose the secret without telling the child. c. tell her that secrets that hurt people need to be talked about openly d. ask the parents for guidance

c. tell her that secrets that hurt people need to be talked about openly

A hospital social worker receives definitive information that a social work colleague is engaged in a sexual relationship with a client. In deciding what to do, the social worker must first consider: a. the impact on the client b. the impact on the profession c. the Code of Ethics d. the views of the supervisor and administration

c. the Code of Ethics

A widower requests help from a social service agency for himself and his two young children. The man explains that his wife died three weeks ago and that he now needs childcare services for his children so that he can return to work. He is obviously in a state of grief and shock over the loss of his wife. The presenting problem in this situation is. a. the recent death of his wife b. the absence of supporting relatives c. the need for childcare services d. the need for employment

c. the need for childcare services

Empathy has been described as an attitude that engages a client in treatment. Which of the following best describes empathy? a. acknowledging what the client is saying b. accepting what the client is saying c. understanding what the client is saying d. judging what the client is saying

c. understanding what the client is saying

A social worker hears that a foster child has been placed in home where the child is being sexually abused by an older foster child in the home. The social worker's best or first course of action is to: a. remove the child b. remove the offending child c. visit the home for an assessment d. call the parents of the child being abused to warn them.

c. visit the home for an assessment----HEARS, therefore no removal at this time, must assess first to support the action of removal if necessary

When considering Native American customs and developing service plans, social workers should understand that: a. all Native Americans have the same culture and tribal differences are minor b. there are important reasons why Native Americans never integrated into American culture c. with more than 500 tribal groups, there are substantial cultural and social differences d. it is important for Native Americans to separate from traditional culture if they want to succeed economically

c. with more than 500 tribal groups, there are substantial cultural and social differences

You are completing the first session with a women recently separated from her significant other after 7 years of co-habitation which she was hoping would culminate in marriage. She has been separated for 6 weeks and she has met "the man of her dreams." She can't stop thinking about him or talking about him to her friends. You realize she is using a common ego defense mechanism. The ego defense mechanism BEST describing her situation is ... a.Distortion b.Projection c.Displacement d.Intellectualization

c.Displacement Displacement shifts sexual or aggressive impulses to a more acceptable or less threatening target. This allows the redirection of emotions to a safer outlet and also allows the separation of emotions from real objects. The redirection of the intense emotion toward someone or something that is less offensive and/or less threatening, with the benefit of avoiding the issue directly is very powerful. In this situation, she has displaced the sexual feelings she has for her ex-partner onto the new man in her life. It is safe to place them on him, while it is unsafe (due to rejection and emotional abandonment) to place her sexual feelings on her ex-partner.

A client whose mother dies following a long term illness states to the social worker that he believes that his angry thoughts about his mother caused the death. This is an example of (a) delusions (b) grandiosity (c) ideas of reference (d) magical thinking

d

A hospital social worker encounters a family that has just experienced the death of an infant due to sudden infant death syndrome (SIDS). The LEAST appropriate social work intervention with the family unit would be to: A offer sympathy for the family's loss B help family members share their grief C provide information on SIDS D make funeral arrangements

d

A withdrawn child in a residential treatment setting had progressed somewhat with a play therapy approach, but now seems disinterested. The social worker should A. processed with the treatment plan as though nothing has changed. B. transfer the case to a more experienced worker. C. seek a psychiatric evaluation. D. evaluate the treatment situation in supervision.

d

Q A client reacts to criticism with rage, is interpersonally explosive, has a grandiose sense of self, is preoccupied with thoughts of unlimited success, and has a sense of entitlement. This meets the criteria for which of the following DSM personality disorder diagnoses? A Paranoid B Antisocial C Narcissistic D Histrionic

d

The major goal of behavioral treatment of depression reflects the view that depression is the result of (a) role confusion (b) Negative cognition (c) poor interpersonal skills (d) absence of positive reinforcement

d

When critiquing a research article in a professional journal, a social worker should minimally be able to: A project the applicability of the findings to another group B incorporate the results into policy planning at his or her own agency C replicate the study in order to determine its reliability D evaluate the appropriateness of the research design

d

Which of the following statements is true of women who were victims of incest during childhood? A They are less protective of their children from potential abuse. B They disclose information about incestuous relationships openly in the course of treatment. C They all experience permanent damage as a result of abuse. D They are more likely to experience depression and anxiety.

d

All but one of the following are National Association of Social Workers (NASW) standards for cultural competence: a. Social workers should endeavor to seek out, employ, and retain employees who provide diversity in the profession. b. Social workers shall endeavor to resources and services in the native language of those they serve, including the use of translated materials and interpreters. c. Social workers should develop the skills to work with clients in culturally competent ways, and with respect for diversity. d. Social workers should work with diverse clients only if they have had specific training in that client's unique cultural background.

d. Social workers should work with diverse clients only if they have had specific training in that client's unique cultural background. Although it is ideal for social workers to receive specific training regarding each of the individual minority populations they typically serve, they should still ensure that someone from an unfamiliar background receives needed services even where no staff with special training in that background is available.

Parents continually refuse to communicate with their child who is in a residential treatment facility. The social worker should FIRST: a. respect the parent's need for separation by not contacting them b. request that the child be discharged c. contact the parents about the child's placement d. assess the child's response to the parents' actions

d. assess the child's response to the parents' actions

Which of the following approaches would be MOST appropriate to use in evaluating client satisfaction with the social worker's provision of services? a. using a single system design, evaluate the quality of services b. in the final interview, ask the client about their level of satisfaction c. after submitting process recordings, ask supervisor for feedback d. at termination, ask clients to complete a service evaluation and satisfaction form

d. at termination, ask clients to complete a service evaluation and satisfaction form

A client is referred to you due to ongoing depression. Your best intervention is: a. refer to a doctor for medication b. refer to a psychiatrist for medication c. refer to support groups d. begin individual counseling

d. begin individual counseling

A client is diagnosed with the eating disorder anorexia nervosa. To completely assess the client for comorbidity the social worker should also assess for the presence of: a. bulimia nervoas b. attention deficit hyperactivity disorder c. bipolar I disorder d. body dysmorphic disorder

d. body dysmorphic disorder

A social worker on a home visit meets with N., a 75-year-old female. N. reports that she has no family or friends to assist in her care. She also reports that she feels helpless to defend herself and believes that neighborhood children are stealing her belongings from her. Her home is cluttered, but the woman seems capable of finding things and noting where they were left. She appears somewhat malnourished; although she has food in the fridge, The neighbors have complained and have repeatedly called the police reporting that the woman is terrorizing the neighnorhood children by screaming at them from her yard. Based on the current information what is the BEST plan for the social worker to address this situation? a. arrange for home care b. call adult protective services c. help the woman to develop a plan to live alone d. do a complete assessment of the woman's competency to complete her own ADL's and live alone.

d. do a complete assessment of the woman's competency to complete her own ADL's and live alone.---not an immediate risk, but once assessment completed B may be appropriate choice.

A social worker has recently accepted a position at a local agency. In reading the agency policy hand book, all the following information can be found EXCEPT: a. the agency mission statement b. agency policy and procedures c. workload management issues d. employee recruitment information

d. employee recruitment information ---recruitment is not employment related.

A woman brings her husband to a busy emergency room because he is not feeling well. He dies while there. What should the social worker do? a. counsel the wife on grief and loss b. tell the wife that dying is part of life c. explain to the wife the stages of grief and loss d. find a room so that the wife can have privacy to deal with her feelings

d. find a room so that the wife can have privacy to deal with her feelings RU SAFE/ SFAREAFI B and C ignore the importance of discussing and addressing emotions. A is wrong because - We must give clients the respect and privacy to process a sudden death. -we must allow clients to feel their feelings before jumping into counseling, especially in this hospital setting.

A major criterion for a diagnosis of a substance abuse disorder involves a. occasional use b. an arrest record c. family violence d. impairment in social and/or occupational functioning due to use

d. impairment in social and/or occupational functioning due to use

In the past when a child has wet himself on the playground, the parents have immediately spanked him. From a behavioral perspective this is an example of: a. chaining response b. negative reinforcement c. unconditioned reinforcement d. punishment

d. punishment ---correct because they wanted the behavior to decrease NOT because the child was spanked

A social worker makes a home visit to a Cambodian family whose young child was diagnosed with CP. The worker found that the child waws kept in a darkened room and though fed, was isolated from the family, denied any affection, and seldom bathed. The grandfather wanted the child removed from the family. The social worker should first a. try to relate to the family to learn more about their values b. ask the mother to ask the grandfather to move out c. call Cambodian community agencies to ask for support. d. report the situation to child protective services

d. report the situation to child protective services

A social worker is receiving a client with long-standing depression. It becomes known that the client has racist beliefs. The social worker should: a. respectfully confront the client's prejudiced beliefs b. educate the client about the negative effects of racism c. refer the client to another therapist who might have similar belief d. respect the client's beliefs and focus on the presenting problem

d. respect the client's beliefs and focus on the presenting problem

Delusions, hallucinations and inappropriate affect are characteristics of a. narcissistic personality b. anxiety disorder c. antisocial personality d. schizophrenia

d. schizophrenia

People from Latino cultures tend to accept help from experts who: a. sound very knowledgeable and work for prestigious institutions b. have a reputation for being knowledgeable about a certain subject and are a relative or friend of the person in need c. are not Latino, but are highly credentialed d. speak Spanish, are knowledgeable about the area of concern, and have a good reputation in the community

d. speak Spanish, are knowledgeable about the area of concern, and have a good reputation in the community

Common indicators of sexual, physical and emotional abuse include all of the following EXCEPT: a. violence against younger children b. sexualized behavior c. isolation d. stuttering

d. stuttering

A fearful client in a mental health agency is repeatedly asked for money by his drug dependent son. Recently, the son has threatened violence and calls the father at all hours. How should the social worker advise the father? a. suggest he disconnect his phone b. ask the client to tell the son to come to the agency and discuss his problems with the social worker c. do nothing as it is likely that the behavior will stop on its own d. suggest that he has the right to be fearful and should probably call the police

d. suggest that he has the right to be fearful and should probably call the police

When seeing a client in therapy, from a biopsychosocial perspective a worker would most likely: a. focus on the individual client and what is reported b. avoid taking a history as it can bias new information c. focus only on the presenting problem d. take a complete history, including information about health concerns, family, employer, etc:

d. take a complete history, including information about health concerns, family, employer, etc:

14. Two social workers are in a crowded restaurant when one of the social worker's clients approaches them. What should the social worker do? a. pretend he doesn't know the client b. introduce the client as his friend to the other social worker c. tell the client to leave them alone d. tell the client he'll call him later

d. tell the client he'll call him later Always follow the client's lead in public settings and then kindly divert the attention away from communicating in public. Keep client confidentiality without shaming or harming a client's ego.

Of the following, which would be least important in the acculturation of an Asian girl in treatment with a social worker? a. the wishes of her parents b. the social worker's experience with similar situations c. the social worker's skills d. the girl's network of friends

d. the girl's network of friends Acculturation: assimilation to a different culture, typically the dominant one.


Conjuntos de estudio relacionados

RETIREMENT PLANS: EDUCATION + HEALTH

View Set

public speaking week 10 study guide

View Set

Chapter 1 Quiz - General Insurance

View Set

AP Psychology Major Figures in Psychology

View Set

Macro 2315 Final Exam practice questions Bishop

View Set